Vous êtes sur la page 1sur 268

Obstetrics and Gynecology

5th OBGYN Final Exam-2016 2


6th year Final exam- 2016 27

5th Year Final Exam 2015 53


6th Year Final Exam 2015 69

5th Year Final Exam 2017 86


6th Year Final Exam 2017 104

Page 1 of 124
Ehsan OBGYN Final Exam-2016
1. Regarding female urodynamic investigations, all are true except:
a. Flow rate during voiding is usually >15 mL/sec
b. Residual urine is usually <50 mL
c. Are important for the accurate diagnosis of urge and stress incontinence
d. Bladder capacity is usually between 200 and 300 mL
e. First desire to void usually occurs between 150 & 250 mL

Answer: d
Bladder capacity is between 400-600 mL
2. Regarding dizygotic twins, all are true except:
a. Have different genetic factors
b. Constant incidence worldwide
c. Same or opposite sex
d. Results from fertilization of two ova by two sperms
e. Have double or fused placenta

Answer: B
3. Regarding secondary amenorrhea, all are true except:
a. A high serum FSH levels indicate premature ovarian failure
b. Asherman’s syndrome is recognized cause
c. Pregnancy is the most common cause
d. May be caused by Sheehan syndrome
e. It is the absence of menses for more than six cycles interval if previously regular

Answer: E
It is the absence of menses for more than 3 months if previously regular, and 6 months if
previously irregular.
4. Fetal hyperinsulinemia causes increased growth of the following fetal tissue, except:
a. Brain
b. Liver

Page 2 of 124
c. Heart
d. Muscles
e. Subcutaneous fat

Answer: A
5. Risk factors for uterine rupture in labor include all of the following except:
a. Administration of oxytocin
b. Myomectomy
c. Previous caesarian section
d. Difficult forceps delivery
e. Amniocentesis

Answer: E
6. The most likely cause of acute abdominal pain immediately after an IUCD insertion is:
a. Uterine Perforation
b. Uterine contraction
c. Spontaneous expulsion of the device
d. Copper allergy
e. Pelvic inflammatory disease

Answer: A
7. Symptoms of menopause include all of the following except:
a. Infertility
b. Stress incontinence
c. Loss of libido
d. Breast enlargement
e. Dry vagina

Answer: D
8. Hydatidiform mole is characterized by all of the following except:
a. Hypothyroidism
b. Uterus larger than date

Page 3 of 124
c. Severe Hyperemesis

d. Theca lutein cysts of the ovary


e. Early onset preeclamsia
Answer: A
It is associated with hyperthyroidism is 7% of cases
9. The most common predisposing factor for breech presentation is:
a. Congenital uterine abnormality
b. Congenital fetal abnormality
c. Multiparity
d. Prematurity
e. Placenta previa

Answer: D
10. The earliest sign of placental abruption is:
a. Elevated INR
b. Skin ecchymosis
c. Hematuria
d. Intrauterine fetal death
e. Thrombocytopenia

Answer: E
11. All of the following may be considered in the management of urge incontinence,
except:
a. Intravesical therapy (e.g. Botulinum toxin)
b. Tolterodine and oxybutynin
c. Bladder training
d. Retro-pubic bladder neck suspension operations
e. Avoid caffeine and based drinks

Answer: D
Management of urge incontinence: *Cut down volume of fluid consumed – should
consume between 1 & 1.5 liters a day. * Avoid caffeine based drinks • Bladder training:
the patient is instructed to void on a timed schedule, starting with a relatively frequent
interval *Antimuscarinic drugs (e.g. tolterodine and oxybutynin)*

Page 4 of 124
Estrogen • Intra-vesical therapy (capsaicin, Botulinum toxin) •
Sacral nerve root neuromodulation • Surgery (cystoplasty, urinary
diversion) in refractory cases
12. All of the following can be diagnosed by ultrasound at 12 weeks
gestation except:
a. Polyhydromnios
b. Incomplete miscarriage
c. Ovarian cyst
d. Uterine fibroid
e. Triplets

Answer: A
13. Good antenatal care can prevent or minimize all of the following conditions, except:
a. Placenta previa
b. Iron deficiency anemia
c. RH immune reaction
d. Pyelonephritis
e. Fetal death due to placental insufficiency

Answer: A
14. Regarding epithelial ovarian cancers, one is true:
a. The majority of epithelial cancers are familial
b. CA 125 is not helpful in the follow up of ovarian cancer
c. Endometroid type 30% coexist with primary endometrial cancer
d. Usually present with acute symptoms
e. Borderline type carries worse prognosis than invasive type

Answer: C
15. All of the following should be routinely monitored during magnesium sulphate
infusion, except:
a. Blood pressure
b. Respiratory rate
c. Urine output
d. Pupil size
e. Deep tendon reflexes

Answer: D

Page 5 of 124
16. Cord entanglement and intrauterine fetal death occur most in
which of the following:
a. Conjoined twins
b. Monochorionic monoamniotic
c. Monochorionic diamniotic
d. Footling breech
e. Dichorionic diamniotic

Answer: B
17. Features of threatened miscarriage include all of the following except:
a. Ultrasound is essential for the diagnosis
b. About 60% of cases continue to term if fetal heart was seen by U/S
c. The uterine size is usually corresponding to date
d. The cervix is closed
e. The typical presentation is mild vaginal bleeding with little or no pain

Answer: B
18. Complications of postdate include all of the following except:
a. Respiratory distress syndrome
b. Fetal hypoxia
c. Shoulder dystocia
d. Oligohydromnios
e. Meconium aspiration

Answer: A
19. Polycystic ovarian syndrome is characterized by all of the following except:
a. Obesity
b. Increased androgen levels
c. Oligomenorrhea
d. Increased LH:FSH ratio
e. Decreased estrogen levels

Answer: E
20. Regarding ectopic pregnancy, one is true:
a. Expectant management is possible in selected cases
b. Pelvic mass almost always present

Page 6 of 124
c. Commonest in the left tube than the right tube
d. A positive pregnancy test and absent intrauterine sac is
diagnostic for ectopic pregnancy
e. Recently most cases are silent

Answer: B
21. One of the following is a malignant germ cell tumor of the ovary:
a. Thecoma
b. Androblastoma
c. Mucinous carcinoma
d. Dysgerminoma
e. Mature cyst teratoma

Answer: D
22. Regarding hydatidiform mole, one is true:
a. Preeclampsia in the most common symptom and it occurs in 70% of patients
b. Fetal parts are present
c. Diagnosis is confirmed with very low levels of human β gonadotropin
d. Risk of malignancy is 15-20%
e. Prophylactic chemotherapy should be given

Answer: D
23. During labor, all are true except:
a. The risk of malposition is increased in patients with android pelvis
b. Patients on oxytocin should be monitored with CTG
c. The normal blood loss after vaginal delivery is 500-750 mL
d. The head is engaged when the station is more than zero
e. Active management of the third stage will decrease the risk for PPH

Answer: C
24. All of the following are risk factors for postpartum hemorrhage, except:
a. Cesarean delivery
b. Uterine over distention
c. Uterine fibroids
d. Previous PPH
e. Platelets count of 100,000

Page 7 of 124
Answer: E
25. Regarding recurrent miscarriages, all are true except:
a. Defined as 3 or more consecutive induced abortions
b. Occurs in about 1% of women
c. Can be caused by thrombophilia
d. Can be caused by uterine anomalies
e. Is idiopathic in about 50% of cases

Answer: A
26. All of the following are indications for delivery in preterm premature rupture of
membrane, except:
a. Twin pregnancy
b. Severe abruptio placenta
c. Fetal distress
d. Chorioamnionitis
e. Reaching 37 weeks gestation

Answer: E
27. All are true about RH- isoimmunization, except:
a. Intrauterine blood transfusion is one of the treatment options
b. Fetal hydrops is a recognized fetal complication
c. Middle cerebral artery Doppler is used in the follow up of affected fetus
d. Unlikely to happen in the first pregnancy
e. Anti- D should be given to all sensitized patients

Answer: E
28. Regarding Rokitansky syndrome, all are true except:
a. The patient is in need for supplement hormones
b. Presents with primary amenorrhea
c. Associated with renal and skeletal abnormalities
d. Is the congenital absence of the vagina and uterus
e. Presents with normal breast development and normal axillary and pubic hair

Answer: A
29. All of the following can be sexually transmitted, except:

Page 8 of 124
a. Trichomonas vaginalis
b. Chlamydia
c. Vaginal candidiasis
d. HIV
e. Gonorrhea

Answer: C
30. Genital organs are supported by all of the following except:
a. Transverse cervical ligaments
b. Pelvic fascia
c. Levator ani muscle
d. Round ligament
e. Uterosacral ligament

Answer: D
31. Regarding hyperemesis gravidarum, all are true except:
a. Is typically associated with hypertension
b. Is less common after 12 weeks of gestation
c. More common in twin pregnancy
d. May be associated with abnormal thyroid function test
e. In severe cases may be complicated with Wernicke’s encephalopathy

Answer: A
32. G2P2 had cesarean section 1 week ago, presented with fever. The differential
diagnosis include all of the following except:
a. Mastitis
b. Lung atelectasis
c. DVT
d. Urinary tract infection
e. Infected C/S wound

Answer: B
33. A 25 year old pregnant presented for routine antenatal care at 10 weeks gestation,
the following investigations should be done for her, except:
a. Toxoplasma IgG
b. Rh- state

Page 9 of 124
c. Obstetric ultrasound
d. Rubella IgG
e. Urine analysis

Answer: A
34. Regarding endometroid endometrial carcinoma, one of the following statements is
true:
a. Estrogen dependant
b. Occurs mainly before the age of 50
c. Usually presents with pelvic pain
d. Usually advanced stage at the time of diagnosis
e. The diagnosis is confirmed by increased endometrial thickness on ultrasound

Answer: A
35. Regarding stress incontinence, all are true except:
a. Involuntary leakage of urine occurs with sneezing, coughing and laughing
b. Is the most common type of incontinence
c. Is best treated with anticholinergic medications
d. During cytometer, there are no uninhibited detrusor contractions during filling
e. Predisposing factors include obesity and vaginal delivery

Answer: C
36. Regarding chlamydia trachomatis infection, all of the following are true except:
a. Is a common cause of vaginal discharge
b. May predispose to perihepatic adhesions
c. May be treated with erythromycin or tetracycline
d. Is a gram negative intracellular organism
e. Can cause neonatal pneumonia

Answer:
37. All of the following are used for pain relief in labor except:
a. Pedundal block
b. Epidural
c. Narcotics
d. Nitrous oxide
e. Diazepam

Page 10 of 124
Answer: A
Answered by Dr. Elias
38. The denominator in face presentation is:
a. Occiput
b. Chin
c. Bregma
d. Nose
e. Anterior fontanel

Answer: B
39. Which of the following should always be stopped during pregnancy:
a. Metoclopromide
b. Metformin
c. Tricyclic antidepressant
d. Isotretenoin
e. Propanalol

Answer: D
40. All of the following are absolute contraindications for combined oral contraceptive
pills, except;
a. Breast cancer
b. Previous pregnancy while on pills
c. Previous DVT
d. Thrombophilia
e. Liver adenoma

Answer; B
41. Regarding growth restriction, only one of the following is true:
a. May be caused by dieting in pregnancy
b. Usually occurs after 35 weeks of gestation
c. Could be a presentation of chromosomal abnormalities
d. Usually associated with polyhydromnios
e. Head circumference is the ideal parameter to detect growth restriction

Answer:

42. All of the following drugs are matched correctly with their side effects except:
a. Hydralazine …. Bradycardia

Page 11 of 124
b. Magnesium sulphate …. Respiratory depression
c. Methyl dopa ….. postpartum depression
d. Nefidipine ….. headache
e. Indomethacin ….. premature closure of ductus arteriosus

Answer: A
43. Features of missed abortion include all of the following except:
a. Most cases are diagnosed accidentally during routine u/s examination
b. Ultrasound is essential for the diagnosis
c. The best method of treatment in the 2nd trimester is medical termination
d. The uterus may be small for date
e. Means death of the fetus before 28 weeks of gestation

Answer: E
44. The infectious agent that is involved in the development of cervical cancer is:
a. Herpes virus
b. Human papilloma virus
c. Syphilis
d. Gonorrhea
e. Chlamydia

Answer: B
45. Regarding trichomonas vaginalis, all are true except:
a. Presents with green vaginal discharge
b. Complications during pregnancy include low birth weight and prematurity
c. KOH wet mount shows motile flagellated microorganisms
d. Infection during pregnancy can cause neonatal pneumonia
e. Male partner should be treated

Answer: D
46. Regarding fibroid presentation, all of the following are known symptoms except:
a. Menorrhagia
b. Intermenstrual bleeding
c. Subfertility

Page 12 of 124
d. Asymptomatic
e. Urinary frequency

Answer: B
47. Regarding IVF, all of the following are indications except:
a. Absent or blocked fallopian tubes
b. Ovarian failure
c. Severe male factor infertility
d. Severe endometriosis
e. Infrequent ovulation after failed treatment with other therapies

Answer: B
48. Regarding mechanism of labor, all of the following are true except:
a. The presenting part enters the pelvic brim in a transverse diameter
b. In multipara engagement usually takes place during labor
c. The posterior shoulder delivers first
d. The fetal head delivers in extension
e. The internal rotation indicates rotation of the fetal head in the AP- diameter

Answer: C
49. The following conditions may be associated with diabetes, except:
a. Hypertension
b. Postmaturity
c. Fetal encephalocele
d. Ventricular septal defect
e. Fetal anal atresia

Answer: B
50. Symptoms of endometriosis include all of the following except:
a. Dysmenorrhea
b. Dyschezia
c. Chronic pelvic pain
d. Vaginal discharge
e. Dyspareunia

Page 13 of 124
Answer: D

51. All of the following are used in the treatment of endometriosis,


except:
a. NSAIDs
b. COCPs
c. Oral progesterone
d. Conjugated estrogen
e. GnRH analogue
Answer: D
52. Meigs syndrome is associated with the following ovarian neoplasm:
a. Brenner tumor
b. Non- gestational choriocarcinoma
c. Yolk sac tumor
d. Serous adenoma
e. Fibroma

Answer: E
53. Twins pregnancy may be complicated by all of the following except:
a. Placenta previa
b. Post-term labor
c. Polyhydromnios
d. Preeclampsia
e. Malpresentation

Answer: B
54. PG1, presented at 37 weeks in labor, CBC: Hb 10 g/dL, WBCs 5*10^9 /L, platelets
100000/ μL, she’s not known to have any medical illnesses, which statement is true:
a. Epidural anesthesia is safe in this patient
b. Platelet transfusion is required if platelets <80000 / μL
c. The most likely cause is ITP (immune thrombocytopenia purpura)
d. She has pancytopenia and should receive prednisolone
e. The patient should be delivered by cesarean section

Answer: A
55. Splitting of a single fertilized ovum between 8-12 days results in:
a. Conjoined twins
b. Monochorionic diamniotic twin

Page 14 of 124
c. Dichorionic diamniotic
d. Dizygotic twin
e. Monochorionic monoamniotic

Answer: E
56. All of the following are true about abdominal pain in pregnancy except:
a. Abruptio placenta can present with abdominal pain only
b. Appendicitis id the commonest surgical cause of abdominal pain
c. Gallbladder stones can cause pancreatitis
d. Peptic ulcers usually get worse during pregnancy
e. Complicated ovarian cyst need immediate surgical intervention

Answer: D
Peptic ulcers actually improve during pregnancy
57. In a patient presented with moderate vaginal bleeding at 36 weeks, all of the
following in favor of placenta previa except:
a. Tender uterus
b. History of bleeding in early pregnancy
c. Previous cesarean delivery
d. Transverse lie
e. Good fetal condition

Answer: A
58. All of the following are recognized risk factors for fibroids except:
a. Family history
b. History of uterine surgery
c. Black race
d. Null parity
e. High estrogen levels

Answer: B
59. Regarding vulvovaginal candidiasis, all of the following are true except:
a. Less common in combined oral contraceptive pill users
b. The discharge is thick, cottage cheese like
c. It causes erythema with excoriation

Page 15 of 124
d. Main symptom is pruritus
e. KOH wet mount shows budding yeast cells and pseudohyphae

Answer: A
60. Premature menopause may be related to all of the following, except:
a. Radiotherapy
b. Autoimmune disorder
c. Endometriosis
d. Mumps
e. Bilateral salpingoopherectomy

Answer: C
61. All of the following are complications of severe preeclampsia, except:
a. Polyhydromnios
b. Intrauterine growth restriction
c. Intracranial bleeding
d. Abruption
e. Renal failure

Answer: A
62. A previously pregnant woman complains of crying, loss of appetite, difficulty sleeping
and feeling of low self-worth 3 days after she had her delivery and this feeling lasted for 1
week. Which term best describes her postpartum symptoms:
a. Blues
b. Major depression
c. Psychosis
d. Schizoaffective disorder
e. **

Answer: A
63. The following are causes for large for date uterus, except:
a. Multiple pregnancy
b. Transverse lie
c. Full bladder
d. Polyhydromnios
e. Wrong date

Page 16 of 124
Answer: B
64. HRT is beneficial in the management of patients with all of the
following except:
a. Mood change
b. Vaginal atrophy
c. Coronary heart disease
d. Flushing
e. Osteoporosis

Answer: C
65. The following are biophysical profile components, except:
a. Fetal weight
b. Fetal heart pattern
c. Fetal tone
d. Liquor volume
e. Fetal breathing movement

Answer: A
66. Regarding asthma in pregnancy, all of the following are true except:
a. Salbutamol inhalers should be avoided
b. It is underdiagnosed
c. Uncontrolled cases may predispose to intrauterine growth restriction
d. Pregnancy doesn’t increase the frequency of the attacks
e. Steroid inhalers are safe

Answer: A
67. In a 20 year old single complaining of severe dysmenorrhea because of
endometriosis, the treatment of choice is:
a. Continuous oral progesterone
b. Mirena
c. Cyclical COCP
d. GnRH analogue
e. Bilateral oophorectomy

Answer: A
68. Regarding coagulation disorders in pregnancy, one statement is true:

Page 17 of 124
a. Treatment of proven DVT in pregnancy should be started with
heparin and then shifted to warfarin
b. Superficial thrombophlebitis is treated by bed rest, analgesia and local heat
c. Warfarin is excreted in breast milk
d. Heparin crosses the placenta and should be avoided in the 1st trimester
e. D-dimer test is important in the diagnosis of DVT in pregnancy

Answer: B
69. The ischial spine of the mother is the reference point used to determine which of the
following:
a. Fetal attitude
b. Presentation
c. Fetal engagement
d. Lie
e. Fetal position

Answer: C
70. Regarding ectopic pregnancy, only one of the following is true:
a. The commonest presentations are acute
b. Previous ectopic is a recognized risk factor
c. Missed period is always present
d. Ultrasound and β-HCG are diagnostic
e. A positive urine pregnancy test is found in half of cases

Answer: B
71. The following factors affect normal fetal weight except:
a. Parity
b. Race
c. Maternal built
d. Climate and weather
e. Fetal sex

Answer: D
72. Initial assessment of patient presented with APH at 36 weeks of gestation include all
of the following except:
a. Uterine contractions

b. CBC

Page 18 of 124
c. Vital signs
d. CTG
e. Vaginal examination
Answer: E
73. Majority of women presenting with placental abruption have:
a. Chorioamnionitis
b. Blunt trauma to abdomen
c. Preeclampsia
d. No evident factor
e. Multiple pregnancy

Answer: D
74. All of the following are contraindications for tocolytics except:
a. Active labor
b. Fetal abnormalities
c. Antepartum hemorrhage
d. Fetal compromise
e. Chorioamnionitis

Answer:
75. Regarding the accurate estimation of gestational age, all of the following are true
except:
a. The ultrasonic estimation at 32 weeks of gestation is usually inaccurate
b. In the presence of lactation, ultrasonic estimation is recommended
c. It is best done by measuring the crown rump length in the second trimester
d. It is calculated from the first day of the last menstrual period
e. It reduces the risk of post-maturity

Answer: A?
76. The initial evaluation of an infertile couple include: a. Semen analysis .

b. Femal karyotyping.

Page 19 of 124
c. Ovarian biopsy .
d. Hysteroscopy and D&C.
e. Antisperm antivodies assay.
Answer: A

77. Regarding cervix cancer , all of the following are true except :

a.Multiparous women have an increase risk .


b.Smoking is a co-factor in the etiology of the disease.
c. Pap smear is only a screening test for the disease.
d. Squamous carcinoma is the most common cell type.
e. Rarley occur before the age of 40.
Answer: B

78. The most important risk factor for preterm labor is :


a. Previous preterm delivery.
b. Bacterial vaginosis.
c. Age < 19
d. Cervical incompetence.
e. Preterm premature rupture of membrane.
Answer: A
79. Regarding cervical ectropion, all of the following are true except :
a. It is commonly seen in post-menopausal women.
b. Cervical smear is mandatory before treatment.
c. Post coital bleeding is a common presentation.
d. Can be caused by combines contraceptive pills.
e. Excessive mucoid secretion is a common presentation.

Answer: D
80. Regarding diabetes with pregnancy, only one of the following statement is true:
a. Strictly controlled diabetes will prevent development of fetal macrosomia.
b. Sudden fetal death may occur with well controlled diabetes.
c. Excessive fetal growth usually starts around the 20th week of gestation.
d. Neonatal morbidity among babies of diabetic mother is around 40%.

Page 20 of 124
e. Risk of congenital malformations is significantly increased in
gestational diabetes.

Answer: B
81. Intrapartum fetal tachycardia may be caused by all of the following except :
a. Prematurity
b. Maternal fever.
c. Chorioamnionitis.
d. Fetal anemia.
e. Pethidine.

Answer: E
82. Regarding folate deficiency anemia in pregnancy, all of the following are risk factor
except :
a. Anticonvulsant therapy.
b. Thalassemia trait.
c. Meckel gruber syndrome.
d. Haemolytic anemia.
e. Hereditary spherocytosis.

Answer : C
83. Regarding epilepsy in pregnancy, all of the following statement are true except :
a. Monotherapy is the gold standard.
b. Breast feeding is not contraindication.
c. Risk of congenital abnormality is around 3%.
d. 1/3 of the patients will experience an increase in seizure frequency.
e. Folic acid supplement is indicated.

Answer: B

84. In a patient who had forceps delivery with complete delivery of the placenta and
developed sever bleeding with dots, uterus was contracted, what is the most common
of bleeding :
a. Uterine inversion.
b. Uterine atony.
c. Coagulation defect.
d. Placenta previa.
e. Genital tract trauma.

Page 21 of 124
Answer: E
85. Regarding placenta previa, all of the following are true except :
a. Associated with increase risk of hysterectomy.
b. Often causes sever PPH.
c. Typically presents with painless vaginal bleeding.
d. Vaginal ultrasound causes bleeding.
e. Accrete is associated with increase maternal mortality.

Answer:
86. All of the following are progesterone only contraception except:
a. Norplant.
b. Implanol.
c. Mirena coil.
d. Lippes loop .
e. Depo-provera.

Answer: D
87. Implantation in the uterus occur at which stage of development :
a. Blastocyst.
b. Zygote.
c. Morula formation .
d. Embryonic disk.
e. Eight-cell embryo.

Answer: A

Page 22 of 124
88. Pregnant 30 weeks, has one twin vaginal delivery, one breach
delivery and one ectopic pregnancy, what is her gravidity and
parity ?

A. G5P2+1.
B. G4P2+1.
C. G5P3+1.
D. G4P2.
E. G3P2+1.

Answer: B
89. Induction of labor is indicated in all of the following except :
a. Diabetes mellitus.
b. Abnormal antenatal CTG.
c. Chorioamnionitis.
d. Intrauterine fetal death.
e. Growth restriction.

Answer: B
90. In eclampsia, which is true ?
a. Diuretic drugs are essential in all cases.
b. Always occur intrapartum.
c. Urinary output is increased.
d. Ergometrine should be avoided in the third stage of labor.
e. Cesarean section must be carried out in all cases.

Answer: D
91. Which one of the following has the highest rate of progression to uterine cancer?
a. Uterine leiomyoma.
b. Simple hyperplasia with atypia.
c. Complex hyperplasia.
d. Complex hyperplasia with atypia.
e. Simple hyperplasia.
Answer: D
92. The peak secretion of progesterone in 28 days cycle occurs on :
a. 21st day of cycle.
b. 7th day of cycle.

Page 23 of 124
c. 14th day of cycle.
d. 36 hours before ovulation.
e. 1st day of cycle.

Answer: A
93. Epidural anesthesia is recommended in all the following conditions except:
a. Delivery of twins.
b. Complete breach delivery.
c. Maternal cardiac disease.
d. Platelet count less than 80,000.
e. Primigravida in labor.

Answer: D
94. Which of the following procedures assesses both uterine cavity and tubal lumen:
a. HSG .
b. Laparoscopy.
c. Hysteroscopy .
d. Ultrasound.
e. Colposcopy.

Answer: A
95. Regarding menorrhagia, all of the following statements are true except:
a. Adenomyosis is a recognized cause
b. Don’t occur in ovulatory cycle.
c. Could be due to ineffective contraction of myometrium vessels.
d. Can be treated by endometrial ablasion.
e. Is defined as prolonged and regular menstrual bleeding.

Page 24 of 124
Answer: C
96. Factors that increase the risk of genital prolapse include all of the
following except:
a. Forceps delivery.
b. Hysterectomy.
c. Cesarean section.
d. Caucasian race.
e. Chronic cough.

Answer: C
97. Regarding breast feeding, all are true except:
a. Reduce the risk of breast cancer.
b. Suckling stimulate uterine involution.
c. Breast engorgement causes fever within 24 hour after delivery if no milk pumping.
d. The predominant immunoglobulin is IgA.
e. Colostrum contains more protein than breast milk.

Answer: C
98. The following are risk factor for uterine atony except:
a. Magnesium sulfate.
b. Chorioamnionitis.
c. Prolonged labor .
d. Nifedipine.
e. Intrauterine growth restriction.

Answer: E
99. All of the following are risk factor for endometrial carcinoma except:
a. Obesity.
b. Unopposed estrogen therapy.
c. Late menopause.
d. Tamoxifen therapy.

e. Multiparity
Answer: E
100. The most common etiology factor of postpartum mastitis is:
a. Aeropic streptococcus.
b. Neisseria.
c. E.coli.

Page 25 of 124
d. Anaerobic streptococcus.
e. Staphylococcus aureus.

Answer :E

Page 26 of 124
Obstetrics and Gynecology
6th year Final exam- 2016
1. Regarding breastfeeding, all the following statements are correct except

a. Suckling stimulates the release of oxytocin

b. Breast engorgement causes fever within 24 hours after delivery

c. Colostrum contains more protein than breast milk

d. Reduces the risk of breast cancer

e. The predominant Immunoglobulin is IgA which provides protection in infants

Answer: Breast engorgement causes fever within 24 hours after delivery

2. Regarding ectopic pregnancy; only one of the following statements is correct

a. Bleeding usually precedes abdominal pain

b. Most patients have gradually increasing intermittent abdominal pain

c. A ruptured corpus luteum cyst may cause a similar clinical picture

d. Vaginal bleeding is essential for diagnosis

e. Recurrence in future pregnancy is 25%

Answer: A ruptured corpus luteum cyst may cause a similar clinical picture

*Recurrence in future pregnancy according to the slides is 10-20%

3. Regarding cervical incompetence, all the following are true except

a. Can be caused by multiple pregnancy

b. Can be diagnosed by ultrasound

c. Is a cause of preterm labour

d. Can be caused by cone biopsy of the cervix

e. Best treated in the 2nd trimester of pregnancy

Page 27 of 124
Answer:

Statements b-e are definitely correct. Choice A is probably the wrong


answer

4. A 32 weeks pregnant female has a Hb of 9 on a routine antenatal assessment. Which of


the following is the best step regarding management of her anemia?

a. Give blood transfusion

b. Start intramuscular iron

c. Give IV iron

d. No action needed

e. Treat with oral iron and vitamins

Answer:

*Probably the answer is e. Treat with oral iron and vitamins

5. A 30 year old primigravida is admitted with labor pains. On examination her cervix is 2
cm dilated and 1 cm long. Name the stage of labor in which she is in

a. Active phase of labour

b. Second stage of labour

c. Latent phase of labour

d. Late first stage of labour

e. Not in labour

Answer: Latent phase of labor

6. All of the following are correct except

a. Growth restricted fetus should be monitored with continuous CTG

b. Active management of the third stage reduces PPH

c. When only 1/5th of the head is palpable, the head is considered engaged

Page 28 of 124
d. With active management the normal length of the 3rd stage is 2 hours

e. The risk of malposition increases in patients with android pelvis

Answer: With active management the normal length of the 3rd stage is 2 hours

7. A primigravid female had vaginal delivery 10 days ago and now presents with foul
smelling discharge, fever and vaginal bleeding. She is hemodynamically stable. Ultrasound
shows retained products of conception. Your next step of management is

a. Immediate evacuation followed by antibiotics

b. Antibiotics for 5 days and review later

c. IV antibiotics followed by evacuation

d. Wait results of HVS culture to start antibiotics

e. Immediate evacuation and send products for culture and sensitivity

Answer: IV antibiotics followed by evacuation

8. Which of the following statements regarding malpresentation and malposition are


incorrect

a. Persistent occipito-posterior position in the second stage of labor is delivered by cesarean


section

b. Footling breach are best delivered by cesarean section

c. The use of vacuum is contraindicated in face presentation

d. Both polyhydramnios and oligohydramnios increase the risk of breech presentation

e. Traction forceps are used for delivery of the after-coming head in breech presentation

Answer:

*Check the clinical skills lab note. It might give an idea of the answer

9. About small for gestational age fetus, all of the following are true except

a. Best diagnosed by ultrasound

Page 29 of 124
b. Should be delivered before 37 weeks

c. Approximately half of the cases are missed on routine abdominal


palpation

d. May be associated with fetal abnormalities

e. Is a recognized consequence to maternal cigarette smoking

Answer: Should be delivered before 37 weeks

10. All of the following are causes of fever and chills 7 days after delivery except

a. Nipple cracks

b. Endomyometritis

c. Breast abscess

d. Mastitits

e. Pyelonephritis

Answer: Nipple cracks

11. Which of the following statements regarding contraception in the postpartum period is
correct

a. COCP is a good choice during breastfeeding

b. IUCD is contraindicated

c. Contraception usually starts 4-6 weeks after delivery

d. Progesterone only pills is contraindicated in breastfeeding

e. Lactation only is adequate contraception

Answer: Contraception usually starts 4-6 weeks after delivery

12. Which of the following statements is not true about Rh isoimmunization

a. Middle cerebral artery Doppler is used in the followup of treated fetus

b. Intrauterine blood transfusion is one treatment option


Page 30 of 124
c. Fetal hydrops is one of the fetal complications

d. Anti-D should be given to all sensitized patients

e. Sensitization can occur if patient didn’t take Anti-D after delivery of Rh-positive baby

Answer: Anti-D should be given to all sensitized patients

13. Which of the following is true regarding uterotonic agent administration the active
management of the third stage of labor

a. 5-10 IU oxytocin after delivery of the placenta

b. 5-10 IU oxytocin after delivery of the baby

c. Oxytocin infusion at crowning till delivery of the placenta

d. Ergometrine IV with anterior shoulder delivery

e. 5-10 IU oxytocin after delivery of the anterior shoulder

Answer: 5-10 IU oxytocin after delivery of the anterior shoulder

14. All of the following are common symptoms of urinary incontinence except

a. Stress incontinence

b. Overflow incontinence

c. Urge incontinence

d. Frequency

e. Straining to void

Answer: Straining to void

15. All of the following are symptoms of menopause except

a. Hot flashes

b. Loss of weight

c. Insomnia

Page 31 of 124
d. Anxiety

e. Genital prolapse

Answer: Loss of weight

16. Which of the following is true about epidural analgesia in labor?

a. It is contraindicated in severe IUGR

b. It is contraindicated in females with a previous history of DVT

c. Induced hypotension can cause fetal heart changes

d. Should not be administered during the second stage of labour

e. Associated with an increase in rate of cesarean delivery

Answer:

*Probably: induced hypotension can cause fetal heart changes

Epidural analgesia may increase risks of instrumental delivery but not cesarean section

17. A primigravid female presents to the office for her first prenatal visit. She is 12 weeks
pregnant. All of the following investigations are ordered except

a. HbsAg

b. Rubella IgG

c. Urine culture

d. Glucose tolerance test

Answer: Glucose tolerance test

18. Which of the following is the best reason for the treatment of high blood pressure in
pregnancy?

a. Prevent maternal complications such as stroke

b. Lower the incidence of pre-eclampsia

c. Lower the incidence of oligohydramnios


Page 32 of 124
d. Lower the incidence of intrauterine demise

Answer: Prevent maternal complications such as stroke

19. Which of the following statements is incorrect about congenital anomalies

a. Tracheo-esophageal fistula is associated with oligohydramnios

b. Neural tube defects are associated with banana sign

c. Anencephaly is associated with polyhydramnios

d…..

Answer: Tracheo-esophageal fistula is associated with oligohydramnios

20. Which of the following is true about diabetes in pregnancy

a. Risk of polyhydramnios is lower if it diabetes is controlled

b. OGTT should be performed if glycosuria occur before 16 weeks of gestational age

c. The best way to control diabetes is with two doses of soluble insulin

d. During labor serum glucose must be maintained between 6-8 mmol/L

e. If the patient’s HbA1c is 11%, the pregnancy must be aborted

Answer:

21. Regarding dysfunctional uterine bleeding, which of the following statements is


incorrect?

a. Can be ovulatory

b. Occurs only in menopausal females

c…

d…

Answer: Occurs only in menopausal females

22. Regarding abdominal pain in pregnancy, which of the following is false

Page 33 of 124
a. Peptic ulcer usually deteriorates in pregnancy

b. Ovarian torsion requires immediate surgical intervention

c. Appendicitis is the most common cause

d. Gallstones may cause pancreatitis

Answer: Peptic ulcer usually deteriorates in pregnancy

23. Regarding menopause which of the following statements is incorrect

a. Average age is 55

b. Delayed menopause is a risk factor for endometrial cancer

c. Smoking induces early menopause

d…

e…

Answer: Average age is 55

24. Medical treatment of missed miscarriage is with

a. Oxytocin infusion IV

b. Prostaglandin F2 alpha IV

c. Misoprostol vaginally

d. Ergometrine

Answer: Misoprostol vaginally

25. A couple presented with infertility, semen analysis shows a count of 14 million, motility
20% and morphology of 24%. The most appropriate action is

a. Varicocolectomy

b. Insemination with donor sperm

c. Clomiphene citrate

Page 34 of 124
d. IVF

e Intrauterine insemination with washed husband sperm

Answer:

26. Pregnant lady at 37 weeks gestation with symphysis fundal height of 34 cm, all of the
following are possible causes except

a. Uncertain dates

b. Oligohydramnios

c. Transverse lie

d. Estimated fetal weight of 4000 grams

e. PROM

Answer: Estimated fetal weight of 4000 grams

27. The main adverse outcome associated with urinary tract infection is

a. Intrauterine growth restriction

b. Intrauterine fetal death

c. Septicemia

d. Anemia

e. Preterm labor

Answer:

*Probably answer is preterm labor

28. Not a sign of chorioamnionitis

a. Fever

b. Tachycardia

c. Abdominal tenderness

Page 35 of 124
d. Hyper-reflexia

Answer: Hyper-reflexia

29. In case of the need for a blood transfusion with the inability to cross-match. Units of
which of the following blood groups must be administered

a. O Rh negative

b. A Rh negative

c. A Rh positive

d. O Rh positive

e. B Rh negative

Answer: O Rh negative

30. Combined oral contraceptives are contraindicated in all of the following except

a. Migraine headache with focal headache

b. Severe liver cirrhosis

c. IHD

d. Diabetes mellitus

Answer: Diabetes mellitus

31. A 41 year old female who is a smoker, presents complaining of DUB. All of the following
can be lines of management except

Answer: Combined oral contraceptives

32. A 165 cm tall female, which of the following is the most likely type of pelvis?

a. Gynecoid

b. Android

c. Platypeloid

d. Anthropoid
Page 36 of 124
Answer:

33. Combined oral contraceptives decrease the risk of all of the


following except

a. Benign breast disease

b. Salpingitis

c. Colorectal cancer

d. Endometrial cancer

e. Cervical cancer

Answer: Cervical cancer

*No idea about colorectal cancer

34. All of the following are contraindications for tocolysis except

a. Transverse lie

b. A women in labor with 7 cm dilation

c. Abruptio placenta

d. Chorioamnionitis

Answer: Transverse lie

35. A young female patient presents with foul smelling discharge and abdominal pain. She
is diagnosed as a case of acute PID. All of the following are possible steps in the
management except

a. Oral clindamycin

b. Oral doxycycline

c. Hysteroscopy with D and C

d. Laparoscopy

Answer: Hysteroscopy with D and C

Page 37 of 124
36. All of the following are true about genital prolapse except

a. It is found in 25% of multiparous females

b. It is the main cause of stress incontinence

c. Procidentia is prolapse of the uterus through the introitus

d. More common in Asians than Blacks

Answer: It is the main cause of stress incontinence

37. Which of the following is true about epithelial ovarian cancer

a. If diagnosed in early stages it has a good prognosis

b. Serous has a better prognosis than endometroid

c. Oral contraceptives are not a protective factor

d…..

Answer:

38. A young patient who is infertile due to hypogonadotropic hypogonadism. What is the
best treatment for her infertility?

a. Clomiphene citrate

b. LH and FSH

c….

Answer: LH and FSH

39. All of the following are true about a patient with androgen insensitivity syndrome
except

a. The patient will have normal external male genitalia

b. Patient will have a karyotype with XY

c. It is an androgen receptor mutation

d…
Page 38 of 124
Answer: The patient will have normal external male genitalia

40. Which of the following statements about primary amenorrhea is


false

a. Primary amenorrhea is absence of menses and secondary sexual characteristics at the age
of 16

b. Gonadal dysgenesis is the most common cause of primary amenorrhea

c. A karyotype is the next step in a patient with elevated FSH and normal breast development

d..

Answer: Primary amenorrhea is absence of menses and secondary sexual characteristics at


the age of 16

41. All of the following have a risk of recurrence except

a. Gestational diabetes

b. Pre-eclampsia

c. Placental abruption

d. Shoulder dystocia

e. Cord prolapse

Answer: Cord prolapse

42. Which of the scenarios is a patient with severe pre-eclampsia

a. Patient with BP 150/100 and +1 proteinuria

b. Patient with BP 140/100 and +2 proteinuria

c. Patient with BP 140/100, +1 proteinuria and severe headache

d. Patient with BP 150/100 and 2g in 24 hour urine collection

Answer: Patient with BP 140/100, +1 proteinuria and severe headache

43. All of the following indicate severe pre-eclampsia except

Page 39 of 124
a. Pulmonary edema

b. Oligohydramnios

c. Headache and neurological symptoms

d. Renal failure

Answer: Oligohydramnios

44. Primigravida with 7 weeks of amenorrhea presents with mild abdominal pain and
spotting and a positive pregnancy test. Abdominal US shows no gestational sac. She is
hemodynamically stable. What is the best next step of management?

a. Order a serum b-hCG

b. Transvaginal US

c. Laparoscopy

d. Methotrexate

Answer:

45. The shortest distance between the sacral promontory and the lower edge of the
symphysis pubis is represented by which of the following?

a. Obstetric conjugate

b. Diagonal conjugate

c. True conjugate

d…

Answer: Diagonal conjugate

46. A 38 week pregnant female presents with abdominal pain and moderate vaginal
bleeding. Her cervix is 5 cm dilated. CTG shows mild contractions and obstetric US is
normal. What is the next best step of management?

a. Emergency CS

b. AROM + oxytocin

Page 40 of 124
c. Prostaglandins

d. Expectant management

Answer:

47. All of the following drugs are teratogenic in pregnancy except

a. Low molecular weight heparin

b. Warfarin

c….

Answer: Low molecular weight heparin

48. Delivery in a patient with PPROM is indicated in all of the following except

a. Malpresentation

b. Chorioamnionitis

c. Reached term

d. Fetal distress

Answer: Malpresentation

49. A patient 35 weeks pregnant presents to the clinic for a followup visit. On US her fetus
is in a breech position. Which of the following is the next best step of management?

a. Schedule a repeat visit in 2 weeks

b. External cephalic version

c. Deliver by elective cesarean section

d…

Answer: Schedule a repeat visit in 2 weeks

50. All of the following are causes of congestive dysmenorrhea except

a. PID

Page 41 of 124
b. Asherman’s syndrome

c. Pelvic adhesions

d. Pelvic congestion

e. Genital prolapse

Answer: Genital prolapse

51. All of the following are associated with Type I diabetes mellitus in pregnancy except

a. Trisomy 21

b. Polyhydramnios

c. Large baby

d. Congenital anomalies

Answer: Trisomy 21

52. A 30 year old patient presents with infertility, dyspareunia and dysmenorrhea for a
number of years. What is the next best investigation for this patient?

a. Hysterosalpingogram

b. Laparoscopy

c. Hysteroscopy

d..

Answer: Laparoscopy (case of endometriosis)

53. Which of the following is mismatched

a. Subocciptobregmatic – 12.5 cm

b. Occipito-mental – brow presentation

c. Occipito-frontal --- 11.5 cm

d. Submentobregmatic – Face presentation

Page 42 of 124
Answer: Suboccipitobregmatic – 12.5 cm (should be 9.5 cm)

54. All of the following are true about antiphospholipid syndrome


except

a. Thrombocytopenia can develop in this disease

b. Arterial and venous thrombi can occur

c. It is a congenital thrombophilia

d..

Answer: it is a congenital thrombophilia

55. All of the following are true about PCOS except

a. It predisposes to weight gain

b. FSH levels increase

c. Patient can have anovulatory amenorrhea

d. It is associated with Type II diabetes mellitus

e. It is possibly associated with a risk of atherosclerosis

Answer:

56. All of the following are true about HGSIL except

a. Cone biopsy is indicated if the cervix if colposcopy is inadequate

b. Cone biopsy is indicated can be therapeutic

c. Endometrial sampling is always needed

d…

Answer: Endometrial sampling is always needed

57. Which of the following statements is true about CIN (cervical intraepithelial neoplasia)?

a. Persistent CIN I can be treated with LEEP

Page 43 of 124
b. Patients who underwent cone biopsy don’t require followup

c. CIN is a cytological diagnosis

d. CIN III is defined as dysplasia involving the whole epithelium with invasion into stroma

e…

Answer: Persistent CIN I can be treated with LEEP

58. Which of the following parameters doesn’t decrease in a normal pregnancy

a. Platelet count

b. Packed cell volume

c. Gastric emptying

d…

Answer:

*Probably platelet count

59. All of the following can be preventable with good antenatal care except

a. Iatrogenic preterm birth

b. Gestational thrombocytopenia

c…

Answer: Gestational thrombocytopenia

60. Which of the following statements is incorrect about monozygotic twin pregnancies?

a. They can be of the same or opposite sex

b. The most common type is monochorionic diamniotic

c..*

*Statement about the thickness of the membrane between them

Answer: They can be of the same or opposite sex

Page 44 of 124
61. Which of the following is true about endometrial cancer

a. Lymph node invasion is related to depth of myometrial invasion and


grade of the tumor

b. Type II endometrial cancer is not estrogen related (or dependent*) but HPV related

c..

Answer: Lymph node invasion is related to depth of myometrial invasion and grade of the
tumor

62. A young patient complains of DUB. All of the following can be used in her management
except

Answer: Copper IUCD

63. Which of the following statements is correct

a. Tetanus vaccine is safe in the third trimester

b. Rubella vaccine is safe in the first trimester

c…

Answer: Tetanus vaccine is safe in the third trimester

64. All of the following are true about candidal vulvovaginitis except

a 80% of the cases are caused by candida albicans

b. Presence of a copper IUCD is a risk factor

c. It is the most common cause of vaginal discharge in a pregnant woman

d….

Answer:

*Probably: Presence of a copper IUCD is a risk factor

65. All of the following are true about Trichomonas vaginalis vaginitis except

a. Yellow frothy discharge is often observed

Page 45 of 124
b. Treated with metronidazole

c. Sexually transmitted

d. Wet mount shows flagellated motile micro-organisms

e….

Answer: Answer was e.

66. All of the following patients must be given anti-D in an Rh-negative female with an Rh-
positive husband except

a. Threatened abortion at 6 weeks

b. Ectopic pregnancy

c. Amniocentesis

d. External cephalic version

e..

Answer: Threatened abortion at 6 weeks

67. Which of the following is false about the CTG in a term baby

a. Normal variability is >5 bpm

b. Baseline heart rate is 100-160 bpm

c. It can be done more frequently in higher risk patients

d..

Answer: Baseline heart rate is 100-160 bpm

68. All of the following are risk factors for uterine fibroids except

a. Multiparity

b. Black race

c. Smoking

Page 46 of 124
d. Family history

e..

Answer: Smoking

69. While conducting a pelvic examination for a 38 week female in labor, you feel a nose
and mouth. The patient’s chin is pointed towards the mother’s pubic ramus. Which of the
following is true about the presentation/position of the child?

a. Face presentation- mentoanterior

b. Persistent occipitoposterior

c….

Answer:

70. A patient in the first trimester is found to have a positive urine culture for 100,000
colony forming units. Which of the following is the best line of management?

a. Admission and IV antibiotics

b. Oral antibiotics for 5-7 days

c. Single dose of oral antibiotics

d…

Answer: Oral antibiotics for 5-7 days (although c. is also scientifically correct)

71. All of the following are associated with endometriosis except**

a. Early pregnancy loss

b. Ectopic pregnancy

c. Late menopause

d…

Answer: Late menopause

Page 47 of 124
72. A 21 year old female patient presents with a solid unilateral
ovarian mass on ultrasound and has a significantly elevated alpha
feto-protein level. The most likely type of her ovarian mass is a(n)

a. Immature teratoma

b. Choriocarcinoma

c. Endodermal sinus tumor

d..

Answer: Endodermal sinus tumor

73. Which of the following is the best test to screen for gestational diabetes at 24-28 weeks
of gestational age

a. Fasting blood glucose

b. HbA1c

c. OGTT

d. 1 hour test after 50 g glucose challenge

e..

Answer: 1 hour test after 50 g glucose challenge

74. Which of the following statements about dating in pregnancy is incorrect

Answer: The date is calculated from the last day of the last menstrual period

75. A primigravida complains of excessive vaginal bleeding after delivering her child.
Delivery was difficult and instrumental delivery was used. Uterus is firm and is 2 cm below
the umbilicus. The most likely cause of her vaginal bleeding is

a. Uterine inversion

b. Uterine rupture

c. Retained products of conception

d. Uterine atony

Page 48 of 124
e. Genital tract laceration

Answer: Genital tract laceration

76. Which of the following statements about uterine inversion is incorrect

Answer: Chronic inversion occurs after 24 hours of delivery

77. All of the following statements are true about induction of labor except

a. Post-date pregnancy is the most common indication for induction

b. It increases the risks of cesarean section

c. Indicated in 35% of pregnancies

d…

Answer: Indicated in 35% of pregnancies (slides state 10-20%)

78. All of the following can lead to fetal tachycardia except

a. Pethidine

b. Hydralazine

c. Maternal fever

d…

Answer: Pethidine

79. A patient with placenta previa presents with mild vaginal bleeding at 32 weeks of
gestational age that stopped completely. Fetal CTG is reactive. What is your plan regarding
her delivery

a. Plan for an elective cesarean section at 37 weeks

b. Only deliver by cesarean if bleeding recurs

c..

Answer: Plan for an elective cesarean section at 37 weeks

80. All of the following are true about Chlamydia trachomatis except
Page 49 of 124
a. It is a gram negative intracellular bacteria

b. It causes severe symptoms

c. It can lead to ectopic pregnancy and infertility

d…

Answer: It causes severe symptoms

81. All of the following are risk factors for pre-eclampsia except

Answer: Smoking (a confusing risk factor mentioned was a young age (something like 19))

82. All of the following are true about uterine fibroids except

a. Red degeneration most commonly in pregnancy

b. Mostly present with menorrhagia

c. Submucosal fibroid can be treated through hysteroscopic removal

d…

Answer:

83. A young patient at 33 weeks presents with abdominal pain and moderate vaginal
bleeding. Which of the following is the most likely cause of her problem?

Answer: Placental abruption

84. A primigravid patient at 36 weeks of gestation presents with abdominal pain and
vaginal bleeding. Multiple late decelerations are seen on CTG. The patient is fully dilated,
fetus is ROA, +1 station and membranes are ruptured. What is your next step of
management?

a. Deliver by immediate Cesarean section

b. Instrumental delivery

c. Prostaglandins

d. Expectant management

Page 50 of 124
e..

Answer:

*Probably instrumental delivery

85. All of the following statements are true about Adenomyosis except

a. It is diagnosed by laparoscopy

b. Hysteroscopy can show uterine adhesions

c. It is a recognized cause of recurrent miscarriages

d..

Answer:

86. Which of the following statements is true about coagulation disorders in pregnancy

a. DVT is more common in pregnancy than in purperium

b. Warfarin is contraindicated while lactating

c. Patients with DVT are treated with heparin and then warfarin

d. 50% of DVTs are asymptomatic

e…

Answer:

87. All of the following are side effects of progesterone only contraceptives except

a. Functional cysts

b. Acne

c. Weight gain

d…

Answer: was d or e

88. About pelvic surgeries, which of the following is mistmatched

Page 51 of 124
a. Posterior colporrhapy – Rectocele repair

b. Cone biopsy – vaginal procedure

c….

Answer: Not A or B…

89. Which of the following statements is true about traction forceps?

a. They are used to deliver the after-coming head

b. They have a sliding lock

c. They have a lower risk of facial injury than vacuum

Answer:

90. All of the following are true about abortions except

a. In inevitable abortions uterus is usually small for date

b. In incomplete abortions the cervix is open

c. The best method of management for missed abortion is medical

Answer: In inevitable abortions uterus is usually small for date

91. Which of the following statements is incorrect about cervical cancer

Answer: in Stage IA, survival is 60%

Page 52 of 124
5th Year Final Exam 2015:
1. Investigations for a 30 years old female P4+0, complaining of menorrhagia, include all of the
following except:
a. Hysteroscopy and dilatation and curettage
b. Thyroid function tests
c. Cervical smear
d. Complete blood count
e. Vaginal ultrasound

Answer:
2. Regarding endometriosis all the following are correct except
a. Progressive disease
b. Is best diagnosed by pelvic ultrasound
c. Improves during pregnancy
d. Is benign
e. Usually no abnormal findings on physical examination

Answer:
3. Congestive (secondary) dysmenorrhea can be caused by all of the following except
a. Pelvic adhesions
b. Cervical stenosis
c. Pelvic inflammatory disease
d. Adenomyosis
e. Asherman’s syndrome

Answer:
4. Uterine corpus is mainly composed of:
a. Smooth muscle
b. Endometrium
c. Fibrous tissue
d. Elastic tissue
e. Estrogen receptors

Answer:
5. Menarche usually occurs between:
a. 14 and 16 years
b. 6 and 8 years
c. 8 and 10 years
d. 17 and 18 years
e. 11 and 13 years

Page 53 of 124
Answer: 11 and 13 years

Page 54 of 124
6. Regarding management of labour, all the following are correct except:
a. Assessment of vital signs is essential
b. Complete blood count should be ordered
c. Fetal assessment by CTG is required
d. Oxytocin should be used in all patients
e. Obstetric ultrasound is usually required

Answer:
7. All the following can be transmitted sexually except
a. Gonorrhea
b. Trichomonas vaginalis
c. HIV
d. Vaginal candidiasis
e. Chlamydia

Answer: Vaginal candidiasis


8. Umbilical cord entanglement mostly occurs in
a. Monoamniotic twins
b. Breech presentation
c. Dichorionic/diamniotic twins
d. Conjoined twins
e. Monochorionic/diamniotic twins

Answer:
9. Hormonal replacement therapy increases the risk of all of the following except:
a. Stroke
b. Alzhemier’s disease
c. Thromboembolic disease
d. Breast cancer
e. Endometrial cancer

Answer:
10. Regarding placenta previa, all the following are correct except:
a. Previous cesarean section is a risk factor
b. Malpresentation is common
c. The placenta is within 2 cm from the cervical os
d. A placenta covering the cervical os at 20 weeks is a major placenta previa
e. Is a risk factor for preterm delivery.

Answer: d. A placenta covering the cervical os at 20 weeks is a major placenta previa


11. The most common etiologic agents in postpartum mastitis is:
a. Neisseria
b. E. Coli

Page 55 of 124
c. Aerobic Streptococcus
d. Anaerobic Streptococcus
e. Staphylococcus aureus

Answer: e. Staphylococcus aureus


12. Ovarian neoplasms most commonly arise from
a. Metastatic disease
b. Ovarian germ cells
c. Ovarian epithelium
d. Ovarian sex cords
e. Ovarian stroma

Answer:
13. The prognosis of endometrial carcinoma is worse with all the following except
a. Myometrial invasion
b. When the tumor is poorly differentiated
c. In elderly
d. When there is cervical involvement
e. Endemetroid type

Answer: e. Endemetroid type


14. Regarding active management of 3rd stage of labor and PPH, only one of the following is correct
a. Syntocinon is contraindicated in cardiac patients
b. Passage of 600 ml of blood after delivery is considered as PPH
c. A placenta that looks complete rules out retained placental pieces
d. First step includes 5-10 units of syntocinon after delivery of the fetus
e. IV syntocinon induces uterine contractions within 5-10 min

Answer:
15. Only one of the following tumors produce estrogen
a. Thecoma
b. Endodermal sinus tumor
c. Choriocarcinoma
d. Dysgerminoma
e. Serous cyst adenoma

Answer: Thecoma
16. All the following are shown in partogram in labour except
a. Number of uterine contractions in 30 minutes
b. Cervical dilatation
c. Liquor status
d. Maternal pulse
e. Urine dipstick for protein

Page 56 of 124
Answer:
17. All the following are delivered vaginally except
a. Vertex/non-vertex twins
b. Fetus with an estimated weight of 4.0 kg
c. Vertex/ Vertex twins
d. Growth restricted fetus
e. Non-Vertex/vertex-twins

Answer: e. Non-vertex/vertex-twins
18. The following procedure can assess both uterine cavity and tubal lumen is:
a. Ultrasound
b. Hysteroscopy
c. Colposcopy
d. Laparoscopy
e. Hystero-salpingogram (HSG)

Answer: e. Hystero-salpingogram (HSG)


19. The minimal time required before repeating the semen analysis for spermatogenesis response is:
a. 30 days
b. 60 days
c. 15 days
d. 90 days
e. 120 days

Answer:
20. Regarding causes of fetal growth restriction, only one of the following is correct
a. Race
b. Hot climate
c. Dieting in pregnancy
d. Gestational diabetes
e. Anemia

Answer: e. Anemia
21. The following are most likely causes for abdominal pain at 20 weeks gestation except
a. Ectopic pregnancy
b. Spontaneous miscarriage
c. Complicated ovarian cyst
d. Acute appendicitis
e. Urinary tract infection

Answer: a. Ectopic pregnancy*


22. Regarding candidal vaginitis, all the following are correct except
a. Is treated with doxycycline
b. Is associated with thick white discharge

Page 57 of 124
c. May be associated with dyspareunia
d. Frequently associated with intense vulval pruritus
e. Whiff test is negative

Answer: a. Is treated with doxycycline


23. Complications of postdate include all of the following except
a. Respiratory distress syndrome
b. Oligohydramnios
c. Shoulder dystocia
d. Fetal death
e. Meconium aspiration syndrome

Answer: a. Respiratory distress syndrome


24. Normal duration of menstrual cycle is
a. 7-10 days
b. 1-3 days
c. 1-2 days
d. 1-4 days
e. 2-7 days

Answer: e. 2-7 days (The question should be written as normal duration of menses and not menstrual
cycle)
25. The median age of menopause is
a. 47
b. 51
c. 45
d. 41
e. 57

Answer: b. 51
26. Regarding ovarian tumors, all of the following are germ cell tumors except
a. Yolk sac tumor
b. Teratoma
c. Dysgerminoma
d. Choriocarcinoma
e. Endometroid tumor

Answer: Endometroid tumor


27. All of the following can be prevented or minimized by good antenatal care except
a. Constitutional fetal macrosomia
b. Fetal death
c. Fetal anemia
d. Iatrogenic post-maturity
e. Breech in labor

Page 58 of 124
Answer:
28. All of the following are recognized factors for uterine fibroid except
a. White race
b. Black race
c. Nulliparity
d. Polycystic ovarian syndrome
e. Family history

Answer: White race


29. Regarding abortion, all the following are correct except
a. Asherman’s syndrome is a recognized cause of recurrent abortion
b. In inevitable abortion, the cervix is closed
c. In incomplete abortion, the cervix is open
d. Ovulation can occur 2 weeks after abortion
e. In missed abortion, the best method of treatment is medical termination

Answer: b. In inevitable abortion, the cervix is closed


30. In placental abruption, all of the following are correct except
a. The patient commonly present in labor
b. May be associated with proteinuria
c. Fetus is often in transverse presentation
d. The main fetal risk is hypoxia
e. DIC is one of its serious complications

Answer: Fetus is often in transverse presentation


31. In a primigravida presented with lower abdominal pain, one of the following confirms the
diagnosis of labour
a. Dilated and fully effaced cervix
b. 2 uterine contractions in 10 minutes
c. Passage of small amount of blood per vagina
d. The head is at zero station
e. Leakage of liquor

Answer: a. Dilated and fully effaced cervix


32. The normal lining of the fallopian tube is
a. Columnar epithelium with cilia
b. Squamous epithelium
c. Fibrous connective tissue
d. Transitional epithelium
e. Cuboidal epithelium

Answer: a. Columnar epithelium with cilia

Page 59 of 124
33. One of the following ovarian tumors is most likely to result in virilisation
of a 35 years old woman
a. Sertoli-Leydig cell tumor
b. Thecoma
c. Brenner tumor
d. Mucinous cystadenoma
e. Dysgerminoma

Answer: Sertoli-Leydig cell tumor


34. Regarding hydatiform mole, all the following are correct except
a. Occurs once in 1200-1500 pregnancies in the west
b. Has only maternal chromosome in complete mole
c. May present with a small for date uterus
d. May be complicated with pulmonary embolus after evacuation
e. Complete mole is best diagnosed by ultrasound

Answer: b. Has only maternal chromosome in complete mole


35. Vulvo-vaginal candidiasis occurs more frequently in the following except:
a. Thyrotoxicosis
b. Pregnant
c. Diabetes
d. On oral contraceptive pills
e. On Long term antibiotic therapy

Answer:
36. A 28 year old patient complains of amenorrhea after having dilatation and curettage, the most
likely diagnosis is
a. Anorexia nervosa
b. Kallman’s syndrome
c. Pelvic inflammatory disease
d. Asherman’s syndrome
e. Turner’s syndrome

Answer: d. Asherman’s syndrome


37. Regarding control of bleeding after delivery, all the following are correct except
a. Platelets <50000/ml is a risk factor for bleeding
b. Lower genital tract laceration may cause severe bleeding due to edema and engorgement
c. The lower uterine segment is effectively contractile and retractile
d. Deposition of fibrin over placental bed helps stop bleeding
e. Contraction and retraction of the muscles in the upper segment is essential to close blood vessels.

Answer: The lower uterine segment is effectively contractile and retractile

Page 60 of 124
38. The following ligaments that provide the most support to the uterus are
the

a. Broad ligaments
b. Round ligaments
c. Arcuate ligaments
d. Uteroovarian ligaments
e. Cardinal ligaments
Answer:

a. Perinatal mortality is usually lower


b. Oral hypoglycemic drugs should be considered if glycemic control is difficult
c. Intrauterine fetal death incidence is highest at the beginning of third trimester
d. There is increased risk of neonatal hyperglycemia and hypercalcemia
e. Folic acid of 5 mg should be started pre-pregnancy
Answer:
40. Regarding cervical ectropion, all the following are correct except
a. It’s a cause of post coital bleeding
b. Is pathological in post-menopausal women
c. Is associated with increased vaginal discharge
d. Is more common during pregnancy
e. Is increased in pill-users

Answer:
41. Risk factors for endometrial carcinoma include all of the following except
a. Late menopause
b. Obesity
c. Combined oral contraceptive pills
d. Nulliparity
e. Family history

Answer: c. Combined oral contraceptive pills


42. Regarding labor all of the following are correct except
a. The length of the first stage of labor in primigravida is longer than that in multigravida
b. The latent phase is part of the second stage
c. Second stage of labor indicates delivery of the baby
d. The normal duration of second stage may reach 3 hours in primigravida
e. Oxytoic drugs are commonly used in third stage of labor

Answer: The latent phase is part of the second stage


43. Inverstion of the uterus is almost always subsequent to
a. Multiple pregnancy
b. Polyhydramnios

Page 61 of 124
c. Difficult forceps delivery
d. Traction of the umbilical cord before placental separation
e. Abruptio placenta

Answer: d. Traction of the umbilical cord before placental separation


44. Implantation in the uterus occurs at which stage of development
a. Eight cell embryo
b. Blastocyst
c. Embryonic disk
d. Zygote
e. Morula formation

Answer: Blastocyst
45. All of the following are risk factors for uterine atony except
a. High parity
b. Instrumental delivery
c. Chorioamnionitis
d. Prolonged labor
e. Primigravida with small for date uterus

Answer: e. Primigravida with small for date uterus


46. Regarding cardiotocograph (CTG) at term all the following are normal findings except
a. Baseline heart rate of 145 bpm
b. 8 uterine contractions in 10 minutes
c. One shallow early deceleration in the second stage of labor
d. Two accelerations in 30 minutes
e. Beat to beat variability of 15 bpm

Answer: 8 uterine contractions in 10 minutes


47. One of the following describes genuine stress incontinence
a. Incontinence that occurs during sleep
b. Incontinence due to involuntary bladder detrusor contractions
c. Involuntary urine loss preceded by a strong desire to void
d. Incontinence that occurs after coughing or sneezing
e. All involuntary urine loss

Answer: d. Incontinence that occurs after coughing or sneezing


48. Regarding false labor, all the following are correct except
a. The uterine tightness are irregular
b. Can be suppressed with analgesia
c. Can start as early as 26 weeks’ gestation
d. Are associated with cervical changes
e. The intra-uterine pressure doesn’t exceed 20

Page 62 of 124
Answer: d. Are associated with cervical changes
49. Contraindications for induction of labor include all of the following except
a. Post-date pregnancy
b. Cervical carcinoma
c. Abnormal antenatal CTG
d. Previous classical caeasarean section
e. Triplets

Answer: Post-date pregnancy


50. All the following can be diagnosed by ultrasound examination at 10 weeks gestation except
a. Chorionicity
b. Twins
c. Corpus luteal cyst
d. Missed miscarriage
e. Placenta previa

Answer: e. Placenta previa


51. All of the following are risk factor for placental abruption except
a. Pre-eclampsia
b. Smoking
c. Oligohydramnios
d. Direct abdominal trauma
e. Chorioamnionitis

Answer: Oligohydramnios*
52. Regarding partogram in labor, all the following are correct except
a. It is essential to avoid unnecessary caesarean delivery
b. It can demonstrate arrest of labor
c. It is done in second stage of labor
d. It is the best way to assess progress of labor
e. It can demonstrate uterine hyperstimulation

Answer: c. It is done in second stage of labor


53. Regarding rhesus disease, only one of the following is correct
a. Is a cause of first trimester miscarriage
b. Is a cause of hydrops fetalis
c. Occurs when the Rh status for both partners is negative
d. Is associated with higher risk of fetal anomalies
e. Anti-D immunoglobulin is IgM

Answer: b. Is a cause of hydrops fetalis


54. The best modality to diagnose endometriosis is by
a. Laparoscopy

Page 63 of 124
b. Pelvic ultrasound
c. B-hCG
d. ….

Answer: a. Laparoscopy
55. All the following modalities can be used to treat DUB (Dysfunctional uterine bleeding) except
a. Endometrial ablation
b. Combined oral contraceptives
c. Progesterone only pills
d. Tranexam

e. Mefanemic acid
Answer:
56. About growth restriction, which one of the following statements is correct

Answer: most SGA babies have normal growth pattern


57. Which of the following is true about post-partum contraception
a. Lactation only is an effective method of contraception
b. Oral contraceptives should be started
c. Should be started 4-6 weeks after delivery
d. IUCD is contraindicated after vaginal delivery
e. ….

Answer:
58. All the following are routine tests in an antenatal care visit except
a. OGTT
b. Antibody screen
c. CBC
d. Rubella IgG
e. ….

Answer:
59. All of the following occur in the early stages of menopause except
a. LH is elevated
b. FSH is elevated
c. There is an increase in the negative feedback on the pituitary
d. Estradiol is reduced
e. ...

Answer:
60. In a 17 weeks pregnant female with hypertension, all of the following can be causes for her
hypertension except
a. Pre-eclampsia
b. Thrombophilia

Page 64 of 124
c. Cushing’s syndrome
d. Molar pregnancy
e. Systemic Lupus Erythematosus

Answer:
61. All of the following are true about the fetal skull except
a. The posterior fontanel is diamond in shape
b. During labor severe molding can be an indicator of difficult labor
c. …..

Answer: The posterior fontanel is diamond in shape


62. The earliest time fetal movement can be detected by the mother is
a. 10 weeks
b. 16 weeks
c. 24 weeks
d. 6 weeks

Answer: 16 weeks
63. The most common presentation of endometriosis is
a. Dysparaunia
b. Infertility
c. Menorrhagia
d. Chronic pelvic pain
e. …

Answer:
64. Which of the following is correct about ectopic pregnancy
a. Accounts for 10% of maternal mortality rate
b. Pelvic mass is a common finding
c. Bathroom sign is found in most cases
d. More common in the left side than the right side
e. Most cases need salpingectomy

Answer:
65. If a patient’s last menstrual period was 25/12/2015, her expected date of delivery (EDD) will be
a. 1/10/2016
b. 5/10/2016
c. 1/9/2016
d. 5/9/2016
e. …

Answer: 1/10/2016
66. The most important determinant for cancer in endometrial hyperplasia is
a. Cellular atypia
Page 65 of 124
b. Grade
c. ..

Answer:
67. Which one of the following is true about bartholin’s abscess
a. It is usually bilateral
b. It presents as a painful swelling to one side of the clitoris
c. It is usually asymptomatic
d. It is treated by marsipulization Answer: b. It presents as a painful swelling to one side of the clitoris*
68. Which one of the following is true about complete molar pregnancy
a. Risk of malignant transformation is 15-20%
b. Theca lutein cysts are uncommon
c. …

Answer: Risk of malignancy is 15-20%*


69. The hormone that causes the thickening of the endometrium in the first stage of the menstrual cycle is
a. LH
b. FSH
c. Estrogen
d. Progesterone
e. Prolactin

Answer: c. Estrogen
70. The last feature to develop in puberty in females is
a. Breast budding
b. Pubic hair development
c. Menarche
d. ..

Answer: Menarche
71. The number of chromosomes in a normal fetus is
a. 45 XX
b. 45 XY
c. 46
d. 23

Answer: c.46
72. Which of the following vitamins should not be given in pregnancy
a. Vitamin A
b. Vitamin B12
c. Vitamin B3
d. Vitamin C
e. ..

Page 66 of 124
Answer: Vitamin A
73. Which of the following is true about gestational diabetes mellitus

Answer: It increases the future risk of developing diabetes mellitus


74. All of the following are risk factors for genital prolapse except
a. White race
b. Exercise

c. Menopause
d. ..
Answer: Exercise
75. A woman who delivered a few days ago, is now tearful, crying and complains of poor sleep. The most
likely diagnosis is
a. Blues
b. Depression
c. Psychosis
d. …

Answer: Blues
76. Which of the following is most strongly related to cervical cancer

Answer: HPV infection


77. Which of the following is true about the infant of a diabetic mother
a. There is a risk of development of neonatal hypoglycemia and hypercalcemia
b. Large for gestational age
c. …

Answer: b. Large for gestational age


78. The most common tumor in females of reproductive age is

Answer: Leiomyoma
79. All of the following are true about threatened abortion except
a. Fetal heartbeat is absent
b. Most cases make it to term
c. Most common cause of abortion
d. ..

Answer: a. Fetal heartbeat is absent


80. All of the following are true about MgSO4 except
a. Can lead to heart conduction problems
b. Can be used safely in patients with heart diseases
c. Is given in higher doses to treat eclampsia

Page 67 of 124
d. …

Answer: Can be used safely in patients with heart disease*


81. All of the following statements are correct about dating pregnancies except

Answer: It is dated from the last day of the last menstrual period
82. Which of the following is true about bacterial vaginosis

Answer: It causes fishy smell and watery discharge


83. Local anesthesia is better than general anesthesia in

Answer: it has a lower risk of deep vein thrombosis


84. All of the following are matched except

Answer: Suboccipitobregmatic – Face presentation


85. Which of the following best describes urge incontinence
a. Involuntary urine leaking after strong desire to void
b. Incontinence that occurs during sleep
c. Incontinence that occurs after coughing or sneezing
d. All involuntary urine loss

Answer: a. involuntary urine leaking after strong desire to void


86. Which of the following isn’t a cause of first trimester abortions

Answer: Cervical incompetence

Page 68 of 124
6th Year Final Exam 2015:
The exam was 100 questions, 2 forms. Some questions were the same in both
forms.
1) All can be detected on 1st trimester ultrasound except:
a. +microcephaly b. gestational age
c. ensure intrauterine pregnancy
d. chromosomal anomalies e. number of fetuses

2) Not a cause of superficial dyspareunia:


a. lactation
b. +pelvic adhesions
c. vulvar and vaginal candidiasis d. psychological
e. vaginal atrophy

3) Not a cause of post-coital bleeding:


a. cervical polyp
b. cervical carcinoma c. +endometrial polyp d. cervical ectropion
e. cervicitis

4) Not a cause of primary amenorrhea:


a. turner syndrome
b. mullerian agenesis
c. +asherman's syndrome
d. androgen insensitivity
e. prolactin secreting adenoma

5) Not a recognized indication for IVF:


a. +septate uterus
b. obstructive azospermia c. severe oligospermia
d. unexplained infertility e. tubal blockage

6) All are true about clomiphene citrate except: 4 | P a g e

Page 69 of 124
a. causes hyper-stimulation syndrome b. causes hot flushes
c. +indicated in premature ovarian failure d. increases levels of
gonadotropins
e. increases possibility of dizygotic pregnancy

7) All are true about primary dysmenorrhea except:


a. +in both ovulatory & non ovulatory cycles
b. starts at the 1st day of menses, subsides by the 3rd c. associated with vomiting & diarrhea
d. caused by prostaglandins increase in the 2nd half of the cycle

8) Not a tocolytic:
a. +beta blocker
b. nifedipine c. MgSO4
d. atosiban
e. indomethacin

9) About ovarian cancer, one is false: a. epithelial is the most common type b. +tubal ligation
increases its risk
c. endometrioid type may be associated with endometrial carcinoma d. endometriosis is
associated with clear cell type
e. multiparity reduces its risk
10) Inhibin is a tumor marker in:
a. +granulosa cell tumor b. dysgerminoma

11) About drugs in pregnancy, all are true except:


a. ampicillin is relatively safe
b. cephalosporin is relatively safe
c. +tetracycline is safe in 2nd trimester
d. phenytoin is associated with facial clefting

12) Earliest sign of chorioamnionitis:


a. +maternal tachycardia b. fever
c. abdominal tenderness d. vaginal bleeding

13) Not an indication for induction of labor:


a. preeclampsia at 39 weeks

Page 70 of 124
b. placental abruption at 37 weeks c. fetal growth restriction
d. +36 weeks gestation who has well controlled DM

14) Which of the following specifically suggests obstructed labor:


a. +severe moulding b. fetal distress
c. station -2
d. arrest of dilation
e. occipito-posterior position

15) True about shoulder dystocia:


a. +many affected babies are of normal body weight
b. the best management is to keep pushing & traction c. obstruction occurs at the pelvic
outlet
d. 70% is the risk of recurrence
e. preventable in majority of cases

16) 35 years old P3 has endometriosis and doesn't wish to preserve fertility, best
management:
a. continuous OCPs for 6-9 months
b. hysterectomy and bilateral salpingo-oophorectomy c. danazol
d. GnRH
e. progesterone secreting IUD

17) All are risk factors for premature ovarian failure except:
a. mumps
b. radiation
c. autoimmune disease
d. +progesterone mini pills e. smoking

18) All of the following are true regarding antenatal care except:
a. decreases the risk of iatrogenic postmaturity
b. decreases complication on mother and fetus in mother with medical illness
c. prevent iron deficiency anemia d. +prevents fetal anomalies

19) About folate deficiency anemia, which is wrong:


a. +using anti-convulsant therapy reduces the need for folate b. MCV increases
c. hyper-segmented neutrophils on blood film d. second most common anemia 6 | P a g e

Page 71 of 124
e. folate supplements lower the incidence of neural tube defects

20) About congenital anomalies, all are true except:


a. overall incidence of major congenital anomalies is 3%
b. +most common cause is chromosomal abnormalities c. common presentation is 1st
trimester miscarriage
d. may not be evident at birth
e. might be caused by environmental factors

21) While giving MgSO4, all of the following should be monitored except:
a. +pupillary reflex
b. deep tendon reflexes
c. respiratory rate d. blood pressure e. urine output

22) About hyper-emesis gravidarum, all are true except:


a. +polyurea is a sign of severity
b. can be associated with hyperthyroidism
c. can cause Wernicke's encephalopathy in severe cases
d. more with multiple pregnancy e. usually resolves by 12 weeks

23) About GTN, all are true except:


a. myelosuppresion is an uncommon side effect of methotrexate
b. most common site of metastasis is the lungs c. +partial mole never makes it to term
d. best treatment is suction and curettage
e. most common karyotype of complete mole is 46XX

24) Which of the following reduces the efficacy of COCPs:


a. +rifampicin b. ranitidine
c. metochlopramide d. steroids
e. hydrochlorothiazide

25) A female gave birth by C-section 2 weeks ago, placenta was delivered completely, now
she came febrile with secondary PPH. All are true except: a. can continue breastfeeding
b. most common cause is retained products of conception c. best treatment is with
ceftriaxone and metronidazole
d. HVS and endocervical swab are recommended
e. uterus palpable 4 cm above the pubic symphysis indicates sub-involution

Page 72 of 124
now she is having bleeding. The most likely cause would be: a.
+uterine atony
b. uterine rupture

27) The most important risk factor for preterm labor:


a. +previous preterm
b. cervical incompetence
c. premature rupture of membrane

28) About normal vaginal delivery after CS (VBAC), all are true except:
a. incidence of rupture is 0.5-1%
b. 70% will deliver vaginally
c. oxytocin use should be avoided
d. +instrumental delivery is contraindicated
e. success rate increases if labor begins spontaneously

29) Smoking can cause all of the following except:


a. IUGR
b. placental abruption c. +preeclampsia

30) True about cord prolapse:


a. can be diagnosed by palpation while membranes are intact b. +can be delivered vaginally if
cervix was fully dilated
c. common cause of intrapartum fetal death
d. using vaginal PG in induction increases the risk e. incidence 1 in 5,000

31) All fulfill the criteria of normal vaginal delivery except:


a. +gestational age 36+4
b. bleeding in 3rd stage 300 ml of blood
c. infusion of oxytocin after the delivery of the anterior shoulder d. CTG used in monitoring in
1st & 2nd stage of labor

32) All are normally done in labor except:


a. +routine artificial rupture of membrane when cervix is 4 cm dilated
b. asses uterine contractions every 30 min c. vaginal exam every 2-4 hours

33) All are causes for increased AFP except: 8 | P a g e

Page 73 of 124
a. hydatiform mole b. anencephaly
c. +Down syndrome d. fetal demise
e. multiple pregnancy

34) All are true about thyroid disease in pregnancy except:


a. +graves' disease worsens in the 3rd trimester
b. hyperthyroidism is associated with stillbirth c. propylthiouracil is safe in lactation
d. thyroxine is safe in pregnancy
e. radio-iodine scan is contraindicated in pregnancy

35) All are true about endometrial cancer except:


a. obesity is a risk factor
b. +serous type is associated with good prognosis
c. baseline treatment for all patients is hysterectomy and bilateral salpingo-
oophorectomy
d. post-menopause women have worse prognosis than pre-menopause

36) About candidiasis, all are true except:


a. +recurrence is not common
b. fluconazole is not safe during pregnancy
c. vulva will be edematous and inflamed d. whiff test is negative
e. occurs more during pregnancy

37) About post menopausal bleeding, all are true except:


a. endometrial thickness >5 mm on ultrasound is an indication for biopsy b. vaginal atrophy is
a recognized cause
c. +endometrial cancer is the most common cause d. cervical carcinoma should be
considered

38) About cervical cancer, all are true except:


a. +cervical smear is diagnostic
b. with stage Ia, 5 year survival is 95%
c. squamous cell carcinoma is the most common

39) In relation to pregnancy, all of the following may increase during puerperium except:
a. thromboembolism
b. +diabetes

Page 74 of 124
c. thyroiditis
d. torsion of ovarian cyst e. psychosis

40) All are true about recurrent abortions except:


a. incidence is 1%
b. +defined as 3 consecutive induced abortions c. can be due to protein C deficiency
d. can be due to anti-phospholipid syndrome

41) All are routinely done in antenatal care except:


a. +HbA1c
b. platelets count
c. screening for asymptomatic bacteriuria
d. pelvic ultrasound e. urinalysis

42) All are true about antenatal care except:


a. +swimming is contraindicated in the 3rd trimester b. exercise results in faster labor
c. travel before 36 weeks is allowed for all low risk pregnancies

43) In high grade intraepithelial lesions, all are true except:


a. +endocervical curreting and endometrial biopsy are indicated
b. cone biopsy is indicated if the squamocolumnar junction couldn't be visualized
c. mostly caused by persistent infection with HPV 16, 18 d. can be asymptomatic

44) About DVT, all are true except:


a. more on the left side
b. +heparin causes cardiac defect
c. in a patient with history of DVT, treatment should continue 6 weeks postpartum

45) All are true about urodynamic studies except:


a. +bladder capacity is 200-300 ml
b. first desire to void occurs at 150-250 ml c. flow rate during voiding is > 15 ml/sec
d. it's important for accurate diagnosis of urge incontinence e. residual volume should be less
than 50 ml

Page 75 of 124
46) About stress incontinence, all are true except:
a. can be associated with urge incontinence
b. +can be treated effectively by anti-cholinergic medications c. they
have urine leak upon coughing or sneezing
d. vaginal childbirth is a risk factor

47) All are components of the biophysical profile except:


a. fetal tone
b. fetal movement
c. amniotic fluid index d. +fetal weight
e. non stress test

48) All are true about breastfeeding except:


a. breastfeeding is not recommended in HIV
b. +breastfeeding is not recommended in HBV +ve mothers
c. lactation is inhibited by bromocriptine
d. lactation is initiated postpartum by falling level of estrogen

49) About bacterial vaginosis, one is wrong:


a. +sexually transmitted
b. clue cells on wet mount are diagnostic c. usually asymptomatic
d. often presents with foul smelly discharge e. pH is more than 4.5

50) About chlamydia trachomatis, one is wrong:


a. gram negative intracellular organism
b. may be a cause for neonatal pneumonia
c. +most common cause of vaginal discharge d. diagnosed by endocervical swab
e. treated by erythromycin during pregnancy

51) About epidural analgesia, one is wrong:


a. +prolongs 1st stage of labor b. prolongs 2nd stage of labor
c. CTG changes are mainly due to maternal hypotension d. increases risk of instrumental
delivery
e. doesn't increase the risk of CS

52) All can be diagnosed by ultrasound in the 3rd trimester, except:


a. renal agenesis b. +Turner

Page 76 of 124
c. facial cleft
d. oligohydramnios

53) All are causes for large-for-date uterus, except:


a. +bilateral renal agenesis
b. anencephaly
c. wrong date
d. esophageal atresia e. full bladder

54) Not a cause of secondary amenorrhea:


a. +ovarian dysgenesis b. anorexia nervosa

55) 30 years female, para 2, presented with menorrhagia and found to have intramural
fibroid, best management:
a. mirena
b. +myomectomy c. GnRH analogue d. COCPs
e. hysterectomy

56) Progesterone-secreting IUD is used in all of the following except:


a. intramural fibroid b. endometriosis
c. DUB

57) All of the following are risk factors for osteoporosis except:
a. +late menopause
b. low BMI
c. smoking
d. GnRH use

58) About bacterial vaginosis, one is wrong:


a. pH is elevated
b. +profuse foul smelling thick vaginal discharge c. may be a risk for postpartum endometritis
d. clue cells on wet mount are diagnostic e. no pruritus

59) Forceps delivery, presented after that with fresh vaginal bleeding, uterus is contracted,
the most likely cause is:
+genital tract trauma
b. uterine rupture c. uterine atony
60) All of the following increase the risk of uterine rupture except:
a. myomectomy
b. instrumental delivery
Page 77 of 124
c. +amniocentesis d. C section

61) All are components of partogram except:


a. maternal pulse
b. fetal presentation c. cervical position
d. contractions

62) About CIN:


a. it is a cytological description
b. hysterectomy is a standard treatment of CIN III
c. CIN I can regress in 60% of cases

63) About epidural analgesia, one is true:


a. +CSF leakage is a cause for headache
b. hypertension treated with labetalol is a recognized contraindication
c. contraindicated in a patient with history of DVT

64) Preeclampsia increases the risk of all the following except:


a. IUGR
b. +polyhydramnios

65) In preeclampsia, all of the following are indications for delivery except:
a. BP 160/115
b. +platelet 100,000 c. HELLP syndrome
d. neurological symptoms e. proteins >5g/24 hr

66) All are indications to give anti-D except:


a. amniocentesis
b. +complete spontaneous abortion at 7 weeks gestation
c. non-sensitized Rh-ve women whose partner is Rh+ve d. abdominal trauma
e. external cephalic version

67) All are true about Rh-isoimmunization except:


a. intrauterine blood transfusion is the treatment of choice of fetal hydrops occurs
b. +anti-D should be given to sensitized women
c. middle cerebral artery doppler can be used to assess the severity of anemia

68) 7 weeks amenorrhea, presented with left lower quadrant pain and mild vaginal
bleeding. Her vital signs were stable. On ultrasound, the uterus was empty. Her beta hCG
was 3000 mIU/ml. Your management would be:

Page 78 of 124
a. +single dose IV methotrexate
b. repeat beta hCG after 48 hours
c. repeat ultrasound after one week d. immediate laparotomy

69) History of amenorrhea for few weeks, presented with abdominal pain and severe
bleeding with passage of tissues. Now the bleeding has stopped and on ultrasound the
uterus is empty. Your management would be:
a. +this is a complete abortion, observe for bleeding
b. repeat beta hCG after 2 days
c. this is a threatened abortion, reassure her d. antibiotics & evacuation

70) All are considered normal in semen analysis, except:


a. morphology 50% normal
b. +motility 20% forward motile c. liquefaction within 20 minutes d. volume 4 ml
e. density 20 million/ml

71) 37 weeks gestation presented with mild placental abruption, cervix is 5 cm dilated, fetal
CTG reactive. After stabilization of the patient, what's your next step:
a. +AROM and give oxytocin
b. emergent CS
c. give prostaglandin
d. admission and regular CTG every two days e. steroids

72) G3P2, previous deliveries were by C section. Now she's 36 weeks gestation presenting
with painless vaginal bleeding. After resuscitation, what is your next step:
a. +ultrasound to localize the placenta
b. give steroids
c. vaginal exam
d. give steroids & indomethacin
e. immediate C section

73) All are significant findings in 40 weeks gestation except:


a. epigastric pain
b. vaginal spotting
c. +lower limb edema d. blurred vision
e. persistent vomiting

74) All are maternal complications of PET except:


a. subcapsular hematoma b. renal failure
c. +fetal growth restriction d. DIC

Page 79 of 124
e. abruptio placenta

75) All of the following increase the risk of endometrial cancer except:
a. +progesterone therapy b. obesity
c. tamoxifen
d. granulosa theca cell tumor

76) About endometrial cancer, only one is correct:


a. progesterone therapy is used for a 59 years old patient with atypical
hyperplasia
b. +tamoxifen has a recognized association with endometrial hyperplasia

77) Treatment of hypertension in pregnancy is to avoid:


a. IUGR
b. IUFD
c. oligohydramnios
d. placental abruption e. +maternal stroke

78) About diabetes in pregnancy, which one is wrong:


a. well controlled DM can develop macrosomia b. fetal death may occur in well controlled
DM
c. congenital anomalies increase in gestational DM

79) About diabetes in pregnancy, one is correct:


a. +diet control alone increases the risk of developing hypertension
b. normal blood sugar at 20 weeks rules out the development of DM
c. insulin requirement increases in puerperium

80) About delivery in multiple pregnancy, one is wrong:


a. +cephalic presentation of 1st twin and transverse of 2nd is an indication for
C section
b. episiotomy is not routinely done
c. oxytocin augmentation is not contraindicated
d. continuous CTG is recommended for both twins

81) About multiple pregnancy, one is wrong:


a. all dizygotic are diamniotic
b. conjoint twins occur if separation of the zygote happens after 12 days

Page 80 of 124
c. twin-twin transfusion is a recognized complication in monochorionic
twins d. +best time to diagnose chorionicity is 16 weeks gestation

82) About COCPs, one is correct:


a. should be taken with periodic pill breaks
b. +doesn't affect future fertility
c. cause acne

83) About ectopic pregnancy, one is true:


a. can present before missed menses
b. positive pregnancy test and empty uterus on ultrasound are diagnostic
c. fallopian isthmus is the most common site d. most cases are older than 35 years
e. usually asymptomatic

84) About cholestasis, one is false:


a. +may lead to amniotic fluid embolism b. it's a diagnosis of exclusion
c. leads to itching in mainly palms and soles d. associated with fetal distress
e. associated with preterm labor

85) About endometriosis, one is false:


a. has a genetic predisposition
b. +endometrioma can't increase more than 5 cm in diameter c. ovarian injury during surgery
can attribute to infertility
d. transformation to malignancy is rare
e. 1/3 of cases are diagnosed through infertility investigations

86) One of the following is not a vaginal procedure:


a. anterior colporrhaphy b. +ovarian cystectomy
c. resection of Bartholin cyst d. vaginal hysterectomy

87) Hormonal replacement therapy can treat all of the following except:
a. +coronary artery disease b. osteoporosis
c. vaginal dryness d. hot flushes
e. mood swings

88) About genital prolapse, one is wrong:


a. 2nd degree uterine prolapse is descent of the cervix in the introitus b. vaginal
hysterectomy is a suitable management
c. +uretrocele is descent of the vagina through the upper 1/3 of posterior wall

Page 81 of 124
89) Uterine fibroid can cause all of the following in pregnancy except:
a. IUGR
b. +fetal anomalies
c. recurrent miscarriage
d. abnormal presentation e. delivery by C section

90) All are recognized causes of organic menorrhagia except:


a. endometriosis
b. granulosa cell tumors
c. fibroids
d. +Sheehan syndrome

91) About abdominal pain in pregnancy, one is wrong:


a. ovarian torsion requires immediate surgical intervention
b. appendicitis is the most common cause of surgical abdominal pain c. +peptic ulcer disease
deteriorates in pregnancy
d. gallstones may cause pancreatitis

92) All are causes for decreased variability on CTG, except:


a. opioids
b. prematurity
c. sleeping
d. +epidural analgesia
e. hypoxia

Page 82 of 124
93) Patient with primary PPH, apart from resuscitation, what is your
next step in managing her bleeding:
a. +uterine massage & oxytocin
b. bimanual compression of the uterus
c. B lynch procedure
d. uterine artery embolization e. uterine artery ligation

94) Twin pregnancy may increase all of the following complications, except:
a. DM
b. preeclampsia c. +post maturity
d. malpresentation

95) Of the following, only one is correct:


a. umbilical artery doppler is of no value after 34 weeks gestation b. meconium in the trachea
is a very rare finding
c. +oligohydramnios is found in all IUGR babies
d. small for gestation age babies are all growth restricted e. IUGR is defined as weight below
15th percentile

96) About iron deficiency anemia in pregnancy, one is wrong:


a. ferretin is low
b. +baseline management is IV iron c. multiparity increases the risk

97) 24 years old female, primi at 37 weeks gestation, presented with regular uterine
contractions every 3 minutes. Her vital signs were normal, and her CTG was reactive. What
is your management:
a. +allow her to deliver spontaneously
b. ARM & oxytocin
c. vaginal prostaglandin
d. tocolytic e. steroids

98) All of the following are true about labor, except:


a. 2nd stage of labor may take 3 hours in a primigravida with epidural analgesia
b. +prolonged 3rd stage is delivery of the placenta after 20 minutes c. android pelvis
increases the risk of malposition
d. occipito-posterior position is normal in early labor
e. epidural analgesia can be given in active phase of 1st stage of labor

99) About puerperium, one is wrong:


a. commonly develops polyurea
Page 83 of 124
b. +warfarin is a contraindication for breastfeeding
c. after 2 weeks, uterus should be at the level of symphysis pubis d.
ofloxacin is given in case of mastitis

100) One of the following is wrong:


a. +suboccipito-bregmatic is face presentation
b. brow presentation must be delivered by C section

101) Compared to partial mole, all are features of complete mole except:
a. +focal trophoblastic proliferation
b. usually presents with large-for-date uterus c. more

102) A lactating women, 6 weeks pregnant, has mild abdominal pain and vaginal bleeding.
On ultrasound, there was intrauterine sac. What to do? a. +reassure and ask her to come
after 2 weeks
b. give her methotrexate c. induce abortion

103) One of the following is true about diabetes in pregnancy:


a. most diabetic patients present as hypertension
b. +babies of controlled diabetic mothers have nearly same complications as non-diabetic
mothers

104) Term pregnant lady, cervical dilatation 6 cm, uterine contractions 2 in


10 minutes lasting 20 seconds. After two hours, nothing changed. What would you do?
a. +give her oxytocin
b. immediate C section c. instrumental delivery

105) True about ectopic:


a. +the most common site is the fallopian ampulla

106) False about GTN:


a. +patient can take OCPs before hCG is normal

107) All are risk factors for endometrial cancer except:


a. lynch II
b. +turner
c. tamoxifen d. estrogen

108) About epidural analgesia, one is wrong:


a. +can be used in a hypotensive patient

Page 84 of 124
109) One of the following is wrong regarding recurrent abortion:
a. paternal age is a risk factor
b. screening for bacterial vaginosis is useful
c. +it is advisable to do screening for toxoplasmosis

110) One of the following ovarian tumors could be bilateral in 10-15% of cases:
a. +dysgerminoma

Page 85 of 124
Obstetrics & gynecology Final Exam
2017- PULSE

1_Which of the following wrong :

Answer: android pelvis is common female pelvis

2_which of the following is incorrect :

a-continuous ctg with maternal scar


b-early cord clamping is a part of active management
c-android pelvis is associated with inc risk of malformations

A:b

3_we use all of the following in menorrhagia except:

a-Danazol
b-norethisterone
c-Gnrh analogue
d-Tamoxifen
e-Mefanamic acid

A: d

4_we do all of the following in ANC except:


a-Rubella igg
b-Toxoplasma igg
c-Urine analysis
d-CBC

A: b

5_Which one of the following true about tt secondary postpartum hemorrhage(something like that ) :-
1) Ergometrine
2) steroid is the first tt to start with it
3) It's the bleeding that starts 24hr after delivery to 6 weeks postpartum
4) it doesn't occur after CS
51T

Page 86 of 124
A: 4

6_COCP all true except:


Answer : decrease risk for cervical cancer

7_whats false about bregma :


a-Located in posterior fontanelle

b-Drug of choice in syphilis treatment


c-Erythromycin
d- Gentamycin

A: a

8_Not an indication of CS in multiple pregnancy :


a- The first is cephalic and second not
b- Monochorionic twins
c- Triplet

A: a ( not sure )

9_Day one fever after CS :


a- Atelectasis
b- Pneumonia
c- Wound infection

A: a ( not sure )

10_Wrong about PPH :


Answer : vaginal bleeding more than 800ml *

11_The EDD for pt whose last menstrual period on 25-12-2016 :


1-10-2017
1-9-2017

A: 1

12_all can be seen on Hystrosalpingogram except:


a- Endometrial polyp
b- Uterine adhedions
c- Subserosal fibroid
d-patency of fallopian tube

Page 87 of 124
Answer :c

13 _all are risk of endometrial carcimona except:


a- Obesity
b- Lynch syndrome
c- Endometrial polyp
d-

Answer : ?

14_one is epithelial ovarian tumor:


a- Granulose cell tumor
b- Clear cell tumor
c- Teratoma
d- Endodermal sinus tumor
e- fibroma

Answer: b

15_All of the following are risk factors for uterine fibroids except :
a. Nulliparity
b. White race
c. PCOS
d. Family history
e. Obesity

answer : b

16_ case about ovarian cancer , AFP is elevated, HCG and LDH is normal … most likely diagnosis :
a- Choriocarcinoma
b- Dysgerminoma
c- Endodermal sinus tumor

Answer : c

Explanation: **Sertoli ledig tumor secrete androgens **Granulosa stromal cell tumor secrete estrogen
+ inhibin is the tumor marker ** LDH tumor marker for dysgerminoma. ** hcg tumor marker for
choriocarcinoma ** AFP marker for endodermal sinus tumor

Page 88 of 124
17_ all absolute contraindication of IUCD except :

a- Active PID
b- unexplained vaginal bleeding
c- History of ectopic pregnancy
d- trophoblastic malignant tumor
e- Uterine anomaly.
A : c ( not sure )

18_all are IVF commplications except :


a- OHSS
b- Ectopic pregnancy
c- Multiple pregnancy
d- Gestational DM
E- ovarian cyst

answer : d

19_treatment of syphilis in pregnancy :


a- Trimethoprim
b- Gentamycin
c- Erythromycin
d- Penicillin
E- tetracycline

Answer : d

20_ dysfunctional uterine bleeding (DUB), which of the following is false :


a- It can be ovulatory
b- It is common in perimenarchal age
c-it could be due to endometrial polyp

A:c

21_which one mismatch :


a- Face presentation…. Suboccipito- bregmatic
b- Brow presentation … mento occipital

Answer A

Page 89 of 124
22_most accurate method to determine gestational age :
a- certain Last menstrual period
b- Femur length at 25 weeks
c- transvaginal US at 8 weeks

A : c ( not sure )

23_ all causes asymmetrical IUFR except :


1- chromosomal abnormalities
2- SLE
3- thrombophilia
4- renal failure

Answer : 1 (chromosomal abormalties causes symmetrical.)

24_ all of the following causes fetal tachycardia except :


a- Maternal fever
b- Pethidine
c- Anemia

Answer : b

25_ all can be caused by Sheehan’s syndrome except:


a-hypoglycemia
b-amenorrhea
c-lactation
d-fatigue

answer : c

26 _ stress incontinence one is false :


Answer : we should use anti-cholinergics

27_ urge incontinence one is true :


Answer : incontinence follow strong desire to void

Page 90 of 124
28_pregnant lady has delivered 35 week child, previous d27 week, one ectopic
pregnancy :
1-G 3 P2+1
2-G4 p2+1
3-G4 p2

Answer : 2

29_Which one is not an indication to give anti D :


a- Antepartum hemorrhage at 33 wks
b- Complete abortion at 19 wks
c- External cephalic version at 37 wks
d- Threatened abortion> 7 week

A:d

30_Question about not part of partogram :


I think The answer is previous hx of neonatal admission

31 _Most common type of geastational anemia :


The answer is : IDA

32 _Whats abnormal regarding CTG :


Thr answer is : 4 late decelartions

33 _Which one is not a cause for uterine atony :


a -prolonged labor
b-multible pregnancy
c-antepartum hemmorhage
d- previous Cs

e-restricted growth

A:e

34 _progesterone-only contraception is a present in all of the following except :


a-oral
b-implant
c-transdermal patch
d-IUCD
e-IM injection

A:c

Page 91 of 124
35_about DVT which is correct :
- Duplex US can't detect all cases
-its a clinical diagnosis
-risk in pregnancy more than puerperium

A:?

36 _ All of the following are used for pain relief in labor


except :
a- Spinal
b- Epidural
c- Narcotics
d- Nitrous oxide
e-Diazepam

A:e

37 _All the following occur in GDM except :


a-hypocalcemia
b- cardiac anomalies
c-caudal regression syndrome
d- respiratory distress syndrome
e-hypoplesia of islets of pancreas

A:e

38 _ All of the follwing is true except: (nase el so2al)


Answer : Brain Ct is mandatory when LH &FSH increase.

39 _ hypocalcemia :
- cardiac anomalies
-caudal regression syndrome
- respiratory distress syndrome
-hypoplesia of islets of pancreas

A:?

40 _ about placenta previa which is correct :


- speculum examination is contraindicated
-

Page 92 of 124
Answer : ?

41_ Drug of choice in Syphilis treatment during pregnancy is :


a) Penicillin
b) Erythromycine
c)Tetracyclin
d)Gentamycin

A:a

42 _in postmenopausal woman all investigations can be done except :


a- transvaginal ultrasound
b- estrogen
c- cervical inspection
d- Pap smear
e- valvular examination

A:b

43_ in partogram all of the following are included except :


a- cervical dilation
b- patient profile
c- past medical or surgical history
d- head staging
e - uterine contractions

A:c

44 _The following ligaments that provide the most support to the uterus are the :
a. Broad ligaments
b. Round ligaments
c. Arcuate ligaments
e. Cardinal ligaments

A:e

45 _ The most common tumor in females of reproductive age is:


a. Sarcoma
b. Leiomyoma
c. Adenocarcinoma
d. Adenomyosis

A:b

Page 93 of 124
46 _Regarding the accurate estimation of gestational age, all of the following are true except:
a-The ultrasonic estimation at 32 weeks of gestation is usually inaccurate
b- In the presence of lactation, ultrasonic estimation is recommended
c- It is best done by measuring the crown rump length in the second trimester

A:a

47 _Regarding cervical ectropion, all of the following are true except :


a. It is commonly seen in post-menopausal women.
b. Cervical smear is mandatory before treatment.
c. Post coital bleeding is a common presentation.
d. Can be caused by combines contraceptive pills.

A:a

48 _Implantation in the uterus occurs at which stage of development :


a. Eight cell embryo
b. Blastocyst

A:b

49 _The normal lining of the fallopian tube is :


a. Columnar epithelium with cilia
b. Squamous epithelium
c. Fibrous connective tissue
d. Transitional epithelium
e. Cuboidal epithelium

A:a

50 _ All of the following can be prevented or minimized by good antenatal care except :
a. Constitutional fetal macrosomia
b. Fetal death

Page 94 of 124
c. Fetal anemia
d. Iatrogenic post-maturity
e. Breech in labor

A:a

51 _ Regarding stress incontinence, all are true except :


a. Involuntary leakage of urine occurs with sneezing, coughing and laughing
b. Is the most common type of incontinence
c. Is best treated with anticholinergic medications
d. During cytometer, there are no uninhibited detrusor contractions during filling

A:c

52 _ Hydatidiform mole is characterized by all of the following except :


a. Hypothyroidism
b. Uterus larger than
c. Severe Hyperemesis

A:a

53 _ All of the following can be sexually transmitted, except:


a. Trichomonas vaginalis
b. Chlamydia
c. Vaginal candidiasis
d. HIV e. Gonorrhea

A:c

54 _ The infectious agent that is involved in the development of cervical cancer is :


a. Herpes b virus .
B.Human papilloma virus
c . Syphilis

Page 95 of 124
d. Gonorrhea

A:b

55 _ all are contraindicated in pregnancy except:


a-warfarin
b-LMWH
c-Ciprofloxacillin
d-phenytoin

A:b

56 _ which of the following is correct regarding Ante-partum Hemorrhage :


a) vasa previa is rupture of fetal blood vessels
b) grade 2 placenta previa is the placenta partially covering the cervical os

c- sterile speculum examination is contraindicated


d- placenta previa can be diagnosed as early as 22 weeks
e- placenta abrubtio best delivered by CS

,, i think it's A

57 _ regarding Sheehan syndrome which is false :


a-hypoglycemia
b-lactation
c-Amenorrhea

A:b

58 _ Which of the following cause fetal tachycardia :-


a-Maternal fever
b-Maternal hypothyroidism
c-Fetal Distress
d-Pithidine

A:d

Page 96 of 124
59 _ biophysical profile contain all these except :
a-fetal movement
b-fetal breathing movement
c-Doppler

A:c

60 _ At time of EDD the gestational age is :


a-38w
b-39w
c-40w
d-41w
e-42w

A:c

61 _ All are essential tests in placental abruption except:


a-bood film
b- cbc
c-rh-isoimmunisatiin

d-kft
e-pt

A:a

62 _ Most common ovarian neoplasm in pregnancy is:


a- corpus Luteal cyst
b-granulosa cell tumor
c-endometriod
d-clear cell

A:a

63 _ which one is the epithelial type of ovarian cancer :

answer is: clear cell carcinoma

64 _ Epidural anesthesia when compared with regional anesthesia one of the following is false :-

Page 97 of 124
b- causes less post surgery nausea
c- causes less bleeding
d- rapid recovery after surgery
e-Increase Risk of DVT

A:e

65 _ the definite diagnosis of endometriosis is made by:


a-hysteroscopy
b-laparoscopy
c-abdominal ultrasound

A:b

66 _ Relationship of fetal parts to each other is called:


a-Position
b-Station
c-Lie
d-Presentation
e- Altitude

A:e

67 _ All of the following can be used in controlling PPH except:


a-PG E2
b-PG F2a
c-Misoprostol

d-Oxytocin
e- Ergometrin

68 _ Correct about placenta abrubtion:


Diagnosed with US, and other choices can't remember them

69 _ False about lactation:


Answer : Caused by drop in estrogen postnatally

70 _ All Cause IUGR except:


a-Chronic renal faliure
b-SLE

Page 98 of 124
c-thrombopholia
d-Hypertension

A:?

71 _ All cause secondary amenorrhea except:


Answer : Turner

72 _ Correct about placenta previa except:


Answer : Usually affect fetal well being

73 _ Mismatch:
Answer : Brow-mentovertical

74 _ Risk factor for PET except:


Answer : Low social economics ( not sure )

75 _ Endmoetrial CA:
Answer : Most commonly adenocarcinoma

76 _ True about Venous thromboembolism:


Answer : Warafrin execrered in breast milk

77 _ Regional Vs general anesthesia, one is false regarding regional:


Answer : Increases risk of DVT

78 _ Most common malpresentation:


Answer : Complete breech???

79 _ Correct about Gestational thrombocytopenia :


a-Less than 20,000
b-Less than 20000
c-Fetus affected

A:?

80 _ False about Preterm labor :


a-Erythromycin indicated in PROM
b-NSAID not recommended after 32
c-Weekly steroids between 28-34

Page 99 of 124
A : c ( this is according to my answer  so not sure )

81 _ Correct about APH :


Answer :Placenta abrubtion diagnosed with US (mybe there was choice like
that)

82 _ IVF commplications except :


a-OHSS
b-Ectopic preg
c- Mutible preg

A:?

83 _ False about labor :


a-True labor s supressed by epidural anesthesia
b-No cervical changes in false

A : a ( not sure )

84 _ False about Fetal skull :


a- Posterioir fontanele called bregma
b- Slide on each other

Answer : a

85 _ All cause ectopic preg except :


a-COCP
b-mini pills
c-IUCD

A:?

86 _ Uterus after delivery is :


a-Boggy
b-Firm

Answer : ?

87 _ C.I. IUCD Usage except:


a-Previous ectopic
b-GTN

Answer : a

Page 100 of 124


88 _ False about normal CTG in labor :
Answer : Late deceleration

89 _ In postmenupausal bleeding, one is not mandatory:


a-Speculum exam
b-Vulva inspection
c-US

A:?

90 _ False about labor :


a-True labor is suppressed by epidural anesthesia
b-No cervical changes in false
c-the first stage ends with full cervical dilation
d- the third stage is the delivery of the placenta and membranes
e- episiotomy is indicated in all primigravida

A:a(?)

91 _ All cause ectopic pregnancy except :


COCP is the answer

92 _ all of the following regarding secondary amenorrhea are true except :

a-turner mosaic

b-transverse vaginal septum

A : b ( not sure )

93 _ About EDD, the most accurate :


a-Transvaginal us at GA 8 week (?)
b-Femural length
c-Mother sensation of fetal movement
d-LMP

A:a

94 _ About urge incontinence :


a-Involuntary loss of urine preceded by ugre to void
b-Caused by uninhibited bladder detrusor contraction

Page 101 of 124


A:a

95 _ Which is wrong about cervical cancer :


Answer : Adenocarcinoma is the most common type

96 _ Pregnant lady at GA 20 wk, all can be a cause for large for GA ecexpt :
a-GDM
b-Multiple pregnancy
c-Wrong date
d-Fibroid
e-full bladder

A:a

97 _ Age of menarche :
a-14 16
b- 8 10
c-11 13
e-16 18

A:c

98 _ All can be transmitted by vaginal delivery except :


a-HIV
b-HSV2
c-toxoplasmosis
d-Chlymdia
e-Gonorrhea

A:?

99 _ All are causes of recurrent miscarriage except :


a-Chromosomal abnormalities
b-Rubella
c-Spetated uterus
d-Cervical incompetence

A:b

100 _ All are causes of abdominal pain and vaginal bleeding after 8 wk of amenorrhea, except :
a-Placental abruption
b-Inevitable
c-Incomplete miscarriage
d-Ectopic pregnancy

Page 102 of 124


A:a

101_ In labour which is wrong :


a-The fetus enters pelvic brim on AP diameter
b-Internal rotation for the head
c-THe head exists by extension
d-Ant sholuder is first

A:a

102 _ All fowllowing are risk for fibroid except :


a-Reterograde menses
b-African
c-Nullparity
d-Family history

A:a

103 _ All can be used in sever PET except :

a-Meythel dopa
b-Diazepam
c-Hydrazaline
d--
e-

A:?

104 _ Which is wrong about chole stasis ?


a-Dx by exclusion
b- Air embolism is complication
c- Itchy (Palm sole )

A:?

105_ All following cause asymmetrical IUGR except :


a-SLE
b-HTN
c-RF
d-chromosomal abnormalities

A:D

Page 103 of 124


� ‫ﺗﻢ بﺤﻤﺪ‬

‫ﺗﻤﻨ�ﺎي ﻟ�ﻢ بﺎﻟﺘﻮﻓﻴﻖ‬ ‫ﻣﻊ‬
‫ي‬

Obstetrics & gynecology Final Exam 2017- Ehsan

1) Regarding abdominal pain in pregnancy, which of the following is false:


a. Peptic ulcer usually deteriorates in pregnancy
b. Ovarian torsion requires immediate surgical intervention
c. Appendicitis is the most common cause
d. Gallstones may cause pancreatitis

Answer: Peptic ulcer usually deteriorates in pregnancy.

2) All of the following are risk factors for uterine atony except:
a. Augmentation of labor by oxytocin
b. Use of MgSO 4
c. Breech Presentation
d. Antepartum Hemorrhage (APH)
e. Instrumental Delivery

Answer: Antepartum Hemorrhage (APH)

3) 68. All of the following are risk factors for uterine fibroids except
a. Nulliparity
b. White race
c. PCOS
d. Family history
e. Obesity

Answer: White race

4) Regarding dysfunctional uterine bleeding (DUB), which of the following is false:

a. It is commonly managed with hormonal therapy

Page 104 of 124


b. It can be due to large submucosal fibroid
c. It is common in perimenarchal age
d. It can be ovulatory
e. It can be treated with hysterectomy

Answer: It can be due to large submucosal fibroid

5) A 32 weeks pregnant female has a Hb of 9. The best initial management is:


a. Give blood transfusion
b. Start intramuscular iron
c. Give IV iron
d. No action needed
e. Treat with oral iron and vitamins

Answer: Treat with oral iron and vitamins

6) Regarding infertility, which of the following is false:

a. Ovulation occurs 26 hours after LH peak


b. Asthenospermia is the medical term for abnormal morphology
c. Seminal fluid makes up to 90% of the ejaculation
d. Endometriosis may interfere with tubal motility
e. Cystic fibrosis may lead to congenital bilateral absence of vas deferens

Answer: Asthenospermia is the medical term for abnormal morphology

7) Regarding twin delivery, which is false:

a. episiotomy is not routinely performed


b. Continuous fetal heart rate monitoring is recommended
c. Epidural analgesia can be safely used
d. If the second fetus becomes tansverse in lie, an emergency C/S should be
performed
e. Augmentation with oxytocin if needed is not contraindicated

Answer: If the second fetus becomes tansverse in lie, an emergency C/S should be
performed

Page 105 of 124


8) Regarding postmenopausal bleeding, which is false:

a. Endometrial thickness of 0.5cm is an indication for biopsy


b. Endometrial CA is the most common cause
c. May be due to vaginal atrophy
d. Cervical CA is a recognized cause
e. The incidence is higher at age 80 compared to age 50

Answer: Endometrial CA is the most common cause

9) Regarding placental abruption, one is correct:

a. Nearly half of placental abruptions are in established labor


b. Placental abruption never presents after 40 weeks gestational age
c. Smoking decreases the risk of placental abruption
d. All patients with placental abruption should receive tocolytics
e. Continuous fetal heart rate monitoring is not mandatory in established cases

Answer: Nearly half of placental abruptions are in established labor.

10) Sperm capacitation refers to the process by which the spermatozoa become capable
of:

a. Fertilizing the ovum


b. Penetrating the cervical mucus
c. Stimulating meiosis of the ovum
d. Producing acrosomal enzymes
e. Dispersing the zona radiate

Answer: Producing acrosomal enzymes

11) G3P2 at 38 weeks GA presents with abdominal pain and bright red vaginal bleeding.
On examination the uterus was firm and tender. She has regular uterine contractions
every 2 minutes. The patient had normal U/S at 34 weeks. Which of the following is the
most likely cause of her problem:

a. Placental Abruption
b. Placental Accreta
c. Labor
d. Placenta Previa
e. Heavy show

Page 106 of 124


Answer: Placental Abruption

12) The shortest distance between the sacral promontory and the
lower edge of the symphysis pubis is:
a. Obstetric conjugate
b. Diagonal conjugate
c. True conjugate
d. Interspinous diameter
Answer: Diagonal conjugate

13) One of the following tests is not used to assess ovulation:


a. Estrogen concentration on day 2 of the cycle
b. Mid-luteal phase serum progesterone concentration
c. Basal Body Temperature charting
d. Urinary LH surge testing
e. Serial U/S scan

Answer: Estrogen concentration on day 2 of the cycle

14) One of the following is not a contraindication for tocolytics:


a. Placental abruption
b. Transverse lie
c. Chorioamnionitis
d. Labour with 7cm dilation
e. Fetal distress

Answer: Transverse lie

15) Regarding cervical incompetence, which is false:


a. Can be diagnosed by U/S
b. Can be caused by cone biopsy of the cervix
c. Is a cause of preterm labour
d. Is best treated with cervical suture at the end of the 1st trimester
e. Is a recognized cause of first trimester abortion

Answer: Is a recognized cause of first trimester abortion

16) Which of the following statements about primary amenorrhea is false:

Page 107 of 124


a. Primary amenorrhea is absence of menses and secondary sexual
characteristics at the age of 16
b. Gonadal dysgenesis is the most common cause of primary
amenorrhea
c. In imperforated hymen, blood collects in vagina
d. Imperforated hymen can be corrected by surgery
e. A karyotype is the next step in a patient with elevated FSH and normal breast
development
Answer: Primary amenorrhea is absence of menses and secondary sexual
characteristics at the age of 16

17) Regarding drugs in pregnancy, which of the following is true:


a. Rubella vaccine can be given in 3rd trimester
b. Tetracycline can cause hemolytic anemia of newborn
c. Tetanus vaccine is safe in 3rd trimester
d. Cephalosporins are classified as category A
e. The most common congenital anomaly associated with phenytoin use is cardiac defect.

Answer: Tetanus vaccine is safe in 3rd trimester

18) Regarding intrauterine infection, which of the following is false:


a. Intrauterine infection is an important cause of brain injury
b. May result from hematogenous transplacental spread secondary to maternal
bacteremia
c. May result in periventricular leukomalacia (PVL)
d. Does not occur in the presence of intact membrane
e. Chorioamnionitis is an indication for induction of labor

Answer: Does not occur in the presence of intact membrane

19) You feel a fetal nose and mouth. The chin is pointing toward the maternal left pubic
rami. This is a presentation of:
a. Face, mentoanterior
b. Occiput, transverse position
c. Brow presentation
d. Transverse lie
e. Vertex presentation

Page 108 of 124


Answer: Face, mentoanterior

20) The next step in the management of a patient with severe


dysmenorrhea and infertility, and normal pelvic U/S is:
a. Laparotomy
b. HSG
c. MRI
d. Laparoscopy
e. CT scan

Answer: Laparoscopy

21) Regarding clomiphene citrate, which is false:


a. can cause hirsutism
b. May cause hot flushes
c. It is given IV
d. Ideally should not be used more than 12 months
e.It can cause ovarian hyperstimulation

Answer: It is given IV

22) Regarding a pregnant patient with DM, one of the following is false:
a. Glucosuria is a reliable parameter for control
b. Glycosilated Hb is best kept below 6
c. DKA is more common in pregnant than non-pregnant
d. Insulin requirement increases as pregnancy advances
e. Polyhydramnios is a common complication

Answer: Glucosuria is a reliable parameter for control

23) A woman who suffers from anterior pituitary failure (Sheehan’s syndrome) can be
induced to ovulate by:
a. HMG injection
b. Pulsatile GnRH
c. Bromocriptine
d. Clomiphene citrate
e. Low dose estrogen

Page 109 of 124


Answer: HMG injection

24) The following ligaments that provide the most support to


the uterus are the
a. Broad ligaments
b. Round ligaments
c. Arcuate ligaments
d. Uteroovarian ligaments
e. Cardinal ligaments

Answer: Cardinal Ligaments

25) One of the following is not associated with pre-eclampsia:


a. Thrombocytopenia
b. Hemo concentration
c. Increased renal perfusion
d. Reduced intravascular volume
e. Increased risk of thrombosis

Answer: Increased renal perfusion

26) All of the following are true about a patient with androgen insensitivity syndrome
except:
a. The patient will have normal external male genitalia
b. Patient will have a karyotype with XY
c. It is an androgen receptor mutation
d. the patient has no uterus
e. The gonad should be removed after puberty

Answer: The patient will have normal external male genitalia

27) Which of the scenarios is a patient with severe pre-eclampsia


a. Patient with BP 150/100 and +1 proteinuria
b. Patient with BP 140/100 and +2 proteinuria
c. Patient with BP 140/100, +1 proteinuria and severe headache
d. Patient with BP 150/100 and 2g in 24 hour urine collection

Answer: Patient with BP 140/100, +1 proteinuria and severe headache

Page 110 of 124


28) Regarding Rh-isoimmunization, which is false:
a. intrauterine blood transfusion is the treatment of choice of fetal
hydrops occurs
b. anti-D should be given to sensitized women
c. middle cerebral artery doppler can be used to assess the severity of anemia
d. Can occur after maternal blood transfusion

Answer: anti-D should be given to sensitized women

29) Regarding the presenting of fetal diameters, one of the following is a false match:
a. Sub-occipitobregmatic diameter  Face presentation
b. Sub-mentobregmatic diameter  Hyper-extended fetal head
c. Bi-parietal diameter (BPD) = 9.5 cm
d. Occipitofrontal = 11.5cm
e. Mento-vertical diameter  Brow presentation

Answer: Sub-occipitobregmatic diameter  Face presentation

30) The most common tumor in females of reproductive age is:


a. Sarcoma
b. Leiomyoma
c. Adenocarcinoma
d. Adenomyosis
e. Choriocarcinoma

Answer: Leiomyoma

31) All of the following have a risk of recurrence except


a. Gestational diabetes
b. Pre-eclampsia
c. Placental abruption
d. PPROM
e. Cord prolapse

Answer: Cord prolapse

32) Regarding premature ovarian failure, one of the following is true:

Page 111 of 124


a. Occurs in 15% of women
b. Should be confirmed by ovarian biopsy
c. It is unlikely to be caused by infection
d. It is unlikely to occur before puberty
e. May be caused by previous surgery

Answer: May be caused by previous surgery

33) Regarding bacterial vaginosis, one of the following is true:


a. It is a rare vaginal infection
b. It is treated with anti-fungal agents
c. It is associated with fishy smell discharge
d. It is always symptomatic
e.It is usually associated with severe vulvar itching

Answer: It is associated with fishy smell discharge

34) Regarding mechanism of labour, which is false:


a. The rest of the fetal body delivers by flexion
b. The commonest fetal position is occipito-anterior
c. The presenting part enters the pelvic brim in flexion
d. The external rotation indicates rotation of fetal head to AP- diameter

Answer: The external rotation indicates rotation of fetal head to AP- diameter

35) One of the following is not a risk factor for cord prolapse:
a. Multiple pregnancy
b. Transverse lie
c. Footling breech
d. Short cord
e. Polyhydramnios

Answer: Short cord

36) Regarding thrombocytopenia during pregnancy, which is true:


a. HIV can cause thrombocytopenia during pregnancy
b. 20,000 platelet count is safe for regional anesthesia

Page 112 of 124


c. ITP presents only in first trimester
d. Fetal blood sampling is advised in all cases of thrombocytopenia
in pregnancy
e. Gestational thrombocytopenia carries a significant pathological effect on the fetus

Answer: HIV can cause thrombocytopenia during pregnancy

37) Regarding placenta previa, one is correct:


a. The diagnosis of placenta previa is based on clinical evaluation
b. Most low lying placentae diagnosed during detailed anomaly scan will move up to the
upper segment by reaching term
c. X-matching one unit of blood is enough
d. All patients with placenta previa should be delivered by C/S
e. Vaginal U/S is contraindicated in placenta previa

Answer: Most low lying placentae diagnosed during detailed anomaly scan will move
up to the upper segment by reaching term

38) Regarding epithelial ovarian cancer, one is correct:


a. COCP is a risk factor
b. Carries good prognosis if diagnosed early
c. Staging of the disease is based on radiological findings and Ca125 level
d. Familial in 50% of cases
e. Borderline type is best treated by chemotherapy

Answer: Carries good prognosis if diagnosed early

39) One of the following is not used in the management of acute PID:
a. Clindamycin
b. Laparoscopy
c. Oral doxycycline
d. D&C
e. Removal of IUCD

Answer: D&C

40) Regarding cervical ectropion, all the following are correct except
a. It’s a cause of post coital bleeding

Page 113 of 124


b. Is pathological in post-menopausal women
c. Is associated with increased vaginal discharge
d. Is more common during pregnancy
e. Is increased in pill-users

Answer: C or E !

41) One of the following does not cause recurrent miscarriage:


a. Bicornuate uterus
b. Cervical incompetence
c. Subserosal fibroid
d. Hormonal dysfunction
e. Chromosomal abnormalities

Answer: Chromosomal abnormalities

42) Investigations for a 30 years old female P4+0, complaining of menorrhagia, include
all of the following except:
a. Hysteroscopy and dilatation and curettage
b. Thyroid function tests
c. Cervical smear
d. Complete blood count
e. Vaginal ultrasound

Answer: Hysteroscopy and dilatation and curettage

43) Regarding ectopic pregnancy, one is true:


a. Mini-pill contraception is a risk factor
b. A positive urine pregnancy test is diagnostic
c. Vaginal bleeding is usually heavy
d. Amenorrhea is present in all patients
e. Most cases are ruptured ectopic

Answer: Mini-pill contraception is a risk factor

44) One of the following is not associated with polyhydramnios:


a. Twin-Twin transfusion syndrome
b. Diabetes

Page 114 of 124


c. Esophageal atresia
d. Vasa Previa
e. Anencephaly

Answer: Vasa Previa

45) What's the risk of complex atypical endometrial hyperplasia to progress into
endometrial Ca:
a. 1%
b. 5%
c. 15%
d. 25%

Answer: 25%

46) One of the following does not explain reduced variability in fetal heart tracing:
a. Barbiturates ingestion
b. Sleeping phase
c. Fetal stimulation
d. Asphyxia
e. Prematurity

Answer: Fetal Stimulation

47) Regarding multiple pregnancy, one of the following is true:


a. The rate of mono-zygotic twins is racial dependent
b. Chorionicity is best determined at 7 weeks GA
c. The rate of structural abnormalities is less than in singleton
d. Monozygotic twins are more common than dizygotic twins
e. In monozygotic twins, the risk of Down syndrome is the same for both fetuses

Answer: Chorionicity is best determined at 7 weeks GA

48) One of the following is not associated with breech presentation:


a. Multiple pregnancy
b. Prematurity
c. Placenta Previa
d. Congenital uterine anomalies

Page 115 of 124


e. Full maternal bladder

Answer: Full maternal bladder

49) The most common causative agent of postpartum mastitis is:


a. Neisseria
b. Aerobic streptococcus
c. Staph. Aureus
d. Chlamydia

Answer: Staph. Aureus

50) All of the following patients must be given anti-D in an Rh-negative female with an
Rh-positive husband except
a. Threatened abortion at 6 weeks
b. Ectopic pregnancy
c. Amniocentesis
d. External cephalic version

Answer: Threatened abortion at 6 weeks

51) Regarding molar pregnancy, one of the following is less likely to occur in complete
mole than in partial mole:
a. Focal rather than diffuse trophoblastic proliferation
b. Theca lutein cyst
c. Hyperthyroidism
d. Uterus Large for gestational age

Answer: Focal rather than diffuse trophoblastic proliferation

52) Regarding management of coagulation disorders in pregnancy, one of the following is


true:
a. Heparin should be stopped 7 days before performing C/S safely
b. Osteoporosis is more common in LMWH than in HMWH
c. Activated Protein C resistance is an example of inherited thrombophilia
d. Warfarin can be used safely during pregnancy
e. Heparin will cause thrombocytosis if used during pregnancy

Page 116 of 124


Answer: Activated Protein C resistance is an example of
inherited thrombophilia

53) Regarding continuous epidural anesthesia, one of the following is true:


a. It is contraindicated in woman with past history of DVT
b. Leakage of CSF is a recognized cause of headache
c. Hypertension treated with labetalol is a recognized contraindication
d. It is contraindicated in suspected IUGR

Answer: Leakage of CSF is a recognized cause of headache

54) Congestive (secondary) dysmenorrhea can be caused by all of the following


except
a. Regular use of COCP
b. Cervical stenosis
c. Pelvic inflammatory disease
d. Adenomyosis
e. Asherman’s syndrome

Answer: Regular use of COCP

55) Contraindications for induction of labor include all of the following except
a. Pre-eclampsia
b. Adnormal antenatal CTG
c. Transverse lie
d. Previous classical C/S section
e. Triplets

Answer: Pre-eclampsia

56) Regarding instrumental vaginal delivery, one of the following is true :


a. If the head is in -1 station, it is safe to use the forceps
b. Rupture of membrane is a must to assess the engagement
c. Forceps can be used to assist vaginal breech delivery
d. Majority of the cases are elective procedures

e. Perinatal morbidity is similar to normal delivery

Page 117 of 124


Answer: Forceps can be used to assist vaginal breech delivery

57) Mrs. A presented to the clinic complaining of painless vaginal bleeding after 8 weeks
of amenorrhea. She delivered vaginally 6 months ago and currently she is lactating. B-
HCG found to be (1200 IU/L). Transvaginal U/S showed an empty uterus. One of the
following is correct regarding her case:
a. Diagnostic laparoscopy should be the first step in her management
b. Repeating B-HCG after 7 days is advised
c. This picture might represent an early intrauterine pregnancy
d. Complete molar pregnancy is a strong possibility
e. Methotrexate injection is the ideal way of managing her

Answer: Both B and C seem to be correct!

58) Which of the following is the best reason for the treatment of high blood pressure
in pregnancy?
a. Prevent maternal complications such as stroke
b. Lower the incidence of pre-eclampsia
c. Lower the incidence of oligohydramnios
d. Lower the incidence of intrauterine demise
e. Incidence of IUGR

Answer: Prevent maternal complications such as stroke

59) All are true about antenatal care except:


a. swimming is contraindicated in the 3rd trimester
b. exercise results in faster labor
c. travel before 36 weeks is allowed for all low risk pregnancies

Answer: swimming is contraindicated in the 3rd trimester

60) About hyper-emesis gravidarum, all are true except:


a. can be associated with hyperthyroidism
b. can cause Wernicke's encephalopathy in severe cases
c. more with multiple pregnancy
d. usually resolves by 12 weeks
e. Associated with HTN

Page 118 of 124


Answer: Associated with HTN

61) Regarding cervical incompetence, all the following are true except
a. Can be caused by multiple pregnancy
b. Can be diagnosed by ultrasound
c. Is a cause of preterm labour
d. Can be caused by cone biopsy of the cervix
e. Best treated in the 2nd trimester of pregnancy

Answer: Can be caused by multiple pregnancy

62) All of the following are symptoms of menopause except


a. Hot flashes
b. Loss of weight
c. Insomnia
d. Anxiety
e. Genital prolapse

Answer: Loss of weight

63) Combined oral contraceptives are contraindicated in all of the following except
a. Migraine headache with focal headache
b. Severe liver cirrhosis
c. IHD
d. Diabetes mellitus

Answer: Diabetes mellitus

64) All of the following indicate severe pre-eclampsia except


a. Pulmonary edema
b. Oligohydramnios
c. Headache and neurological symptoms
d. Renal failure

Answer: Oligohydramnios

Page 119 of 124


65) A patient 35 weeks pregnant presents to the clinic for a
followup visit. On US her fetus is in a breech position. Which of the
following is the next best step of management?
a. Schedule a repeat visit in 2 weeks
b. External cephalic version
c. Deliver by elective cesarean section

Answer: Schedule a repeat visit in 2 weeks

66) All of the following are true about antiphospholipid syndrome except
a. Thrombocytopenia can develop in this disease
b. Arterial and venous thrombi can occur
c. It is a congenital thrombophilia

Answer: it is a congenital thrombophilia

67) A patient presents with a solid unilateral ovarian mass on ultrasound and has a
significantly elevated LDH level, negative AFP and HCG. The most likely type of her
ovarian mass is a(n)
a. Immature teratoma
b. Choriocarcinoma
c. Endodermal sinus tumor
d. Dysgermninoma

Answer: Dysgerminoma

68) A primigravida complains of excessive vaginal bleeding after delivering her child.
Delivery was difficult and instrumental delivery was used. Uterus is firm and is 2 cm
below the umbilicus. The most likely cause of her vaginal bleeding is
a. Uterine inversion
b. Uterine rupture
c. Retained products of conception
d. Uterine atony
e. Genital tract laceration

Answer: Genital tract laceration

69) All of the following are risk factors for pre-eclampsia except:

Page 120 of 124


a. extreme ages
b. smoking
c. nulliparity

Answer: Smoking

70) One of the following is not a tocolytic:


a. beta blocker
b. nifedipine
c. MgSO4
d. atosiban
e. indomethacin

Answer: beta blocker

71) A woman who delivered vaginally a few days ago, is now tearful, crying and complains
of poor sleep. The most likely diagnosis is:
a. Blues
b. Depression
c. Psychosis

Answer: Blues

72) The most important risk factor for preterm labor:


a. previous preterm
b. cervical incompetence
c. premature rupture of membrane

Answer: previous preterm

73) All fulfill the criteria of normal vaginal delivery except:


a. gestational age 36+4
b. bleeding in 3rd stage 300 ml of blood
c. infusion of oxytocin after the delivery of the anterior shoulder
d. CTG used in monitoring in 1st & 2nd stage of labor

Answer: gestational age 36+4

Page 121 of 124


74) About stress incontinence, all are true except:
a. can be associated with urge incontinence
b. can be treated effectively by anti-cholinergic medications
c. they have urine leak upon coughing or sneezing
d. vaginal childbirth is a risk factor

Answer: can be treated effectively by anti-cholinergic medications

75) About chlamydia trachomatis, one is wrong:


a. gram negative intracellular organism
b. may be a cause for neonatal pneumonia
c. most common cause of vaginal discharge
d. diagnosed by endocervical swab
e. treated by erythromycin during pregnancy

Answer: most common cause of vaginal discharge

76) All of the following are risk factors for genital prolapse except
a. White race
b. Exercise
c. Menopause
d. Hysterectomy

Answer: Exercise

77) All of the following are true regarding Preterm labor except:
A. Complicates 50% of pregnancies
B. Home uterine activity monitoring was proved to decrease the risk
C. Positive fetal fibronectin indicates high risk for preterm delivery
D. Previous preterm is a strong risk factor

Answer: Home uterine activity monitoring was proved to decrease the risk

78) All of the following are routine investigations for infertility except:
A. Progesterone at 21 days in a 28-day cycle
B. Semen analysis
C. Tests of tubal patency
D. Day 2 FSH, LH, testosterone, FBS

Page 122 of 124


E. Day 21 FSH and LH

Answer: Day 21 FSH and LH

79) Complications of post maturity include AII the following except


a. Prolonged labor
b. Oligohydramnios
c. Intrauterine fetal death
d. Respiratory distress syndrome
e. Meconium aspiration syndrome

Answer: Respiratory distress syndrome

80) Regarding follicle-stimulating hormone (FSH), only one of the following statements
is correct:
a. Is responsible for estradiol production from the granulosa cells
b. Brings about follicular rupture
c. Is raised in polycystic ovary syndrome
d. Is necessary for the initial stages of embryo development
e. Is necessary for maintenance of the corpus luteum

Answer: Is responsible for estradiol production from the granulosa cells

81) All are normal physiological changes in pregnancy except:


a. Increase HR
b. Increase GFR
c. Increased gastric time emptying
d. Increase WBC

Answer: They all seem correct!

82) The age of menopause is:


a. Genetically determined
b. Affected by OCP use
c. Smoking has no effect
d. Affected by the age of last pregnancy

Answer: Genetically determined

Page 123 of 124


83) One of the following is NOT a cause of delayed puberty:
a. Cold climate
b. Hypothyroidism
c. Prolactinoma

Answer: Cold climate

84) One of the following doesn't cause hirsutism:


a. Cyproterone citrate
( Past year question, no idea of other choices :p )

85) One of the following is NOT correct about candidal vulvovaginitis:


a. Intense itching
b. Common in pregnancy
c. pH > 6

Answer: pH > 6

86) Regarding fatty liver in pregnancy, which is wrong:


a) Rx is ICU and delivery
b) They have hypoglycemia
c) Parity is not an indicator

Answer: Both B and C seem to be correct

Page 124 of 124


Obstetrics & Gynecology Final Exams

(2010 – 2014)

Obstetrics and Gynecology 1


‫بسم هللا الرحمن الرحيم‬
‫تحية طيبة وبعد ‪...‬‬
‫تضع لجنتكم " لجنة الطب البشري " بين أيديكم هذه المجموعة من أسئلة‬
‫السنوات الخاصة بمادة النسائية والتوليد والتي دأب فريق عمل من الدفع‬
‫السابقة لتوفيرها لكم بأفضل ترتيب و أكمل تنسيق لتكون عوناً لكم في تفوق‬
‫طمحتم له طويل ً ‪ ،‬فوفقكم هللا جميعاً لما يجب ويرضى وحقق هللا أمانيكم‬
‫الرائعة كقلوبكم ‪.‬‬

‫‪Index‬‬
‫‪1. 2014 5th Year Final Exam …………………………………………………… 3‬‬
‫‪2. 2014 6th Year Final Exam …………………………………………………… 17‬‬
‫‪3. 2013 5th Year Final Exam …………………………………………………… 37‬‬
‫‪4. 2013 6th Year Final Exam …………………………………………………… 60‬‬
‫‪5. 2012 6th Year Final Exam …………………………………………………… 71‬‬
‫‪6. 2012 5th Year Final Exam …………………………………………………… 82‬‬
‫‪7. 2011 6th Year Final Exam …………………………………………………… 90‬‬
‫‪8. 2011 5th Year Final Exam …………………………………………………… 95‬‬
‫‪9. 2010 6th Year Final Exam …………………………………………………… 105‬‬
‫‪10. 2010 5th Year Final Exam ………………………………………………… 125‬‬

‫‪2‬‬
1. 5th Year Final Exam 2014:
1. Gonorrhea can infect all the following except:
- (+) Nose
- Mouth
- Cervix
- Vagina
- Urethra
** N gonorrhoeae causes:
1. Urogenital infections in women in the form of Cervicitis [most common], Urethritis, PID
[salpingitis, endometritis, and/or tubo-ovarian abscess, and perihepatitis known as Fitz-Hugh-
Curtis syndrome] and Bartholinitis. It is also a less common cause of Vaginitis.
2. Urogenital infections in men in the form of Urethritis and Epididymitis or epididymo-
orchitis.
3. Extragenital infections in the form of Proctitis, Pharyngitis, Disseminated Gonococcal
infection [endocarditis, meningitis or gonococcal dermatitis-arthritis syndrome: arthralgia,
tenosynovitis, and painless non-pruritic (non-itchy) dermatitis], and Conjunctivits [Neonatal,
and adult].

2. All of the following can cause Oligohydramnios except:


- Potter Syndrome
- (+) Anencephaly
- Pulmonary Hypoplasia
**Polyhydramnios causes:
- Twin gestation with twin-to-twin transfusion syndrome (increased amniotic fluid in the
recipient twin and decreased amniotic fluid in the donor) or multiple gestations
- Fetal anomalies, including esophageal atresia (usually associated with a tracheoesophageal
fistula), tracheal agenesis, duodenal atresia, and other intestinal atresias
- CNS abnormalities and neuromuscular diseases that cause swallowing dysfunction
(anencephaly)
- Congenital cardiac-rhythm anomalies associated with hydrops, fetal-to-maternal hemorrhage,
and parvovirus infection
- Poorly controlled maternal diabetes mellitus (Oligohydramnios may also be seen if severe
vascular disease is present.)
3
- Chromosomal abnormalities, most commonly trisomy 21, followed by trisomy 18 and trisomy
13.
- Fetal akinesia syndrome with absence of swallowing
**Oligohydramnios causes:
- Fetal urinary tract anomalies, such as renal agenesis (Potter syndrome), polycystic kidneys, or
any urinary obstructive lesion (eg, posterior urethral valves)
- PROM [most common cause] and chronic leakage of the amniotic fluid
- Placental insufficiency, as seen in pregnancy-induced hypertension (PIH), maternal diabetes,
or postmaturity syndrome when the pregnancy extends beyond 42 weeks' gestation
- Maternal use of prostaglandin synthase inhibitors or ACE inhibitors
**Oligohydramnios adversely affects fetal lung development, resulting in pulmonary
hypoplasia that typically leads to death from severe respiratory insufficiency.

3. One of the following is X-lined:


- Huntington's Disease [AD]
- (+) G6PD
- Cystic fibrosis [AR]
- Achondroplasia [AD]

4. All of the following are true about the risk malignancy index (RMI) except:
- Higher score in postmenopausal compared to premenopausal
- Score of malignancy referral > 200
- Used to differentiate between malignant and benign ovarian tumors
- US rather MRI score is used
- Serum CA125 is important

**ALL of them seem CORRECT!


**RMI = U x M x CA125
- The ultrasound result is scored 1 point for each of the following characteristics: multilocular
cysts, solid areas, metastases, ascites and bilateral lesions. U = 0 (for an ultrasound score of 0),
U = 1 (for an ultrasound score of 1), U = 3 (for an ultrasound score of 2–5).
- The menopausal status is scored as 1 = pre-menopausal and 3 = post-menopausal
- The classification of 'post-menopausal' is a woman who has had no period for more than 1
year or a woman over 50 who has had a hysterectomy.
- Serum CA125 is measured in IU/ml and can vary between 0 and hundreds or even thousands
of units.

4
5. The least anticonvulsant to cause congenital anomalies during pregnancy:
- Valproate [Most teratogenic]
- Carbamazepine
- Sodium valproate
-(+) Phenothiazine
- Phynotoin

6. All of the following are true about the failure rate of contraception except:
- (+) implanon 2-4%

**Natural methods: 15% Failure Rate


Mini pills: 2-3%
Combined pills: 1-2%
Injectable progesterone (Depo-Provera): <1%
Implanon: 1%

7. Which one doesn't cause restricted growth:


- Advanced DM
- Connective tissue diseases
- Rh isoimmunization

**ALL seem correct!


**Causes of IUGR:
1. Fetal factors (reduced fetal growth potential): Chromosome defects (trisomy 18, triploidy),
Single gene defect (sickle's syndrome), Structural abnormalities (renal agensis), Infections
(CMV, Toxoplasmosis)
2. Maternal factors (reduced fetal growth support): Undernutrition (poverty, eating disorder),
Maternal hypoxia (altitude, CHD), Drugs (smoking, alcohol, cocain), Advanced diabetes, Uterine
malformation
3. Placental factors (reduced fetal growth support): Reduced uteroplacental perfusion
(inadequate trophoblast invasion, antiphospholipid syndrome, DM, sickle-cell disease, multiple
gestation), Reduced feto-placental perfusion (single umbilical artery, twin-twin transfusion
syndrome), Infarction, Abruption, placental tumors TTTs, Velamentous cord insertion

5
8. The presentation that's associated with abnormal lie:
- (+) Shoulder
- Face
- Breech
- Vertex
9. Which of the following is not an abnormal presentation:
- Face
- Breech
- Brow
- (+) Occipitoposterior [It means that presentation is vertex since Occipit is the dominator in
vertex presentation]

10. Not true about the management of septic abortion:


- (+) Immediate curettage [Should be after 12 hours after antibiotic therapy]

11. Which one of the following is not a risk factor for preeclampsia:
- (+) Smoking
- Multiparity

12. The tumor marker for granulosa cell tumor:


- (+) Inhibin
- AFP
- Ca125

13. Mismatch:
- Granulosa cell tumor >> Sex cord
- (+) Thecoma >> Germ cell [sex cord]
- Yolk sac tumor >> Germ cell
- Dysgerminoma >> Germ cell

14. One of the following doesn't cause post-coital bleeding:


- (+) Chlamydia trachomatos [causes intermesntrual bleeding]
- Cervical polyp
- Ectropion
- Cervical intraepithelial neoplasm

6
15. A patient complains of bleeding 12 days after vaginal delivery, one of the following is
true:
- (+) Put the patient on antibiotics [as endometritis is one of the causes of secondary PPH]

16. A patient complains of vaginal bleeding 10 days after vaginal delivery, what's the most
likely cause:
- (+) Retained products of conception [Most common cause of secondary PPH]

17. All of the following are true about regional anethesia in obstetrics except:
- Epidural needs higher dose than spinal
- Can cause meningitis
- Epidural can cause headache
- (+) Epidural can cause hypotension less than spinal
- Contraindicated in coagulopathy

18. All of the following can cause organic menorrhagia except:


-(+) PCO
- Copper IUCD
- Endometriosis
- Adenomyosis

19. One of the following is false about hypertension in pregnancy:


- Eclampsia doesn't necessarily need to have high blood pressure

**Eclampsia in the absence of hypertension with proteinuria has been demonstrated to occur
in 38% of cases reported in the United Kingdom. Similarly, hypertension was absent in 16% of
cases reviewed in the United States. [Medscape] but make sure of it!

20. One of the following is false about incomplete abortion:


- Diagnosed clinically

**Inevitable and incomplete abortions are diagnosed clinically, US is essential for the diagnosis
of threatened and missed abortions.

7
21. One of the following is correct about gestational DM:
- (+) Increase the incidence of shoulder dystocia
- Induce delivery at 37 weeks
- Most of them treated with insulin
- Always present after 24 gestational weeks

22. A patient with retinopathy and nephropathy may have all the following when getting
pregnant except:
- Placental insufficiency
- (+) Fetus weight more than 90th percentile [as it is advanced DM]
- Congenital anomalies
- Restricted growth

23. All of the following are associated with ectopic pregnancy except:
- Tachycardia
- Bathroom sign
- (+) Heavy vaginal bleeding
- Excited cervix

24. All of the following are part of early investigations of urinary incontinence except:
- (+) Urodynamic study
- Pelvic US
- Bladder diary
- History
- Urinalysis

25. Which of the following genital prolapsed is considered a true hernia:


- (+) Enterocele

26. One of the following is the latest sign of puberty:


- Pubic hair
- Axillary hair
- Breast enlargement
- (+) Menstruation [according to Tanner staging]
- Growth spurt

8
27. All are correct about ITP except:
- Increase the risk of fetal intracranial hemorrahge
- Delivery by CS has no improvement on fetal outcome
- Steroid initially used in the treatment
-(+) Platelet transfusion is common
- Treatment is needed only if platelet count is less than 50,000

28. Not a contraindication for induction of labor:


- Previous 1 classical CS
- (+) Vaginal prolapse [Previous repair of prolapse is a contraindication]
- Abnormal CTG
- Transverse lie

29. Progesterone-only contraceptives cause all of the following except:


- Weight gain
- Menstrual irregularity
- Acne
- Breast tenderness
- (+) Complex cysts [it causes simple cysts]

30. About fibroid, all are true except:


- More in blacks
- (+) COCPs are a risk factor
- Family history is a risk factor
- Long reproductive age before menopause [Early menarche is a risk factor]

31. Wrong about dysgerminoma:


- Bilateral in 10%
- Radiosensitive
- (+) Associated with hirsutism
- Common before the age of 30
32. All can cause fetal distress except:
- Supine position
- Lupus erythromatosus
- Abruption placenta
- Pethidine
- (+) Renal disease
9
33. Placental abruption all true except:
- PPROM can be a cause and a result
34. All are side effects of D&C except:
- (+) Endometriosis

**Complications of D&C: 1) uterine perforation which may lead to uterus rupture in the
subsequent pregnancy 2) cervical tear and excessive cervical dilatation which may lead to
cervical incompetence 3) infection which may lead to infertility and Asherman's syndrome 4)
excessive curettage which may lead to adenomyosis

35. 26 years old lost 16 kg of her weight in the last month, and she was doing heavy exercise,
you need to do all these investigations except:
- TFT
- Prolactin leve
-(+) bHCG
- Brain CT scan

36. Incomplete abortion underwent E&C, few months later presented with amenorrhea, the
most likely diagnosis:
- (+) Asherman's syndrome [It could be! Depending on the other options that couldn't be
remembered]
- PID

37. Placental abruption, you do all these investigations except:


- KFT
- (+) Urinalysis and culture
- CBC
- 24 hour urine collection [as is can be a consequence of PET]

38. All about benign GTN are true except:


- (+) Blood group has a role in the scoring
- Passage of the vesicles is diagnostic
- Suction and curettage are the TOC for benign GTN
- Induction of delivery is contraindicated

10
39. One of the following is not a normal complaint in pregnancy:
- (+) Itching
- Heartburn
- Abdominal pain
- Leg cramp
40. About menopause:
- Mean age 51
- Smoking lead to earlier menopause
- Premenopausal hysterectomised patients also enter menopause at 51
- Radiotherapy for breast can lead to earlier menopause
- Hot flashes are common in the transition and postmenopausal period
- HRT are contraindicated in women with fibroid

**Wallahi ALL of them seem correct!!

41. Goes with benign ovarian mass:


- Unilateral
- Adherent

42. Favors the diagnosis of placenta previa:


- Transverse lie
- Soft and non-tender abdomen
- Alive baby

**I think this question was supposed to be with except

43. All are causes of uterine atony except:


- Precipitous labor
- Multiple pregnancy
- (+) Oligohydraminous

44. Not a cause of infertility in endometriosis:


- Tubal obstruction
- Sperm motility will be affected
- Infrequent coitus
- (+) Hyperprolactinemia

11
45. All are correct about CVS disease in pregnancy except:
- (+) Usually CS is preferable (something like that)
- Mitral stenosis is the most common lesion in rheumatic heart disease
- Rhemutic heart disease is the most common in pregnancy
- Correction of anemia can help the patient

**The indication for cesarean section is based solely upon obstetric indications because vaginal
delivery is generally preferred in mothers with most CVDs. Elective cesarean section has been
reported to increase the risk of hemorrhage, thrombosis, and infection. [Medscape]
**In the past, Rheumatic heart disease was the most common form of cardiac disease in
pregnant women, it still predominates in devloping coutries and in immirgrant populations in
the US. Congenital heart disease is now the most common of heart disease complicating
pregnancy in the US, in part because advances in the treatment of CHDs have made it possible
for more affected children to reach adulthood and attempt pregnancy. [UpToDate]

46. Not part of management of fetal distress:


- Put the patient on her side
- O2 mask
- IV fluid
- (+) Oxytocin infusion

47. One of the following has the least teratogenic effect:


- Valporate
- Valporic
-(+) Phenytoin
- Phenobarbital
- Carbamazipen

48. One of the following is false:


- Female with epilepsy has higher chance of fetal malformation
- Increase the demise outcome of delivery
- (+) Lactation is contraindicated
- Phenobarbital is the drug of choice in the first trimester

**Most experts believe that taking AEDs doesn't generally contraindicate breast feeding, as
probable benefits outweigh risks. [UpToDate]

12
49. 22 years old female sexullay active presented with vaginal discharge and deep
dysparunia, gonorrehia culture is negative, the most likely cause is:
- Gonorrehia
- (+) Chlamydia
- Bacterial vaginosis
- Candidal infection

51. Not a cause of Fetal Growth Restriction:


- Advanced Diabetes
- Inadequate maternal nutrition
- PET
- Collagen disorders
- Microvascular disease

** ALL of them seem correct!!

52. About ovarian tumor one is wrong:


- (+) Most common histological is muscinous [most common is serous]

53. About postmenopausal bleeding, all true except:


- (+) Always pathological
- Estrogen can be the cause and the treatment
- Most common cause is atrophic vaginitis
- Always take biopsy

54. Time needed for uterus to become pelvic organ:


- (+) 2 weeks
- 4 weeks
- 6 weeks
**I think the question is asking about postpartum period.

55. Ruptured Tubal pregnancy, one is true:


- Pain can be upper abdominal [?? - it could be in rare cases!!]
- Coincides with vaginal bleeding

13
- Colicky pain
- (+) Always unilateral

56. All can be used in the treatment of DUB except:


- Progesterone only pills
- OCPs
- Mefenamic acid
- Tranexamic acid [Antifibrinolytic]
- IUCD

**ALL of them can be used!!

57. One is false about normal pregnancy:


- Breech presentation at 28 weeks [May mean that breech presentation is most common at 28
weeks]
- Fundus is usually first felt at 20 weeks
- (+) At 36 weeks the fundus is at level of umbilicus [it should be higher]

58. All are signs of ruptured ectopic except:


- Shoulder tip and cervical pain
- (+) Massive vaginal bleeding [??]
- Bathroom sign
- (+) Dysurea
- Vasomotor symptoms

59. One drug is teratogenic:


- Diazepam
- (+) ACEI
- Baby aspirin

60. Gonorrhea infection doesn't cause:


- Conjunctivitis
- Skin discoloration [Gonorrhea can cause dermatitis in the form of pustular or vesicopustular
usually, but hemorrhages macules, papules, bullae, or nodules may rarely occur. Also it can
rarely cuase erythema multiforme or nodosum. - UpToDate]
- Painful swollen testicles

14
61. All are risk factors for fibroid except:
- Nulliparity
- African
- (+) OCP
- Increase reproductive years

62. One is mismatched:


- OCP: 1%
- IUCD: 2%
- (+) Implanon: 3-5%

63. In Placenta previa, the main fatal complication is:


- (+) Prematurity
- IUGR
- Anemia

64. Inhibin is a tumor marker of:


- (+) Granulosa cell tumor
- Fibroma
- Dysgerminoma
- Sinus tumor

65. Not a step in the expectant management of 32-week placenta previa:


- Watch for bleeding
- Correct anemia
- (+) Tocolytic agent
- Steroids
- US

66. First sign of DIC:


- (+)Thrombocytopenia
- Bleeding from the cannula site
- Hematuria
-Non-clotting blood

15
67. Not in biophysical profile:
- Breathing
- Fine movement
- Gross movement
- Amount of liquor
- (+) Doppler fetal monitor
68. One is not part of fetal biophysical profile:
- AFI
- Fetal heart rate
- Gross movement
- Breathing tone
- (+) Umbilical doppler ultrasound
69. Woman presented 12 days post partum with vaginal bleeding and lower abdominal pain,
what is included in the management:
- (+) IV antibiotics
70. Neisseria gonorrhea is:
- (+) Bacteria
- Virus
- Protozoa
- Fungus
71. Ritodrine:
- Has comparable effectiveness to atosiban
- (+) Has an effect mainly on beta-1 receptors
- Beta agonist
- Usually causes tachycardia
- Not used anymore as first line because of its side effects
72. All are recommended treatment of endometriosis, except:
- Continuous COCP
- (+) Laser
- GnRH agonists
73. About bacterial vaginosis, which is false?
- (+) Usually symptomatic
- Fishy odor
- Increase in pH is one point of Amsel criteria
- Should be treated in pregnancy to prevent complications

16
2. 6th Year Final Exam 2014
1) All of the following are benefits of ANC, except:
A. Early PET detection
B. Prevents Anemia
C. Prevents Prematurity
D. Prevents Congenital anomalies

2) Comparing abortion to ectopic, only one of the following is true:


A. Shedding of the decidua can be mistaken for products of conception
B. Endometrial curette can rule out ectopic
C. Abortion causes more pain
D. More vaginal bleeding in ectopic

3) A pregnant lady, 7 week gestation, presented with mild vaginal bleeding and severe abdominal
pain. In which condition of the following methotrexate can be used:
A. BhCG level 3500, no intrauterine gestational sac is seen by Abdominal US
B. BhCG level 2500, no intrauterine gestational sac is seen by transvaginal US
C. Adnexal mass with positive heart beat

4) In the management of infertility, All of the following are true except:


A. Unexplained in 30%
B. Hypothalamic pituitary dysfunction is the cause in PCOS
C. Management should be directed more toward females than males.
D. Male factor is the cause in 40% of cases

17
5) Which of the following is an advantage of myomectomy over hysterectomy in the management of
fibroids?
A. Spares reproductive function
B. Less bleeding
C. Less infection
D. Spares hormonal function

6) A patient with secondary amenorrhea who is being investigated infertility, which of the following
is not an important investigation?
A. FSH
B. Serum testosterone
C. Serum androstenedione
D. Urinary hCG
E. Prolactin

7) All are true regarding Rh alloimmunization, except:


A. If a sensitized Rh –ve woman, undergoes amniocentesis, she should receive antiD after the
procedure
B. Negative KB test for Rh –ve woman, she doesn’t need antiD prophylaxis
C. External cephalic version is an indication for prophylaxis
D. More likely to occur if mother and fetus are ABO compatible

8) The main maternal complication for severe placental abruption is:


A. IUFD
B. Real tubular necrosis
C. Renal cortical necrosis
D. Hemorrhagic shock
E. Amniotic fluid embolism

18
9) Pregnant lady presented at 33 weeks, singleton, cephalic, with severe placental abruption, all
should be done except:
A. Urinary catheter
B. Vaginal examination
C. Give steroids

10) 25year old pregnant, presented with severe placental abruption, CTG showed multiple
decelarations. Your management is:
A. Left lateral position and O2 mask
B. Induction of labor
C. Emergent C/S

11) Regarding placenta previa, what is false?


A. Speculum examination is contraindicated
B. Increased risk of IUGR
C. Increased risk of congenital anomalies

12) All are side effects of progesteroneonly pills EXCEPT


A. Acne
B. Weight gain
C. Functional ovarian cyst
D. Inhibits lactation

13) All are disadvantages and/or contraindications for Copperreleasing IUCD, except:
A. Reduces menstrual bleeding
B. Uterine malformation
C. History of PID
D. History of ectopic pregnancy

19
14) All are modalities of tubal ligation except
A. Rings
B. Clips
C. Hysterectomy
D. Saplingectomy
E. Diathermy

15) Monozygotic twins with uncompensated AV shunts, all are true except:
A. Donor twin will have polyhydramnios
B. Donor will have have mirocardia
C. Diagnosis when Hb difference is 50 g/L

16) All are true regarding twin pregnancy, EXCEPT


A. Antenatal betamethasone decrease perinatal mortality
B. Cervical cerclage prevents preterm labor
C. Twenty percent are breech presentation
D. Second twin should be delivered within 30 minutes of the first

17) All are true regarding GTN except:


A. BhCG is always higher than pregnancy levels
B. Almost always presents with large for date uterus
C. Methotrexate is indicated for low risk malignant GTN
D. Most common karyotype for complete mole is 46 XX
E. Usually presents with vaginal bleeding in the first trimester

18) All are true regarding GTN except


A. Chemotherapy is indicated if BhCG levels plateau for 2 weeks
B. Serum BhCG is used for followup
C. Incomplete mole contains fetal tissue
20
D. Snowstorm appearance is characteristic
E. Suction evacuation is the treatment of choice

19) All of the following HPV serotypes are associated with cervical cancer except:
A. 16
B. 6
C. 31
D. 33
E. 18

20) All are true regarding cervical cancer EXCEPT:


A. Surgery is the cornerstone of treatment for all stages
B. Postcoital bleeding is a common presentation
C. Caused by persistent highrisk HPV infection

21) All are true regarding colposcopy, except:


A. 35% alcohol is used to shed debris
B. Mosaic pattern and punctation are abnormal findings
C. Colposcope is a stereotactic binocular microscope
D. Columnar epithelium appears reddish and grapelike

22) All of the following are true regarding Cervical smear EXCEPT
A. HPV testing is beneficial in ASCUS
B. HSIL should be referred for colposcopy
C. Squamous epithelium stains mahogany brown by Lugol’s iodine
D. Leukoplakia is a normal finding
E. Indicated at 21 years or 3 years after the onset of sexual activity

21
23) Regarding Tensionfree Vaginal Tape (TVT), Only one of the following is correct
A. Used for overflow incontinence
B. Success rate 65%
C. Should be done under local analgesia
D. Detrusor instability is a known complication
E. Dysfunctional voiding is a side effect in 15%

24) All of the following are true regarding urinary incontinence, except
A. Substracted cystometry is a useful investigation
b. Anticholinergics are useful for treatment of stress incontinence
C. 2030% of adult female population are affected

25) All are symptoms of urinary incontinence, except:


A. Frequency
B. Straining to void
C. Urge incontinence
D. Stress incontinence
E. Overflow incontinence

26) All of the following are taken in consideration in the choice of endometriosis management,
except:
A. Age
B. Desire to get pregnant
C. Size of endometrioma
D. Score of disease severity
E. Severity of symptoms

27) Regarding endometriosis, only one is correct


A. 65% affects the ovaries
22
B. Fixed retroverted uterus is a normal variant
C. Extent of disease is related to severity of symptoms

28) 35 year old, para 1 by C/S before 6 years, came complaining of secondary dysmenorrhea, pelvic
U/S was normal. What is the most likely diagnosis?
A. Adenomyosis
B. Endometriosis
C. Submucous fibroid
D. Psychosomatic

29) About GnRH analogue, all are true except:


A. Used in precocious puberty
B. Optimal management of adenomyosis
C. It is antiestrogenic

30) Pregnant (10 weeks), presented with severe lower abdominal pain. On examination, uterus is
consistent with 10 weeks, she was found to have open external os, the most likely diagnosis is
A. Inevitable abortion
B. Missed abortion
C. Complete abortion
D. Ectopic pregnancy

31) All of the following could be a cause of recurrent abortions except:


A. Toxoplasomosis
B. Thromboplilia
C. Submucus fibroid
D. PCOS

23
32) Regarding viral infections in pregnancy, All of the following are true except:
A. Toxoplasmosis is a viral infection.
B. CMV transmission risk is 40%

33) A young patient presented with mild vaginal bleeding with severe abdominal pain, what is your
first step in management:
A. Assess hemodynamic stability
B. Serum BhCG
C. Ultrasound

34) All of the following can cause hemolytic disease of newborn, EXCEPT:
A. G6PD deficiency
B. Hexokinase deficiency
C. Sickle cell trait
D. Hereditary spherocytosis

35) All of the following are true about sickle cell disease in pregnancy except:
A. Chronic anemia
B. Bone marrow erythroid hyperplasia
C. IUGR
D. Iron deficiency anemia

36) All of the following are caused by chromosomal abnormalities EXCEPT:


A. Turner syndrome
B. Edward syndrome
C. Potter’s syndrome
D. Myeloid leukemia
E. Sickle cell anemia

24
37) All of the following are true about endometrial cancer are true except:
A. Decreasing in incidence
B. Up to 25% occur in premenopausal women
C. Confined to the endometrium in stage 1a
D. Mean age is 61
E. Most common type is adenocarcinoma

38) Regarding endometrial hyperplasia, Which one of the following is not true:
A. In young women it is treated by endometrial ablation
B. Old people can be treated by hormonal drugs safely
C. It is diagnosed only by biopsy
D. It is a premalignant lesion

39) Ca125 may be raised in all of the following except:


A. Menstruation
B. PID
C. Pregnancy
D. In 80% of mucinous cystadenoma
E. Endometriosis

40) US in a 21yearold female showed an ovarian mass with calcification and bony spicule, what is the
most probable diagnosis ?
A. cystic teratoma
B. Clear cell carcinoma

41) All of the following are true about granulosa tumors except:
A. It is known to have low grade malignant potential
B. Progesteronesecreting tumor

25
42) A young women presented with SOB, found to have pleural effusion and bilateral ovarian
masses, what is your diagnosis?
A. Fibroma

43) All of the following are true about PCOS, except


A. It may be linked to atherosclerosis
B. Increased serum 17hydroxyprogesterone
C. Increased risk of development of diabetes type II
D. Ovulation may be normal
E. It can cause ovulatory amenorrhea

44) All of the following are true about HELLP syndrome except:
A .More in primigravida
B. May be confused with hemolytic uremic sydrome
C. LDH can be elevated
D. Liver enzymes can be elevated
E. Presents with vague epigastric pain

45) All of the following are indications of severity in PET except:


A. Intracranial hemorrhage
B. IUGR
C. 5 gm proteinuria in 24 hours
D. Polyuria

46) All of the following are changes in PET, except:


A. hemodilution

47) All of the following are true about PET except:


A. Diazepam is used to prevent seizures
26
48) Which of the following is not true about MgSO4:
A. Can cause respiratory depression
B. Can cause cardiac toxicity
C. Absence of deep tendon reflexes is indication to increase the dose

49) All of the following increase the risk of breech presentation except:
A. Congenital malformation
B. Multiple pregnancy
C. Full maternal bladder

50) All of the following are risk factors for face presentation, EXCEPT
A. Multiple pregnancy
B. Multiparity
C. Contracted maternal pelvis
D. Anencephaly
E. Fetal goitre

51) All of the following are true regarding transverse lie EXCEPT:
A. In most of the cases the back is anterior
B. 15% risk of cord prolapse
C. 10% Associated with placenta previa
D. Transverse lie of the second twin is an indication for CS
E. In most of the cases the head is on the maternal left side

52) A lady presented after 72 hours with fever of 38 degree, All of the following are true except:
A. UTI is a cause in 5% of cases
B. Breast engorgement is a common cause

27
C. Length of labor is not relevant
D. DVT should be ruled out

53) Which of the following is NOT true about lactation


A. Oxytocin is released from the anterior pituitary
B. Prolactin is released from the anterior pituitary

54) All of the following are used in the management of uterine atony except:
A. Blynch procedure
B. Bakri balloon
C. MgSO4
D. Progesterone analogue

55) A patient just delivered an alive baby, had a massive vaginal bleeding, which of the following is
correct in her management?
A. IM ergometrine
B. IV oxytocine with uterine massage
C. bimanual compression
D. exploratory laparotomy

56) All of the following are true regarding PPH except:


A. Most of the secondary PPH occurs during the first week
B. Most common cause of primary PPH is atony
C. Most common cause of secondary PPH

57) An a febrile ( it was written like this in the exam) presented 10 days after delivery with lower
abdominal pain and mild vaginal bleeding, What is true regarding her management
A. Reassurance
B. Immediate evacuation
28
C. Antibiotics orally for 7 days then review her condition
D. IV antibiotics followed by evacuation after 12 hours

58) A P1 patient, her first delivery by CS, just gave birth to a live child vaginally and the placenta was
completely delivered, is now having heavy vaginal bleeding and generalized abdominal pain, what is
your diagnosis:
A. Uterine Atony
B. Uterine Inversion
C. Uterine rupture
D. Retained products of conception

59) All of the following are true regarding Hyperthyroidism in pregnancy, all of the following are true
except:
A. Could be a cause of preterm labor and stillbirth
B. neonatal hyperthyroidism in offspring of grave’s is often transient
C. Agranulocytosis is an indication for withdrawal of thioamides
D. May be caused by Hashimoto’s thyroiditis E. PTU is the drug of choice

60) Which of the following is NOT true about cardiac diseases in pregnancy?
A. Uncomplicated coarctation of the aorta has 515% mortality rate
B. Isolated ASD carries 30% mortality
C. Rheumatic heart disease is the most common cause in developing countries
D. Pulmonary hypertension may cause sudden mortality death

61) Regarding Cardiovascular changes in pregnancy and lactation, only one of the following is
correct:
A. Peripheral vascular resistance initially rises during pregnancy
B. 40% increase in CO by 12 weeks
C. Cardiac output decreases in the third trimester
29
D. Cardiac output increases during labor
E. Breastfeeding increases cardiac output

62) Regarding regional anesthesia, all of the following are true except:
A. Meningitis is a complication
B. Headache is a complication of spinal anesthesia
C. Epidural works faster than spinal
D. Coagulopathy is a contraindication for spinal anesthesia

63) Pregnant lady in labor, 4 cm dilation, she is having 3 contractions per 10 minutes each lasting for
45 seconds. All of the following are true EXCEPT:
A. Pethidine use is safe
B. Should be augmented with oxytocin
C. May take 2 hours in primi
D. She is in the active phase of the first stage

64) Regarding amniotomy, only one of the following is true:


A. Speeds up the progress of labor

65) Regarding the second stage of labor, which one of the following is not true
A. Fetal heart should be assessed every 5 minutes
B. Caput is diagnostic of CPD
C. Epidural can prolong the second stage of labor

66) All of the following are complications of postmaturity except:


A. Meconium aspiration syndrome
B. RDS
C. Oligohydromnios

30
67) All of the following can cause polyhydromnios, EXCEPT:
A. Esophageal atresia
B. Diabetic mother
C. Anencephaly
D. “easy answer”

68) All of the following are true regarding Preterm labor except:
A. Complicates 510% of pregnancies
B. Home uterine activity monitoring was proved to decrease the risk
C. Positive fetal fibronectin indicates high risk for preterm delivery
D. Previous preterm is a strong risk factor

69) All of the following are true about PPROM except:


A. Can be caused by cervical insufficiency
B. Can be caused by candidal vaginal infection
C. Erythromycin is the antibiotic of choice

70) All of the following are true about gonorrhea except:


A. Can be effectively presented by vaccination
B. Best treated by penicillin
C. Gram negative cocci
D. Incubation period is 28 days
E. Can cause more disseminated disease in females than males

71) All of the following are true about STDs except:


A. Chlamydia is the leading cause of blindness in the world
B. Gonorrhea primarily infects the vagina
C. Syphilis is a systemic disease caused by a spirochete

31
72) All of the following are risk factors for GBS infection except:
A. Previous GBS infection
B. Breech presentation
C. ROM > 18 hours
D. Fever > 38

73) SGA newborns are at the risk of one of the following :


A. Anemia
B. Hypothermia
C. Hyperglycemia

74) All of the following are true about small for gestational age (SGA) except:
A. Uterine artery doppler study can detect high risk infants
B. Biophysical profile assessment is less important for at risk patients
C. Longterm risk of DM

75) All of the following can cause deep dyspareunia except:


A. Endometriosis
B. Ovarian cyst
C. Vaginismus
D. Fibroid

76) All of the following are associated with low GnRH except:
A. Turner syndrome
B. McCuneAlbright syndrome
C. Anorexia nervosa
D. Kallman syndrome

32
77) The highest number of oogonia is found at which age:
A. 1 month in utero
B. 5 months in utero
C. At birth
D. Puberty
E. 21 years

78) All of the following are contraindications for vacuum EXCEPT:


A. Frank breech
B. Previous fetal scalp sampling
C. Mother is hemophilia carrier
D. Less than 33 weeks

79) Regarding instrumental vaginal delivery, one of the following is true :


A. Knowing the position is not important
B. Submentobregmatic presentation can be delivered by vacuum
C. High cavity instrumental delivery is the most common
D. Kielland forceps is used for rotation and traction

80) All of the following are complications of forceps delivery EXCEPT:


A. Sphincter injury
B. Cephalhematoma
C. Facial nerve injury
D. Spiral vaginal tear
E. Facial laceration

81) All of the following are true about fibroids except:


A. More in caucasians
B. Submucosal fibroids can cause subfertility
33
82) All of the following are true about fibroids except:
A. The most common presentation in pregnancy is red degeneration
B. HRT can be used in postmenopausal women with uterine fibroid
C. Menorrhagia is the most common presentation
D. Therapeutic Hysteroscopy can be used in the management of submucosal fibroid

83) A 30yearold patient with DUB, all of the following are possible treatment modalities for
treatment except:
A. Endometrial ablation
B. Mirena
C. Mefenamic acid

84) Regarding congestive dysmenorrhea, one of the following is true


A. Starts in the first day and ends in the 3rd day
B. Can be ovulatory or anovulatory
C. Associated with diarrhea and vomiting
D. Caused by Increased Prostaglandins in the second half of the cycle
E. COCP is a specific treatment

85) Regarding abdominal pain in pregnancy, all of the following are true except:
A. Cholecystitis is the second most common cause
B. Red degeneration is treated by myomectomy

86) Regarding appendicitis in pregnancy, All of the following are true except:
A. Incidence is less during pregnancy
B. Perforated appendix has higher maternal mortality
C. Difficult to diagnose

34
87) Regarding cord prolapse, one of the following is true
A. A common cause for intrapartum stillbirth
B. Incidence is 1 in 5000
C. The initial management is elevation of the presenting part and expedited delivery

88) Regarding shoulder dystocia, one of the following is true:


A. Site of obstruction is the pelvic outlet
B. Many affected babies are of normal birth weight
C. 70% risk of recurrence

89) Which one of the following can reduce C/S incidence


A. Regular antenatal care
B. External cephalic version
C. External fetal monitoring
D. Internal heart monitoring

90) Comparing LSCS with classical C/S, all of the following are advantages of LSCS except:
A. Less intestinal adhesions
B. Less bleeding
C. Less ureteric injury
D. Less paralytic ileus
E. Less risk of spontaneous rupture in subsequent pregnancies

91) All of the following are useful in the diagnosis of NTD except:
A. Maternal serum AFP
B. Amniotic fluid AFP
C. Amniotic fluid acetylcholinesterase
D. Karyotyping
E. Pelvic US
35
92) Regarding HBV infection during pregnancy, All of the following are true except:
A. Newborn of infected mother should receive immunoglobulins and vaccine soon after birth
B. HBV vaccine is contraindicated in pregnancy
C. Infection during the third trimester has a higher risk of transmission

93) Which one of the following patients should be managed expectantly with steroids:
A. 32 weeks placenta previa, mild vaginal bleeding, no other comorbidities
B. 37 weeks placenta previa with vaginal bleeding

94) A patient with poorly controlled insulindependent DM is at higher risk of all of the following
except:
A. Acute pyelonephritis
B. Hypertension
C. Caudal regression syndrome
D. Klinefelter syndrome

95) All of the following are poor prognostic factors for endometrial cancer EXCEPT:
A. Elderly
B. Hypertension and diabetes
C. Invasion of lymphatics
D. Cervical involvement

96) Regarding gestational diabetes, Only one of the following is true


A. Fasting blood sugar may be normal
B. Increase the risk of congenital anomalies

97) Regarding OCPs All of the following are true except:


A. Inhibits ovulation
36
B. Alter cervical mucus
C. Decrease the risk of endometrial cancer
D. Increase the risk of ovarian cancer

”‫ والعاجز من أتبع نفسه هواها وتمنى على هللا األماني‬،‫“الكّيس من دان نفسه وعمل لما بعد الموت‬

3. 5th Year Final Exam 2013:


Note: The exam was 100 questions, 2 hours, 2 forms. Most of the questions were the same in both
forms.

1. A lady had her last menstrual period on 27/12/2012. What is her expected date of delivery?

A. 03/10/2013
B. 20/09/2013
C. Cannot be calculated

Note: This Q seems easy and most of us had chosen "A", but the problem is that it didn't
mentioned whether the Cycle was regular or not… "The "Sure" criteria  "

2. A lady presented with uterine contractions (3 per 10 minutes). The cervix is 4 cm dilated and 1
cm long, what’s the stage of labor?

A. Active first stage

B.+Latent phase

C. Active second stage

D. Third stage

E. She is not in labor

3. A baby with breech presentation. Hips are flexed and knees are extended, what’s the type of
breech?

37
A. Complete

B. Incomplete

C. +Frank

4. In vertex presentation, what’s the bony denominator of the presenting part?

A. +Occiput
B. Sinciput
C. Chin

5. The relation of the presenting part of the fetus to maternal ischial spines is called….

A. Lie

B. Position

C. +Station

D. Presentation

6. All of the following are correctly matched, except:

A. Vertex --- Suboccipito-bregmatic


B. +Brow --- Occipito-frontal
C. Face --- Submento-bregmatic

7. Regarding normal labor, all of the following are true, except:

A. Second stage starts with full cervical dilation and ends with delivery of the baby

B. +True uterine contractions can be suppressed by epidural anesthesia

C. Third stage is delivery of placenta and the membranes

D. Second stage takes longer in primigravida compared to multiparous

E. False contractions don’t cause cervical dilation

38
8. Regarding the mechanism of normal labor, all of the following are true, except:

A. Fetus enters the pelvis in the transverse diameter

B. +Internal rotation is for fetal shoulders

C. The anterior shoulder is delivered before the posterior

D. The head is delivered in extension

9. All of the following are classes of tocolytics, except:

A. Calcium channel blockers

B. +Anticholinergics

C. Oxytocin antagonists

D. Prostaglandin inhibitors

10. All of the following are correctly matched, except:

A. Face --- presentation

B. Flexed --- Attitude

C. Transverse ---- Lie

D. + Vertex --- Position

11. Which of the following is considered an abnormal finding in CTG?

A. Early deceleration in second stage


B. +Late deceleration

C. Baseline heart rate of 120

D. Absence of accelerations

39
13. Which of the following is NOT part of the routine partogram?

A. Vital signs

B. +Pelvimetry

C. Cervical dilation

D. Strength of contractions

14. Regarding fetal skull in relation to maternal pelvis, all are true, except:

A. Platypeloid pelvis is likely to cause obstructed labor

B. Android is the typical male pelvis

C. +Gynecoid pelvis forces the baby into an occipito-posterior position

15. All of the following are true regarding spinal anesthesia, except:

A. +Can cause hypertension

B. Meningitis is a possible complication

C. Contraindicated in case of coagulopathy

D. Contraindicated if skin is infected at the site of needle

16. Regarding the use of pethidine in labor, all are true, except:

A. Can cause neonatal respiratory depression

B. Can cause neonatal bradycardia

C. Easily administered

D. Naloxone is the antidote

E. +Inhibits uterine contractions

40
17. A baby in breech presentation, where would you listen to the fetal heart?

A. +Above the umbilicus on the side of the back

B. Above the umbilicus opposite to the back

C. Below the umbilicus on the side of the back

D. Below the umbilicus opposite to the back

E. At the umbilicus

18. All of the following are risk factors for vaginal candidiasis, except:

A. +Thyrotoxicosis

B. Oral contraceptive use

C. Pregnancy

D. Diabetes

19. Regarding Trichomonas vaginalis infection, only one of the following is correct

A. Commonly causes salpingitis

B. +It’s a parasitic STD that causes a pruritic discharge.

C. Clue cells are characteristic

20. Regarding vulvovaginal candiasis, only one of the following is correct:

A. +can be microscopically diagnosed by using KOH

B. Muco-purulent vaginal discharge

21. By definition, PID is:

A. Infection of the vagina

B. Infection of the cervix

C. +Endometritis and salpingo-oophoritis

41
22. The main postpartum complication of placental abruption is:

A. +Post partum hemorrhage

B. Sheehan syndrome

C. Renal tubular necrosis

D. Renal cortical necrosis

E. Endomyometritis

23. Placenta previa is associated with increased risk of all of the following except:

A. IUGR

B. Primary PPH

C. +Prolonged pregnancy

24. Regarding postpartum hemorrhage, all of the following are true, except:

A. Atony is the most common cause of primary PPH

B. Infection of RPOC is the most common cause of secondary PPH

C. +Secondary PPH occurs between 24 hours and 6 months postpartum

24. A lady presented with vaginal bleeding 12 days postpartum, all of the following are true,
except:

A. This is secondary postpartum hemorrhage

B. +Antibiotic therapy has no role

25. The main risk factor for placental abruption is:


A. Primigravida

B. +PET

42
C. Maternal age

26. All of the following are true about PET except:

A. There’s a decrease in plasma volume

B. +There’s a decrease in peripheral vascular resistance

C. Uric acid elevation is the earliest lab sign

D. Thrombocytopenia is related

E. Elevated liver enzymes is a sign of serious complication

27. All of the following can be avoided with good ANC, except:

A. +Constitutional macrosomia

B. Syphilis

C. Hydrops fetalis
D. Prematurity

28. Gestational diabetes is associated with all of the following, except:

A. +Postmaturity

B. Fetal death

C. Neural tube defects

D. Hypertension

29. Regarding pre-eclampsia, all are true, except:

A. Recurrence rate is up to 10%


B. +Diazepam is used to prevent further convulsions

43
30. Regarding IUGR, only one of the following is correct

A. +Can be caused by advanced diabetes

B. Can be caused by dieting during pregnancy

C. Occurs at 35 weeks gestation

D. Usually associated with polyhydramnios

31. All of the following can be detected by Ultrasound at 12 weeks gestation, except:

A. Wrong date

B. Chorionicity

C. Triplets

D. +Growth restriction

32. All of the followings are part of Rh genetic subtypes, except:

A. D

B. +d

C. E

D. c

E. C

33. Regarding diabetes in pregnancy, only one of the following is correct:

A. The most common is type II

B. Same incidence of fetal anomalies compared to the population

C. +Glycosuria is unreliable in control assessment

D. Glycosylated hemoglobin level of 7-8 % is considered as controlled case

34. A lady who is 22-week gestational age is found to have a uterus consistent with 26 weeks, all
of the following are possible causes, except:

44
A. Full bladder

B. Wrong date

C. Ovarian cyst

D. Uterine fibroid

E. +Gestational diabetes

35. Regarding clomiphene citrate, all of the following are true except:

A. Can cause headaches, blurred vision and hot flushes

B. Can be used in days 2-6 of menstrual cycle

C. +It is a cyclical steroid estrogen with anitestrogenic activity

D. Can cause hyperstimulation syndrome

36. All of the following are progesterone-only contraceptives, except:

A. +Mifepristone

B. Implanon

C. Mirena

D. Norplant

E. Depo-provera

37. What is the most common cause of spontaneous miscarriage?

A. +Chromosomal abnormalities

B. Infection

C. Cervical incompetence
45
38. Regarding recurrent abortion, all are true except:

A. +Defined as three consecutive induced abortions

B. Anti-phospholipid is a cause

39. Regarding threatened abortion, all are true except:

A. U/S is essential for diagnosis

B. Cervix is closed

C. It is the most common type

D. Presents with mild vaginal bleeding

E. ??? The answer :\

40. Regarding menopause, all are true, except:

A. Vaginal and urethral epithelium are estrogen-dependant

B. +Collagen of bones and skin will not be affected by estrogen deficiency

C. Hot flushes is the most common symptoms

D. Vaginal wall will atrophy and collapse…

41. Regarding uterine fibroids, all are true, except:

A. HRT is contraindicated in patients with fibroids

B. Nulliparity and obesity are known risk factors

C. Hysterectomy is a modality of treatment

D. Red degeneration typically presents in the second trimester

E. +Hyaline degeneration occurs when the fibroid outgrows its blood supply
46
42. Regarding abortions, all are true, except:

A. Medical treatment is superior in second trimester abortions

B. +Subserosal fibroids are a known cause of abortion

C. Ovulation can occur as early as two weeks following abortion

43. All of the following surgeries are used to treat genital prolapse except:

A. Anterior colporrhaphy

B. Manchester

C. Le Fort’s

D. +Colposuspension

E. Sacrohysteropexy

44. All of the following are used in treatment of PPH except:

A. Ergotamine

B. +MgSO4

C. Oxytocin

45. Regarding HRT, all of the following are true, except:

A. +It’s the first line for prevention of osteoporosis in postmenopausal women

B. Is not considered in the prevention of coronary artery disease

46. All of the following are raised physiologically in pregnancy, except:

A. WBCs
47
B. +Platelets

C. Renal blood flow

D. Cardiac output

47. Regarding the reduction of incidence of preterm labor, all are true except:

A. +Smoking cessation has no role

48. Which of the following is the best to prevent PPH?

A. +Active management of third stage

B. Correction of anemia before pregnancy

B. Liberal use of blood transfusion

49. Regarding preterm labor, all are part of the conservative management, except:

A. +Frequent vaginal exam to assess cervical dilation

B. CTG to monitor the fetal heart

C. Ultrasound to monitor fetal weight and amount of liquor

50. Regarding PPROM, all are true, except:

A. Occur in 3% of pregnancies and is responsible for about one third of preterm births

B. +Antibiotics have no role in treatment

51. Regarding cervical incompetence, all are true except:

A. Ultrasound is essential for diagnosis

B. +Multiple pregnancy is a risk factor


48
C. Cone biopsy of the cervix is a risk factor

D. Best treated in early second trimester

52. All of the following are associated with IUGR except:

A. Hypocalcemia

B. Hypoglycemia

C. Meconium aspiration

D. +Anemia

53. Regarding placenta previa, all are true except:

A. Associated with IUGR


B. Associated with primary PPH
C. Associated with doubling in incidence of fetal abnormalities
D. More in patients with previous uterine surgeries
E. +It can be diagnosed clinically

54. All of the following are possible complications for induction of labor, except:

A. Hyperstimulation

B. Precipitate labor

C. +Postpartum hemorrhage

55. All of the following are indications for labor induction, except:

A. +Genital herpes

B. Chorioamnionitis

C. Rh-isoimmunization

49
56. All of the following are caused by endometriosis, except:

A. Cyclical hematuria

B. +Primary dysmenorrhea

57. Regarding Turner syndrome, which of the following is true?

A. Low IQ

B. Karyotype 46 XO

C. +Presents with primary amenorrhea

58. All of the following are routine investigations for infertility except:

A. Progesterone at 21 days in a 28-day cycle

B. Semen analysis

C. Tests of tubal patency

D. Day 2 FSH, LH, testosterone, FBS

E. +Day 21 FSH and LH

59. Polycystic ovary syndrome patients may have all of the following, except:

A. Hirsutism

B. +Low body mass index

C. High prolactin level

D. Subfertility

E. Oligomenorrhea

60. Regarding infertility, all are true, except:

A. +Secondary infertility is defined as only failure of pregnancy after at least one term pregnancy
50
B. Primary infertility is define as failure of pregnancy with no previous pregnancies

C. Male factor infertility accounts for 40% of causes

D. Infertility is defined as failure to conceive after 12 months of regular unprotected intercourse


(not same words but it was the correct definition)

61. All of the following are mandatory in a case of urinary incontinence, except:

A. Pelvic ultrasound

B. Bladder diary

C. Urodynamic studies

D. Careful history

E. Urinalysis

62. A patient with imperforate hymen, what’s the typical presentation?

A. +Cyclical lower abdominal pain with amenorrhea.

B. Short stature

63. Regarding the treatment of anemia in pregnancy, all of the following are true, except:

A. 60 mg of iron is adequate dose in pregnancy

B. +Parenteral iron is used to correct anemia faster

C. 0.4 mg folate is required for all pregnant ladies as a minimum

D. Gastrointestinal side effects are the main cause of noncompliance to therapy

E. With optimal treatment, average Hb rise is 0.8 g/dl per week

64. Regarding breastfeeding, all are true, except:

51
A. Colostrum contains antibodies

B. Suckling stimulates oxytocin release

C. +Breastfeeding slows down uterine involution

D. Associated with less allergies in the child

E. Exclusive breastfeeding is essential to have amenorrhea for 6 months

65. Regarding oxytocin, all are true, except:

A. Can cause hypotension

B. Is a nanopeptide

C. +Synthesized in the posterior lobe of pituitary

D. Oxytocin receptors in the uterus are upregulated by the end of pregnancy

E. Has some antidiuretic activity

66. Regarding anemia in pregnancy, all are true except:

A. Folate deficiency is the second most common

B. +An MCV of 70 fL is diagnostic of iron deficiency anemia

67. What is the most common cause of coagulopathy during pregnancy?

A. Dilutional coagulopathy

B. +Placental abruption

C. vonWillebrand disease

D. Sepsis

E. Amniotic fluid embolism

52
68. All of the following used in the diagnosis of molar pregnancy except:

A. Pelvic ultrasound

B. Serial beta hCG

C. Histologic examination

D. +Abdominal/Pelvic CT

69. All of the following are true regarding hydatidifrom moles except:

A. Complete mole has higher malignant potential than partial mole

B. Complete mole is 46 XX or XY

C. Partial mole is triploidy 69 XXY

D. May present with early PET

E. +70% of women with complete mole have hyperemesis gravidarum

70. Regarding gestational trophoblastic disease, all are true, except:

A. Unknown etiology
B. Diet may play a role
C. Results from defective fertilization
D. Most common complete mole is 46XX, partial 69 XXY
E. +Partial moles are more common

71. Which of the following is an epithelial ovarian tumor?

A. Fibroma

B. Endodermal sinus tumor

C. +Brenner

72. Regarding cervical ectropion, all are true except:

53
A. May present as postcoital bleeding

B. +May present in postmenopausal females

C. Cervical smear is mandatory before treatment

73. Regarding epithelial ovarian tumors, all are true, except:

A. The most important prognostic factor is remaining malignant tissue after surgery

B. +Hormonal presentation is common

C. Borderline types are treated with surgery with no need for chemotherapy

74. Regarding postmenopausal bleeding, all are true except:

A. It is always abnormal and requires investigation

B. Endometrial hyperplasia is seen in 5% of cases

C. +Endometrial cancer is seen in 40% of cases

D. Atrophic vaginitis, endometritis are the most common causes

75. Regarding endometrial cancer, which of the following is true?

A. Same incidence and prognosis worldwide

B. +Most cases present in early stages

76. All of the following are risk factors for endometrial cancer except:

A. Late menopause

B. Nulliparity

C. +Combined oral contraceptive pills

D. Granulosa cell tumor

54
77. Regarding cervical cancer, all of the following are true except:

A. Abnormal vaginal bleeding is the most common presentation

B. False negative Pap smear is found in up to 50% of patients with invasive cancer

C. +Radiotherapy is superior to surgery in early stage disease

D. Squamous cell carcinoma is the most common type

78. A 30 year old with DUB, all are possible treatments except:

A. Mefanemic acid

B. Mirena

C. Combined OCP

D. +Endometrial resection and ablation

79. Regarding menorrhagia, all are true, except:

A. Mostly dysfunctional

B. Commonly caused by adenomyosis in nulliparous women

C. Levonorgesterol-releasing IUD is an effective treatment in all age groups

80. All of the following are side effects of progesterone-only pills, except:

A. Acne

B. Weight gain

C. Breast tenderness

D. +Complex ovarian cysts

E. Irregular bleeding

55
81. All of the following contraceptive methods are matched correctly to their failure rates,
except:

A. COCP --- 1%

B. Depo-provera ---- 1%

C. +Implanon --- 1%

D. Progesterone only --- 2%

82. Regarding UTI in pregnancy, all are true except:

A. E.coli is the most common cause

B. Asymptomatic bactirurea should be treated to decrease risk of pyelonephritis

C. +Pyelonephritis is more common on the left side

D. Pyelonephritis needs 2-3 weeks of antibiotics

E. A repeat culture is needed to ensure successful treatment

83. Regarding sterilization surgeries, all are true except:

A. In Jordan, husband’s consent is essential for female sterilization

B. +It’s absolutely contraindicated to do surgery to a lady <28 years

C. Vasectomy won’t lead to sterility in the first week after surgery

84. All of the following are essential to screen for in ANC except

A. Rubella titer

B. +TORCH infections

C. HBV

D. FBS

56
85. Regarding preinvasive cervical lesions, all are true, except:

A. There’s a 30% risk of cancer for high grade CIN after 10 years if left untreated

B. Most low-grade lesions regress spontaneously within a year

C. Always arise from the transformation zone

D. +A pap smear from the ectocervix only is necessary

86. Regarding progesterone-only pills in the treatment of DUB, all are true except:

A. May cause weight gain

B. The most widely used hormonal treatment

C. +More effective in ovulatory rather than anovulatory cycles

D. Can be used in the second half of the cycle

E. Reduce menstrual flow by 25%

87. Regarding ectopic pregnancy, only one of the following is true

A. Most cases are silent

B. Positive pregnancy test and empty uterine cavity on ultrasound are diagnostic

C. Heavy vaginal bleeding

D. +Patient may have an intrauterine contraceptive device

88. Regarding ectopic pregnancy, all are true except

A. Negative pregnancy test excludes the diagnosis

B. +All types of IUD have similar risk of ectopic pregnancy

57
89. Regarding ovarian tumors, all are true, except:

A. +Population screening is beneficial in prevention of ovarian CA

B. Most patients present in advanced stages

C. Most patients have vague, non-specific symptoms

90. All of the following is true regarding endometriosis, except:

A. +Causes intermenstrual bleeding

91. All of the following are symptoms of overactive bladder, except:

A. +Passage of urine with sneezing

92. All of the following make you suspect premature rupture of membrane, except:

A. Positive nitrazine test

B. Positive fern test

C. +Uterine contractions on CTG

D. Visualizing the amniotic fluid …

93. All of the following are contraindications for tocolytics, except:

A. +Preterm breech presentation


B. Chorioamnionitis
C. Uterine atony
D. Severe PET
E. IUGR

94. All the following regarding Estimated date is true except

A. Measure CRL at 12 weeks is most accurate

B. OCP use is important

58
C. Estimates upto 38 weeks GA

D. Estimated from first day of LMP

95. All are true regarding Endometrial CA except

a) Stage 3 is confined to cervix

96. Tumor secretes AFP

a) Endodermal Sinus tumor

97. All are true regarding Ovarian tumors except

a) Epithelial tumors are characterized by hormone secretion

98. Side effect of B sympathomimetics except

A) Tremor

B) Premature closure of Ductus

99) True regarding secondary dysmenorrhea except

a) Caused by endometriosis

b) Ovulatory pain (something like that)

100) woman presented with 2 contractions/10 mins, cervix 4 cm …

a) Not in labor

101) Woman presents 12 days after delivery with hemorrhage one is correct

a) Its primary PPH

b) Start Abs immediately (might be this)

102) Regarding puerperium all true except


59
a) peripheral vascular resistance increases right after delivery

b) vagina may never go back to original structure

c) 6 weeks postpartum

103) Regarding Ovulation which is correct

A- Preceded by LH surge

3. 6th Year Final Exam 2013 :


1. All are indications of Anti D except :
Abdominal trauma
Fetal death
Oligohydramnios
Amniocentesis

2. What layer of ovum prevents the penetration of acrosomal enzyme of sperm of other
species.
Zona pellucida

3. Which one is wrong about hyperemesis gravidarum


Tripleoidy
Management by giving fluids
Can lead to wernicke’s encephalopathy
Begin in the end of first trimester

4. All can occur in infant of diabetic mother except


Caudal regression syndrome
Cardiac defects
Renal defects
60
Hypocalcemia
Hypoplasia of islets cells of pancreas

5. FSH affect at which cells?


Ovarian theca
Ovarian granulosa

6. All are contraindications for IOL except


Contracted pelvis
Transverse lie
Previous 2 CS
Vaginal prolapsed

7. All are routine investigations in ANC except


Hepatitis screen
Rubella titer
Toxoplasma IgM

8. All are increased in AFP except


Encephalocele
Trisomy 21
Turner’s
Trisomy 18

9. All are increased in pregnancy except


Total thyroxin
Fasting blood sugar
Cortisol
free t3 and T4 increase….. the option was some thing like that

10. All are true about cardiac diseases in pregnancy except


Pulmonary hypertension indicates bad outcomes "Not SURE"
Commonest congenital heart disease is cushion defects "NOT SURE"
Pulmonary artery pressure monitoring decrease Eisenmenger’s

61
Rheumatic disease’s incidence is decreasing
Eisenmenger’s brings bad prognosis

11. All are contraindications for IUCD insertion except


Uterine anomalies
PID
Emergency contraception
Irregular periods

12. All are tocolytics drugs except


Ritodrine
Salbutamol
Diazepam
Indomethacin
CCB

13. All are true about adenomyosis except


Round globular uterine enlargement
Located in myometrium
Common in multiparous women
Diagnosed by endometrial biopsy

14. All are acute complications of radiotherapy except


Enteritis
Vesico-vaginal fistula
Bone marrow suppression
Proctosigmoiditis
Acute cystitis

15. All can cause genital prolapsed except


Forceps delivery
Prolonged labor
Hysterectomy
Caesarean section

62
16. All indicate severity of PET except
Oliguria
Severe frontal headache
Alkaline phosphatase
5g/24 hours proteinuria

17. All are causes of cord prolapsed except


Anencephaly
Post-maturity

Note : causes of cord prolapsed in general :


- Premature delivery of the baby
- Delivering more than one baby per pregnancy (twins, triplets, etc.)
- Excessive amniotic fluid
- Breech delivery (the baby comes through the birth canal feet first)
- An umbilical cord that is longer than usual)

18. 30 yr old woman with CIN III by pap smear and currette I think. Next step:
LEETZ
Hysterectomy
repeat in 6 months

19. all of this not tocoletic expt


-diazpam
-endomethcin
-sulbemetml

20. all of this c/i for s/c ex


-vaginal prolapse
-contracted pelvic
-transver

63
21. about of turner syndrom all this true ex
-cystic hymngma
-45 xo

22. About PCOS all are true except:


Progesterone test is -ve
Common in reproductive age
most women have insulin resistance
weight loss is a good treatment

23. All are needed to perform forceps delivery except:


fetus head palbable 3/5 in abdomen
ROM
fully dilated cervix
adequate pelvis

24. All are specified by 1000 except:


Maternal mortality
fetal mortality
fetal morbidity
fetomaternal mortality

25. Most common malignant germ cell tumor in ovaries:


dysgerminoma
other stuff

26. The single most common cause of fetal death:


Asphyxia
IUGR
stuff
There was either two options:
1. Prematurity
2. infections
but it was two different questions,

64
27. All can be prevented by good ANC except:
Placenta previa
macrosomia
Infection

28. All cause nephrotoxicity and bladder toxicity except:


bleomycin…it causes lung fibrosis.
cisplatin
cyclophosphomide
ishy-mide
carbo-ishy

29. About HRT all are true except:


c/i in breast ca
c/i in DVT
first sign that its working is increase in appetite "Not Sure"
causes endometrial hyperplasia "Not Sure"

30. About CS which is wrong:


Always requires general anasthesia
vertical and lower segmental are called classical
vertical is more painful than lower segmental

31. about local anasthesia which is wrong:


Epidural causes headache
spinal causes hypertension
local infection is a c/i
meningitis is a complication

32. Thyroid dx which is wrong:


Most common pre-exisisting endocrine disorder of pregnancy
fetus need mother thyroxine for brain development
maternal total and free t4 is elevated

65
33. About contraceptive surgery (msh hek kan bs b m3na) all are true except:
tubal ligation is absolutely c/i in a woman below 28 yrs
in jordan a husbands consent is needed to do a tubal ligation
vasectomy is ineffective in first 2 weeks
they are reversible
34. About physiological anemia in pregnancy which is wrong:
diagnosed by MCV < 85

35. A woman comes in 34 weeks pregnant diagnosed with breech, she's stable, whats your
next step:
Come again in 2 weeks
ECV
C/S
IOL

36. A woman who is 34 k2no weeks pregnant comes with mild vaginal bleeding and stable
vitals, on U/S she has placental abruption, what's your next step:
deliver C/S
tocolytics "Not Sure"
fluid support "Not Sure"

37. A woman with 8 weeks ammenoria comes with vaginal bleeding and passage of clots and
lower abdominal cramps, on U'S there was a 12 mm empty sac, the -ddx can be all of the
following except:
- threatened abortion
- Complete abortion
- missed abortion
- molar pregnancy

38. Which is wrong about bacterial vaginosis:


- most common bacterial infection
- foul fishy discharge
- mostly asymptomatic

66
- usually theres a local inflammatory process
- best treatment is metrinidazole

39. The layer of ovum that prevents fertilization by other species is:
- Zona pellucida
- theca interna
40. During antenatal care which is not a routine investigation
- toxoplasmosis
- platelets
- rubella

41. In Endometriosis all are correct except:


- CA 125 can be used for SCREENING (I think this is the answer)

42. About turner syndrome all are true except:


- associated coarctation of the aorta
- increased incidence with increased maternal age
- subnormal intelligence
- female phenotype perhaps m not sure if i mixed it with another question
- May have cytic hygroma

43. Effect of renal disease during pregnancy all is true except:


- azathioprine is not teratogenic
- can affect pregnancy outcome ? sthg like that

44. A case of suspected ectopic pregnancy (b-hcg 10000 and no intrauterine sac), patient is
stable no bleeding. What to do?
- laparascopy
- serial b -hcg

45. Regarding recurrent miscarriage which one is wrong:


- the number of previous miscarriages affect the probability of pregnancy

67
46. All characteristics of bacterial vaginosis except:
- Local inflammatory reactions
- fishy odor

47. All are ACUTE complication of pelvic irradiation except:


- enteritis
-proctitis
- uretrovaginal fistula

48. Inhibin is secreted from which ovarian tumor?


- Granulosa cell tumor

49. Androgen insensitivity syndrome which one is wrong:


- karyotype is XY
- Female phenotype
- Normal uterus
-normal testosterone level

50. All of the following are true about hyperemesis gravidarum except:
- mainstay of treatment is hydration
- in severe cases associated with Wernicke's encephalopathy
- more during 2nd and 3rd trimester

51. Which is wrong about recurrent miscarriage:


cervical insufficiency happens in 1st trimester
LMW heparin and aspirin might help
it happens in 1%
Mostly unknown etiology

52. Regarding multiple pregnancies, all the following statements are correct except:
- increase risk of placenta previa
- High order multiples are mainly due to assisted reproductive technologies
- Disseminated intravascular coagulopathy is common in case of single fetal demise

68
53 . A32-year-old woman, gravida 2,para2, comes to the physician for follow-up of an
abnormal Pap test. One month ago, her Pap test showed a high-grade squamous intraepithelial
lesion (HGSIL). Colposcopy demonstrated acetowhite epithelium at 2 o'clock. A biopsy taken of
this area demonstrated HGSIL. Endocervical curettage (ECC) was negative. The patient has no
other medical problems, has never had cervical dysplasia, and takes no medications. Which of
the following is the most appropriate next step in management?
a. Repeat Pap test in I year
b. Repeat Pap test in 6 months
c. Repeat colposcopy in 6 months
d. Loop electrode excision procedure (LEEP)*
e. Hysterectomy

54. Regarding cervical intraepithelial neoplasia (CIN), All the following are correct except
a. High Risk HPV fpes (16, 18, 31,33,or 35), are associated with high-grade cervical lesions
(CIN2, 3) and cervical
cancer
b. Low risk HPV type (6 and i 1), are associated with low grade cervical lesions (CIN1,
Ccndylomata acuminata)
c. At least 70 %f patients with CIN3 will develop invasive cancer within l0 years
d. Low grade lesions CIN1 may spontaneously regress
e. Cone biopsy is adequate treatment for high grade cervical lesions CIN3

55. Regarding cervical erosion (ectropion), All the following statements are correct except
a. It is due to eversion of the columnar epithelium
b. Best treatment method is cryotherapy
c. It is an ulcer
d. Treated conservatively during pregnancy
e. It is common in combined pill users

56. Regarding warfarin, All the following statements are correct except
a. Is teratogenic
b. Monitored by prothrombin time
c. Crosses the placenta

69
d. Contraindicated during lactation
e. Specific antidote is fresh frozen plasma

57. Complications of post maturity include AII the following except


a. Prolonged labor
b. Oligohydramnios
c. Intrauterine fetal death
d. Respiratory distress syndrome
e. Meconium aspiration syndrome

58. Principles of management of septic abortion include All the following except
a. Immediate evacuation of the uterus
b. lntravenous antibiotics
c. Correction of hypovolaemia
d. Cervical swab for culture
e. Blood grouping and cross matched ) 2 units of blood

59. The following chemotherapeutic agents are associated with adverse effects to the kidney
and urinary tract
except
a. Chlorambucil
b. Cyclophosphamide
c.Ifosfamide
d. Cisplatin
e. Carboplatinum

60. Regarding follicle-stimulating hormone (FSH), only one of the following statements is
correct
a. Is responsible for oestradiol production from the granulosa cells*
b. Brings about follicular rupture
c. Is raised in polycystic ovary syndrome
d. Is necessary for the initial stages of embryo development
e. Is necessary for maintenance of the corpus luteum

70
5. 6th Year Final Exam 2012:
1. organic causes of menorrhagia include all of the following except:
a. PCOS
b. endometrial hyperplasia
c. endometrial cancer
d. copper releasing IUCD
e. fibroid

2.About Contraception failure rates, all true except :


a. POP 2%
b. COC 1%
c. depo.provera 1%
d. implanon 1%
e. mirena 0.5%
Cant recall the % :/:/.. check the lec..

3. About lower uterine segment .. one is true:


a. bladder is posterior
b. covered by loose vesciral peritoneum .
c. round ligament from its upper part
d. crossed by the ureters
e. inferior epigastric artery are lateral to it

4. Monozygotic twins .. all true except :


a. mainly diamniotic .
b. increased risk of C/S
c. more congenital heart disease
d. are 1/3 of all twins

5. About female sterilization .. all are true except :


a. is absolutely contraindicated if <28 yrs old
b. needs a consent from the husband

71
6. regarding C.section, all of the following are true except :
a. midline incision is more painful than the lower segment transverse
b. bladder injury is more common than uretric injury

7. All are fertility sparing treatment for fibroid except:


a. LHRH analogue
b. myomectomy
c. Uterine artery embolisation
d. COC
e. mifepristone (RU 486.

8. female with pap smear the result is LSIL , next step :


a. Colposcopy
b. Referral in 6 months

9. Couple, infertility , tall male with small testes. Semen analysis: oligospermia, next step:
a. Karyotype
b. Fsh level
c. Prolactine

10. About recurrent miscarriages all are true except :


a. screening for toxoplasmosis is recommended
b. screening for lupus anticoagulant is recommended
c. can be associated with PA
d. progesterone is recommended in unexplained cases

11. All of the following can decrease effect of COCP except:


a. Phenytoin
b. Amoxicillin
c. Gastroenteritis
d. Rifampicin
e. Valproic acid

12. GA: 37 wks, asymptomatic placenta previa anterior, 1 cm above cervix edge, the patient is
72
at risk off all except:
a. PPH
b. NICU admission due to RDS
c. Severe neonatal anemia
d. C/S
13. Clue cells are seen in :
a. bacterial vaginosis

14. Case: ovarian tumor bilateral 5*5 cm , CA 125 = 500. Modality of Rx:
a. Chemotherapy then surgery
b. Total hystrectomy with bilateral salpingo oopherectomy with omentectomy.

15. About gestational DM .. all are true except:


a. OGTT is not needed after puerperium
b. macrosomia incidence is 10%
c. a fasting glucose of 7.2 at 24 weeks is diagnostic for DM
d. postprandial glucose level > 8 necessitate start insulin

16. All of the following are relatively contraindicated in pregnancy except:


a. Tetracycline
b. carbimazole
c. Cox.2 inhibitor
d. carbamazepine

17. All are symptoms of fibroid except:


a. mennoraghia
b. IMB
c. abdominal distension
d. pressure symptoms
e. infertility

18. one of the following is suggestive of stress incontinence:


a. leakage of urine on sneezing
73
19. all are worrying sign in late pregnancy except:
a. white vaginal discharge
b. watery vaginal discharge
c. generalized itching

20. PCB in all of the following exept:


a. cervicitis
b. cervical polyp
c. cervical carinoma in situ .

21. ASCUS:
a. 10% progress to invasive disease
22. all are surgery for prolapse except:
a. anterior colporaphy
b. colposusspension
c. colpopexy

23. all can be detected by US in third trimester except:


a. turner
b. fetal demise
c. fetal anemia

24. about ANC ... one is true :


a. booking visit is preferred in 11.14 weeks
b. detailed anomaly scan is done at 22.24 weeks
c. at 36 weeks presentation should be confirmed

25. heparin .. one is false :


a.monitored by aPTT
b.contraindicated in lactation

26. laproscopy is useful in all the following except :


a. endometrial polypectomy
74
b. subtotal hysterectomy
c. salpengectomy

27. about cervical incompetence .. all are true except:


a. cerclage done at 11 weeks
b. can be congenital
c. diagnosed by US

28. First step in shoulder dysticia :


a. Episiotomy
b. McRobert (legs up.
c. suprapubic pressure .

29. all are risk factors for cord prolapse except:


a. ECV
b. prematurity
c. oligohydramnios

30. One is false about ectopic:


a. can be excluded if BHCG test is .ve
b. almost always patients have a period of ammenorhea or irregular bleeding
c. aria stella is characteristic
d. 50% of pts have +ve urine pregnancy test

31. At 12 weeks gestation ... a gestational sac found using abdominal US.. CRL = 7mm ... and no
heart beat found .. your next step :
a. repeat US after 1 week
b. terminate pregnancy (missed abortion.
c. serial B.HCG

32. first sign of uterine Dehiscence is :


a. tachycaria
b. hypotension
75
c. fetal distress
d. abnormal abdominal shape

33. Amniotic fluid embolus .. all true except:


a. managed by heparin and FFP
b. usually with coagulation disorder
c. very serious condition

34. about STD .. all true except :


a. gonorrhea can cause infertility
b. HIV patient must be deliverd vaginaly
c. primary herpes can be associated with lymphadenopathy
d. syphilis in third trimester should be treated with penicillin

35. HRT .. all are true except:


a. increase risk of colon cancer.
b. causes endometrial hyperplasia
c. subdermal patches have less SE
d. local estrogen is more effective for atrophic vaginitis .

36. All are matched except :


a. POP . Wt loss
b. danazol . hoarseness of voice
c. COC . headache
d. mirena – spotting
37. about endometrial CA.. one is true :
a. serous papillary type is the commonest
b. serous papillary has worse prognosis than endometriod

38. Molar .. one is false :


a. grape like vesicles
b. malignant sequele can be avoided
c. 50% chance of persistant GTN .

76
39. about HPV vaccine .. one is true :
a. not effective in males
b. can be used in treatment of HPV 16 ,18 related CIN II, III
c. single dose at 12 years old

40. About Kielland forceps true except:


a. Has cephalic curvature
b. Has pelvic curvature
c. Has an english lock
d. Used for rotation

41. All causing 2nd amenorrhea except:


a. Meyer.rokitansky
b. Ashermans syndrome
c. Pcos

42. All causing 2nd amenorrhea except:


a. Transverse vaginal septum

43. About candidal infection all are true except:


a. vaginal discharge
b. ph less than 5
c. +ve WIFF test

44. postdate complication except :


a. shoulder dystocia
b. hyperbiluribinemia
c. placenta insufficient

45. The definitive treatment for Endometriosis :


a. Hysterectomy
b. Hysterectomy & Bilateral Salpengoopherictomy
c. Myomectomy

77
46. All of these require US to Dx except :
a. Threatened Abortion
b. Missed Abortion
c. Inevitable Abortion

47. Unexplained infertility one is false :


a. diagnostic test is ovarian ultrasound .. can’t remember the rest of the choices :/

48. all are routinely done pre. operatively except :


a. chest X.ray
b.CBC

49. all done in case of urinary incontinence except :


a. urine analysis
b.pelvic U/S
c.KFT
d.urodynamic

50. all routinely done in ANC except :


a. PTT

51. one is associated with fetal acidosis :


a.amniotic fluid index = 5
b.reversed end diastolic flow in umbilical artery doppler
c.HR of 110 .

52. primi gravida fully dilated for 1 hr , u accept these findings except :
a. semi.flexed
b.ROP
c. station 0
d. caput

78
53. about Rh .isoimmunisation one is false :
a. anti D given within 72 hrs ,for Rh positive woman .
54. Endometriosis can be a cause of all the following except :
a. infertility
b.amenorrhea
c.dymenorrhea

55. one is true about breech presentation:


a. it carries higher mortality than cephalic whatever the mode of delivery .
b. post.maturity is a known risk factor .

56. Elevated A feto.prtn in all of the following conditions except:


a. down syndrome

57. True about stored blood:


a. Deficient in 2,3 DPG

58. Not an absolute indication for CS delivery:


a. 2 previous CS.
b. cord prolapse.
c. cord presentation .
d. Second twin mal.presentation .

59. About ovarian cancer,one of the folowing is wrong:


a. Dysgerminoma is more common in older age/menopouse woman

60. About primary amenorrehea, what is wrong:


a. Brain MRI must be done
b. Pelvic u/s is essential
c. karyotyping is a must if FSH and LH are low .
d. TSH can be done

61. Not detected in first trimester by U/S :


a. microcaphaly

79
62. Not true in fibroid :
a. 2 % will transform into sarcoma

63. a 22 years old primigraveda GA 30 weeks her BP 150/90 one of the following is true :
a.serum uric acid is elevated
b. you have to repeat BP after 6 hour rest
c. this is pregnancy induced hypertension

64. In DIC all increase except :


a. fibronogin .
b. Aptt .
c. PT .

65. A 40 years old female has a heavy menses what is the best contraceptive method :
a. mirena.
b.OCP
c.progesteron minipills.

66. All of the following are vaginal procedures except :


a. tubal ligation .

67. Something about amenorrhea all of the following is true except :


a. Rokitansky syndrome karyotype is 46 XY .

68. about endometrial CA all are true except :


a. 5 year survival rate in early stages is 30 % .

69. about PAP smear all are true except :


a. HSIL is usually caused by HPV 6 , 11 .

70. One of the following is not of the acute management of preeclampsia :


a. haydralazine

80
b. labetolol
c. nifipedine
d. methyldopa .

71. Methotraxate is not used if:


a. sac less than 40 mm
b. fetal heart activity is detected

72. about a baby with meconium stained liquor at the onset of labor, what to do:
a. if the fetal heart rate was active only observe
b. deliver immediately by CS
c. it is an indication for fetal heart monitoring

73. All of the followings can be used in the treatment of stress incontinence except:
a. Pelvic exercise
b. Colposuspension
c. TVT
d. Toilet training .

‫ فتشمل العبادة‬، ‫إن "جـــمـــيــع" أعمال اإلنسان النافعة تكون له بالنية عبادة‬
‫ وكسبه‬، ‫ وقيامه وقعوده‬، ‫ ويكون المرء متعبداً في طعامه وشرابه‬، ‫الحياة كلها‬
‫ ومن هنا يكون الفهم الصحيح‬، ‫وزواجه‬
: ‫لقوله تعالى‬
َّ َّ ِ‫ت ْالج‬
ِْ ‫نو‬
ِ ‫َاإلنسَ إَِّل لِي َْع ُب ُد‬
" ‫ون‬ ُ ‫خلَ ْق‬
َ ‫" َومَا‬
. ‫ هي غاية الخلق‬، ‫فتكون العبادة بهذا المعنى الشامل‬

. ‫ علي الطنطاوي‬. " ‫"تعريف عام بدين اإلسالم‬

81
6. 5th Year Final Exam 2012 :
1. Which following is oxytocin antagonist:
a. Atosiban
b. Ritodrine

2. All are risk factors for placental previa except:


a. Previous previa
b. Polyhydramnios
c. Previous C/Sec
d. Multiparity

3. All are risk factors for endometriosis except:


a. Sisters have endometriosis too
b. High parity
c. Intfertility

4. Contraindication of ritodrine:
a. Hyperthyroidism
b. Uterine anomalies

5. Which is true about Turner’s Syndrome:


a. 45 XY is the characteristic
b. Associated coartication of aorta
c. Secondary amenorrhea

6. Which is wrong about Granulosa Cells Ovarian Tumor:


a. Bilateral in 50%
b. Malignant in 25%
c. Produce estrogens
d. Associated with endometrial cancer
e. Have Call.Exner bodies

82
7. Commonest complication to fetal on diabetic mother:
a. Spina bifida
b. Anal atresia
c.VSD
d. Renal agenesis
e. Sacral agenesis

8. Spinbarkeit is:
a. Threading of mucus
b. Thickening of mucus
c. Thinning of mucus
d. Crystallization of mucus

9. What is the treatment for Postpartum Blues:


a. Reassurance
b. SSRI
c. Psychological consultation
d. ECT

10. A women with G2 P0 +1 is considered as:


a. Have a stillbirth
b. Have 2 miscarriages
c. Pregnant right now
d. Previous preterm birth

11. Which has the strongest association with recurrent preterm birth:
a. Smoking
b. Previous preterm
c. PET

12. All following is contraindicated in pregnancy except:


a. Ciprofloxacin
b. Erythromycin
c. Doxycycline
d. Vitamin A
83
e. Methrotrexate

13. Most common symptom of Ectopic pregnancy :


a. Abdominal Pain
b. Vaginal bleeding
c. Amenorrhea

14. Which is not a sexual transmitted disease:


a. Toxoplasmosis
b. Chancroid
c. Chlamydia

15. All are uses of clomiphene citrate except:


a. Premature ovarian failure
b. Polycystic ovarian syndrome
c. IVF

16. All are investigations done for hydatidiform mole except:


a. Chest Xray
b. Coagulation profile
c. Pelvic/abdominal CT scan
d. bHCG

17. All are absolute contraindication of COCP except:


a. Liver disease
b. Breast cancer
c. Previous DVT
d. Taking pills in the previous pregnancy

18. All are true about minipills except:


a. Cause functional ovarian cyst
b. Have same effects as COCP in ovulation inhibition

19. The most likely cause of acute abdominal pain for patient who had just have IUCD insertion

84
is:
a. Copper allergy
b. Uterine contraction
c. Uterine rupture

20. Which is wrong about hormone replacement therapy:


a. Give for short duration
b. Only orally
c. Lowest effective dose is recommended
d. Increase breast cancer risk

21. Which investigation done for an infertile couple:


a. Ovarian biopsy
b. Semen analysis
c. Laparoscopy
d. Antisperm antibodies
22. A male patient came to the clinic with asthenozoospermia and teratozoospermia while her
wife is normal. What kind of treatment you’ll choose for them?
a. GIFT
b. ICSI
c. IVF

23. Which is wrong about human milk:


a. Lesser protein than cow’s milk
b. Lesser calories than cow’s milk
c. More lactose than cow’s milk
d. Rich in Vitamin B

24. All are absolute contraindications for induction of Labor except:


a. PET
b. transverse lie
c. Abnormal CTG
d. Previous 2 C/sec
e. Previous pelvic floor repair

85
25. Which is true about stages of labor:
a. Descent, Engagement & Flexion

26. All are initial investigations in ANC except


a. Blood glucose
b. Hematocrit
c. Liver transaminases
d. Antibody screening

27. About thyroid disease in pregnancy, which is false:


a. Hashimoto’s thyroiditis is the commonest cause of hypothyroidism
b. High total thyroxin
c. Low thyroid binding protein
d. Beta blockers are used for initial treatment in hyperthyroidism

28. All are causes of meconium in utero except:


a. Bowel presitalsis
b. Hypoxia
c. Intrauterine cystic fibrosis

29. Which following patient is given Anti D:


a. Rh.ve female with Rh+ve male
b. Rh.ve female who had amniocentesis
c. Rh.ve female with threatened miscarriage at 6 weeks

30. Which of following can cause preterm birth?


a. Gardnarella vaginalis
b. Lactobacillus …
c. Trichomonas

31. All are true about lower uterine segment except:


a. Develop 28.32 weeks of pregnancy
b. Poor contractile
c. Less vascular than upper segment
86
d. 1cm before pregnancy
e. Stretch during labor

32. Which is an inappropriate match:


a. Station 1 cm above ischial spine
b. Postiton occipito posterior
c. Engagement 3/5th palpable
d. Lie Oblique
e. Presentation Shoulder

33. Which has the largest diameter:


a. Brow
b. Occipito.posterior
c. Occipito.transverse
d. Occipito.anterior

34.Causes of heart failure include all except:


a. coticosteroids
b. Underweight
c. Tocolytics .

35.About follicular phase of menstural cycle which one is true:


a. Decrease aromatization
b. decrease estrogen
c. progesteron predominance
d. Fixed 8 days period
e. endometrial proliferation

36. About testicular feminization, all correct except:


a. male phenotype
b. due to androgen insensitivity

87
37. about breast milk the options were:
a. has less proteins than cow's milk
b. has less calories than cow's milk
c. has more lactose than cow's milk

38. about lower uterine segment, the wrong answer among these is:
a. develops between 28.32 weeks
b. has less blood supply
c. less muscular

39. normal physiology during pregnancy:


a. splitting .
b. loud S1 and S2 .
c. 95% have diastolic murmurs that disappear after delivery
d. S2 has no constant pattern or something like that

40. Regarding fontanels:


a. ant closed in the firrst 6 month
b. vertix most common presantation
c. vertix btw ant and pos fontanelle and paraital bone
d. moulding is normal finding

41. about preeclamsia:


a. seizure can be presented after delivery for the first time
b. inta.vascular pressure increase
c. HELLP syndrome can be presented without hypertension
d. chronic hypertensive pt may not need medication in mid tri .

88
42. lady that 10 weeks GA... discovered to have mass 6*6 wt is ur action
a. reassurence
b. reevaluate at 14.15 weeks
c. immediatly surgery

43. most common presantation for endometrial CA:


a. post menopausal bleeding

44.Regarding Epidural Anesthesia in labor, all are wrong except :


a. less effective in pain relief than other methods. (not the answer)

45. All the following cause stress incontenince except:


a. Pregnancy and childbirth
b. COPD and obesity
c. smoking
d. connective tissue diseases

46. DUB treatment , all except :


a. COP .
b. progestrin only pill
c. Mirena
d.GnRH analog

47. what is not a hematological change in pregnancy:


a. decrease blood count.
b. decrease ESR .

ّ ‫أن ليس لإلنسان‬


"‫إَّل ما سعى‬ ْ ‫"و‬

89
7. 6th Year Final Exam 2011:
1. True about ectopic:
a. Better prognosis if implanted in the distal portion of the fallopian tube.

2. True about GDM:


a. Maternal Insulin does not cross the placenta.

3. Not associated with Diabetes :


a. Associated with folic acid deficiency.

4. When applying forceps:


a. It is important to know position of head (or something like that..

5. 21Y/O Amenorrhea, negative PCT, what is the next best hormone to diagnose her:
a. FSH.

6. Correct parameter:
a. Biparietal=9.5, other form had a different answer.

7. Face presentation is:


a. Submentobregmatic.

8. About maternal Bony pelvis:


a. it is long but if I remember correctly it was something about ischial tuberosity.

9. 9 weeks amenorrhea, by Abdominal US shows 20 mm gestational sac: what's your


management : a. Serial B HCG...important to realize here that US is abdominal not vaginal.

10. Treatment of Ovarian CA:


a. 6 cycles of carbo+pacitaxel.

11. Ovarian Cancer secreting hormone:


90
a. Choriocarcinoma, other form I think it was dysgerminoma.

12. Associated with decrease risk of PET:


a. Smoking.

13. Regarding HF mole, which is wrong?


a. OCP is safe and effective during post treatment management.
b. prominent theca lutein ovarian cyst > 6 cm should be surgically managed.

14. All of the following are true about lactation except:


a. can be continued until 4 years of age...
b. Breast milk is poor in protein compared to cows'
c. Placental lactogen hormone maintains milk production.
d. prolactin stimulates lactation.

15. Which is wrong about bacterial vaginosis ?


a. Caused by lactobacillus
b. Cause 2nd trimester abortion .

16. Lactational mother comes after 7 weeks of amenorrhea, complaining of abdominal pain,
wt's the most imp invx:
a. U/S
b. B.HCG (to exclude ectopic.
c.urine analysis and culture
d.endocervical swab

17. Contraindicated drug in Pregnancy:


a. ampicillin
b. cephalosporin
c. Zidovudine
d. tetracyclin

18.What's the risk of complex atypical endometrial hyperplasia to progress into endometrial Ca
:

91
a.1%
b.<5%
c.<10 &
d. >25 %

19. Not a risk factor for genital prolapse:


a. Nullparity .

20. We don't use clomiphene citrate in :


a. Kallman's syndrome

21. Not a relative contraindication for uterine embolization:


a. intramural fibroid
b. future pregnancy
c. previous internal iliac ligation

22. False about fibroid:


a. Something related to Ca and parathyroid hormone
b. polycythemia

23.Can't be treated with laparoscope :


a. Submucosal fibroid.

24. Can’t be treated vaginally:


a. Ovarian (or adnexial . mass.

25. Not associated with bacterial vaginosis:


a. Inflammed vulva

26. Primi, 38 weeks comes with breech …. ANC is otherwise normal…wt's your next step:
a. ECV
b. positional excersie.

27. 30 y old pregnant comes with blurred vision and +3 proteinuria , BP 150 /100…. your next

92
step :
a. Arrange for delivery .
b. Give hydralazine
c. Give steroid then go for delivery .

28. False about RH isoimmunization :


a. ABO incompatipility increase the risk of RH isoimmunization
b. father has to be RH positive
c. mother should be RH.

29. Multiple pregnancies is not associated with:


a. fetal distress
b. shoulder dystocia
c. fetal anomalies

30. Not indication for anti.D?


a. Preterm labor .
b. Manual removal of placenta .
31. Wrong about cervical cancer:
a. It's always coming from pre.ivasive cervical lesion.

32. Correct about cervical cancer:


a. Most common type is squamous cell ca.

33. Q about HM ……wrong about it:


a. Patient can be pregnant after 3 months.

34. True about Down syndrome caused by translocation:


a. constitutes more than 90% of cases
b. increased in the frequency if the mother has balanced translocation

35. Wrong about recurrent miscarriage:


a. Most cases happen in 2nd trimester

93
36.First thing to do in shoulder dytocia after calling for help is :
a. to flex her knees and hips
b. do episiotomy and do head traction

37. Can't use forceps when:


a. Membrane is rupture and vertex presentation but cervix is not fully dilated

38. Wrong about STD:


a. Secondary herpes comes with multiple painful vesicles (primary does..

93. Diabetes type one is associated with all the following except:
a. macrosomia .

40. About Endometriosis all are correct, except:


a. tubal factor most common cause of infertility
b. symptoms usually at luteal phase .

ً
٬ ‫ فإذا اهتزت العقيدة ظهر النقص‬٬ ‫اتصاَّل وثي ًقا باإليمان‬ ‫إن األخلق فى أرضنا تتصل‬
‫ وقد أصابنا "اَّلستعمار" العالمي فى صميمنا عندما‬، ‫ونجم اإلثم واضطربت األمة كلها‬
٬ ‫ لقد تبع ذلك انهيار خلقى محزن‬٬‫أوهى اإلسلم واستبعد إيحاءه فى الحياة العامة‬
. ‫وميوعة َّل تستقر فيها على شىء‬
... ‫محمد الغزالي‬

94
8. 5th Year Final Exam 2011:
1. All of the following decrease vaginal bleeding except:

a. Copper IUCD
b. LHRH analogue
c. NSAIDs

2. Which of the following is not sexually transmitted?


a. Bacterial vaginosis
b. Trichomoniasi
c. Chancroid

3. All of the followings concerning PET are correct except:


a. Being Primigravida is most important risk factor
b. Hypotensive agents stops disease progression
c. Most important risk to fetus is prematurity

4. A pimigravida presented to you at 34 week gestation with blurring of vision. Her blood
pressure was 150/100, how would u manage this patient?
a. Arrange for delivery
b. Give IV Hydralazine & reevaluate symptoms
c. Give IV steroids & wait 48hr then deliver her

5. All of the following are complications of pregnancy preceded by DM except?


a. Neural tube defect
b. Fetal death
c. Post maturity
6. All are done in prenatal testing except:

95
a. Urine analysis for protein
b. Rubella
c. Uric acid
7. Which of the following is incorrect regarding DM in pregnancy?
a. There’s an increase in fetal anomalies in gestational diabetes
b. Fetal death can still occur even when DM is well controlled
c. Macrosomia can occur even when glucose levels are well controlled

8. In preterm prelabor rupture of membranes, all are suggestive of chorioamnionitis except:


a. Bloody vaginal discharge

b. Greenish vaginal discharge


c. A small for date uterus

d. Tachycardia

9. Which of the followings is correct about endometrial adenocarcinoma?


a. More common in PCOS
b. Physical exam can be diagnostic

c. In advanced stages, surgery is main stay of treatment

10. All concerning CIN are correct except:


a. D & C is enough in CIN III
b. HPV (6, 11) cause condyloma & CIN I

c. HPV (16, 18) cause CINII, III


d. 35% of CIN III progress to invasive CA in 10 years

96
e. CIN I rarely progress to invasive CA
11. Prostaglandin E1 causes all the following except:
a. Flushing
b. Pyrexia
c. Hypertension

d. Seizure
e. Apnea

12. A 25 year old female found to have a 5 *3 cm ovarian cyst, X ray showed calcification,
which is the most likely diagnosis:
a. Mature cystic teratoma

b. Dysgerminoma
c. Yolk sac tumor

13. Which of the following is normal in semen analysis?


a. 35 million sperm/ ml
b. 2% normal motility
c. Liquification in 2 hours .D. Normal shape 3%

14. Which of the following indicate ovulation if done at mid cycle?


a. LH surge

b. Serum Estriol
c. Progesterone

d. B. HCG
e. FSH
97
15. A 35 year old female presented with history of 6 month duration amenorrhea, all can be a
cause except:
a. Testicular feminization syndrome
b. Pregnancy
c. Lactation
d. Menopause

16. An asymptomatic primi-gravida presented to you at 10 week gestation, urine culture


showed 100000 colonies of E.coli, how to manage this patient?
a. Admit & give IV antibiotic

b. Give oral antibiotic


c. Repeat culture

d. Reassurance

17. All increase in pregnancy except:


a. WBC
b. Heart rate

c. Cardiac output
d. Peripheral vascular resistance

e. RBC mass

18. All are found in fetal distress management, except:


a. put the patient in supine position
b. stop oxytocin injection

98
c. oxygen mask
d. rapid infusion (dehydrated)

19. The most common presentation of endometriosis is:


a. menorrhagia
b. dysmenorhea
c. intermentstrual bleeding

20. The most common presentation of PID is:


a. dysparonea
b. menorrhagia
c. vaginal discharge
d. abdominal pain

21. 35 gestation week and FSH is 31 cm >> all could be the cause, except:
a. oligohydraminous
b. growth restriction
c. rupture of membrane
d. preterm labor

22. All are treatment of genital prolapsed, except:


a. vaginalal hysterectomy
b. anterior wall colporaphy
c. hysteroplexy

23. All are associated with PCOS, Except:


a. obesity
b. endometrial hyperplasia
c. amenorrhea
d. high FSH:LH ratio
24. All to be done normally to a 25y/o married with menorrhagea, EXCEPT:
a. histology study to a biopsy (obtained from endometrium)
b. ultrasound
c. hormone study

99
d. laparoscopy

25. all are symptoms for urinary ** in women, except:


a. straining to void
b. urgency
c. urge incontinence
d. stress incontinence

26. Regarding bacterial vaginosis, all are true, except:


a. spouse shouldn’t be treated
b. metronidazole is the DOC
c. foul fishy smell.
d. None of above.

27. Not a symptom of cervical CA:


a. vaginal discharge
b. abnormal vaginal bleeding
c. weight loss
d. Pelvic pain

28. All are complications of twin pregnancy, Except:


a. DM
b. preterm labor
c. postpartum bleeding
d. anemia

29. About postpartum hemorrhage one is incorrect:


a. atony : placenta previa
b. atony: twin pregnancy
c. atony: breech presentation
d. secondary hemorrhage: retained particles

30. Treatment of postpartum blues is:


100
a. reassurance
b. psychological counseling
c. drug (something)
d. ECT

31. About ectopic pregnancy only one is true


a. most of the time discovered incidentally during ANC
b. negative urine pregnancy test rule out ectopic pregnancy .
c. show with pelvic mass

32. All cause post partum pyrexia, except:


a. DVT
b. chorioamnionitis
c. mastitis
d. sepsis

33. All of the following is a risk for PET, except :


a. smoking .
b. previous PET
c. genetic
d. obesity

34. one of the following can be done for Rh testing:


a. indirect coomb's from cord
b. fetal bilirubin
c. maternal bilirubin
d. father rhesus antigens

35. PID causes all of the following, except:


a. infertility
b. ectopic pregnancy
c. cervical stenosis

36. ovulation occurs at:


101
a. day 14 of menstrual cycle
b. 14 days before menses
c. 7 days after menses
d. 21 days after menses

37. All of the following increase risk of folate anemia, except :


a. anticonvoulsant drugs
b. hemolytic anemia
c. trait thalasemia
d. spherocytosis

38. About iron def anemia all true except :


a. blood transfusion may be needed during last trimister
b. iron supplement 10mg per day is not enough .
c. None of above .

39. All true about cardiovascular changes during pregnancy, except:


a. stroke volume increase
b. volume increase 20% in singleton pregnancy .

40. all raise heart rate of fetus, except:


a. mother taking pethidine .
b. mother fever
c. chorioaminitis

41. Most common abnormal presentation is:


a. complete breech
b. frank breech .
c. brow presentation

42. All are absolute indication for C section, except:


a. previous uterine scar in midline incision (or something like that)
b. previous scar in uterine body
c. transverse presentation during labor
102
d. prolapsed cord with cervical dilatation 7cm and a live fetus

43. All of these causes growth restriction, Except:


a. age
b. race .
c. diabetes
d. dieting during pregnancy

44. All are used for diagnosis of fibroid, except :


a. laparoscopy
b. hysteroscopy
c. hormonal level .

45. All are used for treatment of endometriosis, except :


a. GNRH analouge
b. hysterectomy
c. danazol
d. NSAID
All can be given but : danazol not given anymore bcz of its side effects ,hysterectomy is done
with ooppherctomy (radical surgery),continous progestrone is giving with estrogen

46. All are STDs, except :


a. toxoplasmosis
b. trachomanlis vaginalis
c. genital warts .
47. All are risk for endometrial Ca, except :
a. Obesity
b. Lynch syndrome
c. Smoking

48. All are risks for dizygotic twins, except :


a. Elderly
b. GnRH
c. HMG

103
49. Only one true about ectopic :
a. May present as vaginal bleeding
b. Amenorrhea essential for diagnosis

50. All can reduce menstrual bleeding, except :


a. Low dose aspirin
b. COCP
c. GnRH

51. All are diagnostic for ovulation, except :


a. level of E2 in luteal phase
b. endometrial biopsy before end of cycle

52. All disrupt bonding to the mother, except :


a. using forceps in delivery
b. Low socioeconomic
c. NICU

53. All cause abdominal pain in 20 wks gestation, except :


a. Ectopic pregnancy
b. Spontaneous miscarriage
c. Appendicitis

54. Only one is true about GDM:


a. GMD is risk for getting DM later on
b. Insulin used in almost all cases

55. About hot flushes , all are true except :


a. HRT used in mild case
b. Most resolve 1.5 yrs
c. Due to defect in thermoregulation at the level of hypothalamus

104
56. Most common ovarian tumor in pregnancy :
a. Corpus luteum cyst
b.Teratoma
c. Serous cystadenoma

57. About ovarian tumor , all are true, except:


a. Dysgerminoma associated with increase in alpha fetoprotein .
b. CA 125 marker for serous type

58. Only one is true about stages of labor:


a. second stage is shorter in multiparus than in primigravida woman.

59. Not a contraindication for OCP uses:


a. 35 and smoker
B. PB 140.160/90.120 (or something like that)

60. in candidal infection, all true EXEPT:


a. metronidazole is the treatment of choice

61. About abortion all are true, except :


a. U/S essential for incomplete abortion

62. About delivery all true, EXCEPT:


a. fetus head flexion increase during delivery

63. Ideal or best treatment of eclampsia :


a. termination
b. bed rest & lying on the side .

ّ ‫"أجمع العارفون باهلل أ‬


‫ن ذنوب الخلوات هي أصل اَّلنتكاسات‬
ّ ‫وأ‬
‫ن عبادات الخفاء هي أعظم أسباب الثبات" – ابن القيّم‬

105
9. 6th Year Final Exam 2010:
1. One of the following is NOT found in a post.term newborn:
b. Hypoglycemia

2. One of the following is NOT correct about active management of labor :


a. Epistiotomy is routinely done

3. One of the following is NOT correct about multiple pregnancy :


a. Clomiphine citrate increases the risk of monozygotic twins

4. The most important cause of fetal risk in multiple pregnancy is:


a. Prematurity

5. One of the following is an indication for CS:


a. Tranverse lie in the 1st twin

6. One of the following is NOT correct about warfarin:

a. It's contraindicated in lactation.

b. It causes structural abnormalities in the fetus

c. Monitered by INR

7. True about ectopic pregnancy:


a. it can present without missed period.

106
8. The stage of embryology where the embryo is a solid mass of 16 cells is called :
a. Morula

9. One of the following statements about FSH is correct:


a. It works on granulosa cells to secrete estradiol

10. The most common cause of uterine rupture in labor is:


a. Previous CS

b. Congenital uterine abnormalities

c. Obstructed labor

d. Induced labor

11. One of following statements is NOT correct about fetal distress in labor:

a. CS immediately performed

b. Detected by CTG

c. Confirmed by fetal scalp blood sampling

12. One of the following is NOT correct about intrahepatic cholestasis of pregnancy:

a. Intense itching

b. Preterm labor

c. Intrauterine fetal death

d. Characterestic skin rash

13. One of the following is NOT true about pregnant patients on anti.epileptics:

107
a. it's not contraindicated in lactation

b. Patients should take folic acid supplements

c. The probability of congenital abnormalities in the fetus is 3%

d. it's more risky not to take the medications to the mother and fetus than to take them.

14. All of the following increase the risk of thromboembolism in pregnancy except:

a. Thrombocytopenia.

15. A patient who's 7 weeks gestation presented with abdominal pain and mild vaginal
bleeding. US shows a 30 mm empty gestational sac. The dx is:

a. Missed abortion

b. Threatened abortion

c. Complete abortion

16. US is mandatory in the dx of all of the following except:

a. Inevitable abortion

b. Threatened abortion

c. Missed abortion

17. One of the following is wrong about HRT in menopause:

a. Tibilone is an estrogenic HRT

b. Breast tenderness is one of the start up symptoms

c. Abnormal vaginal bleeding is a contraindication

108
d. Uterine fibroid is a relative contraindication

18. The age of menopause is:

a. Genetically determined

b. Affected by OCP use

c. Smoking has no effect

d. Affected by the age of last pregnancy

19. One of the following is TRUE about Assisted Reproductive Techniques ART:

a. Clomiphine citrate has an adverse effect on cervical mucous

b. Multiple HCG shots are given to induce ovulation

20. One of the following is NOT true about IVF:

a. The optimal number of ova transfered into the uterus is 3.4 .

21. In sperm preparation, capacitation is:

a. When the sperm acquires acrosomal enzymes

b. When the sperm is able to penetrate the zonna pellucida

c. When the sperm is able to fertilize the ovum

22. One of the following is a normal value in semen analysis:

a. A progressive motility of 25%

b. A count of 35 million/ml

c. A normal morghology of 20%

109
23. One of the following is NOT correct about ovarian hyperstimulation:

a. Drainage of the enlarged ovaries is one modality of treatment

b. It's worse in a conception cycle

c. Low albumin level can be found

24. One of the following is true concerning a patient with Sheehan syndrome:

a. HMG is used for ovulation induction

25. One of the following is NOT correct about gynecological surgeries:

a. LFT is routinely done before surgery

26. One of the following is NOT a side effect of Paclitaxil:

a. Diarrhea

b. Arthralgia

c. Neuropathy

27. One of the following is NOT a cause of delayed puberty:

a. Cold climate

b. Hypothyroidism

c. Prolactinoma

28. One of the following is NOT true about ovarian cancer:

110
a. Patients with early stage are discovered at younger age than those with advanced stage .

29. A 21 year old patient underwent a right ovarian cystectomy. The specimen was found to
contain teeth, cartilage, intestinal glands, and hair. The most likely dx is:

a. Thyroid cancer

b. Benign cystic teratoma

30. One of the following is true about genital prolapse:

a. Pelvic floor facsia supports the vagina

31. One of the following is NOT done initially in the investigations of urine incontinence:

a. Post void US

b. Urodynamic studies

c. Clinical assesment

32. One of the following is NOT true about β.HCG:

a. Used for Dx and follow up of benign and malignant molar pregnancy

b. Used as a tumor marker in non.gestational choriocarcinoma

c. it's a good marker for placental site trophoblastic tumor

33. A patient who's 6 weeks amenorrhea presented with abdominal pain and vaginal bleeding.
Her β.HCG level was 3000U/L. Her TVUS shows no gestational sac but no other abnormality.
The best next step is:

a. Repeat β.HCG in 48 hours

111
b. Give systemic methotroxate

c. Laparatomy (there was no laparascopy choice.

34. One of the following is true about ectopinc pregnancy:

a. Accounts for 10% of maternal mortality

b. β.HCG and US findings going with ectopic are diagnostic

35. 60 years old patient presented with postmenopausal bleeding with normal pelvic
examination. The next step is:
a. TVUS

36. One of the following is true about endometrial cancer:


a. Clear cell is worse prognosis than endometroid .

37. One of the following is NOT a cause of menorrahgia:


a. Ectropion

38. One of the following is the most likely to have endometriosis:

a. A 30 years old female with secondary infertility for 6 years and dyspareunia

b. A 15 years old with lower abdominal pain in the first 2 days of period fading with menses

39. A 17 years old complaining of dysmenorrhea was found to have pelvic endometriosis. The
initial treatment:

a. NSAIDS

b. GnRH for 9 months

112
40. One of the following is NOT true about DUB:

a. Mefamenic acid increases PG

41. A 40 year old patient with heavy menses and hypertension. The best contraceptive method
for her is:

a. COCP

b. Mirena

42. A patient was refered because of unfound IUCD thread. The next step:

a. Pelvic US

b. Abdominal X.ray

43. IUCD is contraindicated in all of the following except:

a. Previous CS

b. Previous PID

c. Previous ectopic

d. Uterine abnormalities

44. One of the following doesn't cause hirsutism:

a. Cyproterone citrate

45. One of the following is correct about PCOS:

113
a. Ovarian morphology found in 20% of females

46. A 30 years old female presented with menorrhagia was found to have a 10*12 cm fibroid.
You would counsel her that:

a. Mirena is best for her

b. Uterine artery embolization is optimal if she wants to preserve fertility

c. Myomectomy is advisable

47. One of the following is NOT true about uterine artery embolization:

a. Submucous fibroid is not a contraindication

b. Premature ovarian failure is a complication

c. PID is a contraindication

48. One of the following is NOT a presentation of fibroid:

a. Dyspareunia (IMB in the other form.

49. A patient with severe acute PID. One of the following is appropriate in her mangement:

a. Laparascopy after 48 hours of appropriate IV antibiotics and no improvement .

50. One of the following is NOT correct about STDs:

a. Cervical warts mandate a cervical smear

b. Primary syphilis presents as pailess ulcer

c. Serology is usefull in Herpes dx

114
51. One of the following is NOT correct about candidal vulvovaginitis:

a. pH > 6

b. Intense itching

c. Common in pregnancy

52. One of the following is NOT correct about ectropion:

a. Maily present with IMB

53. HPV DNA testing is indicated in:

a. ASCUS

b. Atypical squamous cell. cannot exclude high grade lesion

c. Squamous cell carcinoma

54. A patient whose Pap smear showed high grade squamous intraepithelial lesion. Next step
is:

a. Colposcopy

b. Repaet Pap smear in 6 months

c. HPV DNA testing

55. One of the following is NOT correct about cervical cancer:

a. Chemotherapy can replace surgery in early stages .

115
56. One of the following is NOT correct about labor:

a. Prolonged labor due to hypotonic uterine contraction is usually associated with fetal
distress.

57. One of the following is correct about 3rd stage of labor:

a. Cord traction with no uterine contraction increase the risk of uterine inversion

58. One of the following is correct about cephalohematoma:

a. Ossification can occur and may lead to asymmetrical skull

59. One of the following is correct about vaccum delivery:

a. Can cause cephalohematoma

60. A primigravida in labor with cord prolapse and asymptomatic. The fetus presentation is
vertex. The best next step:

a. Emergent CS

b. Deliver vaginally (I can't remember the question exactly but she wasn't in eminent vaginal
delivery.

61. One of the following is NOT a complication of induction of labor:

a. Precipitate labor

b. Fetal distress

c. Uterine rupture

116
62. One of the following is NOT a prerequisite for instrumental delivery:

a. Epidural analgesia

b. Full cervical dilitation

63. One of the following is NOT correct about regional anesthesia:

a. Contraindicated in epilepsy

b. Can lead to postural headache

c. Can cause severe fetal bradycardia

* Note: Some answered it (a. because it's CI in neurological diseases and some answered (c.
because it could lead to maternal hypotenstion which if severe and prolonged could affect the
fetus. So…you pick!!

64.The most consistent sign in severe abruptio placenta is:

a. Oliguria

b. Ecchymosis

c. Fetal distress

*Note: the question was (the most consistent IN severe abruptio. and not (the most consistent
WITH severe abruptio.. I think the 1st phrase means the most constant sign in severe abruptio
which I think is oliguria whereas the second's phrase answer would be ecchymosis. SO
again...you pick!!

65. One of the following is NOT a late complication of CS:

a. Wound dehiscence .

117
66. A 34 weeks pregnant patient presented to the ER with history of heavy vaginal bleeding.
She's hemodynamically stable and dx to have placenta previa. The best next step is:

a. Rescusitate and deliver by CS

b. Induction of labor

c. Give steroid and deliver in 48 hours

d. Blood transfusion and conservative management till maturity is achieved

67. A 32 weeks pregnant patient whose dx to have PP presented with mild bleeding.The best
next step:

a. Establish an IV line and admit her

68. A 28 weeks pregnant patient whose BP is 140/95 and excretes 3 g/24 hours of protien in
urine. She's asymptomatic. This is:

a. Mild PET

69. One of the following is a sign of severe PET:

a. Hyperreflexia

70. A paregnant patient whose dx to have PET presented with seizures. What not to do:

a. Put her in left lateral position

b. Supplemental oxygen

c. Biophysical profile

d. Mg sulphate

118
71. A primigravida in labor. She's at term. Her ANC was with no problems. The most
appropriate for fetal monitoring is:

a. Fetal heart auscultation every 15.30 minutes

b. Continous CTG

72. One of the following is NOT in biophysical profile:

a. CTG

b. Fetal weight

c. Fetal gross movement

d. Fetal breathing .

73. A pregnant patient who's dx to have PET presents at term with blurred vision and BP of
150/100. The best next step is:

a. IV hydralazine and watch for symptoms

b. Give steroids and deliver in 48 hours

c. Arrange for delivery

74. A patient with chronic HTN. What not to do in ANC:

a. LFT

b. KFT

c. Urinalysis

d. Hepatits B screen

75. One of the following is correct about GDM:

119
a. You can decrease the risk of fetal anomalies to values closer to those of non.diabetics by
strict glycemic control

76. One of the following is INCORRECT about GDM:

a. Retinopathy is the 1st to improve with insulin therapy

77. α.fetoprotien is elevated in all of the following except:

a. NTD

b. Down's syndrome

78. One of the following isn't a cause of large for dates:

a. Down's syndrome

b. Anencephaly

c. Cleft palate

79. The most common cause of neonatal mortality is:

a. Prematurity

b. Congenital abnormalities

c. Birth asphyxia:

80. One of the following is NOT true about congenital abnormalities:

a. A major anomaly is present in 30 out of one thousand newborns

b. Some of them may not present at birth

120
c. The most common cause is genetic

81. One of the following is NOT presented per 1000:

a. MMR (maternal mortality rate.

82. One of the following is correct about IUGR:

a. Associated with oligohydramnios in 80%

83. One of the following is NOT correct about Rh.isoimmunization:

a. 1 ml is needed to induce a maternal response

84. One of the following is NOT a cause of delayed uterine involution:

a. Retained pieces of placenta

b. Non breastfeeding

c. Uterine fibroid

d. Full bladder

*Note: actually none of the answers is the answer soo...Wallah mani 3arfeh!!

85. One of the following is NOT a contraindication for breast feeding:

a. Mastitis

86. One of the following is NOT a cause of 2nd trimester miscariage:

a. Placenta previa

121
87. One of the following is INCORRECT about recurrent miscarriage:

a. Steroids are effective in cases of undefined etiology

88. One of the following is INCORRECT about recurrent miscarriage:

a. IVIG is effective in cases of undefined etiology .

89. One of the following is INCORRECT about preterm labor:

a. Cervical length of 2.5 cm is a risk factor

90. One of the following is not a significant finding in a pregnant at term:

a. Ankle edema

91. One of the following is INCORRECT about Ir defeciency anemia in pregnancy:

a. IV Ir can cause severe anaphylaxis

b. MCV <85 fL is diagnostic

92. One of the following is NOT part of your counselling for a pregnant lady travelling on plane:

a. Decrease fluid intake .

93. One of the following is NOT a part of routine booking visit investigations:

a. X.ray pelvimetry

94. A preganant patient is found to have asymtomatic bacteruria during ANC. Management is:

a. Oral A.B.s

95. Which of the following is not suitable for a trial of labor:

a. A patient with previous classical CS

122
96. A patient who's G2P1 presents at 35 weeks gestation with breech presentation. NO
abnormalities detected. She's not in labor. Management is:

a. Visit in 2 weeks

b. X.ray pelvimetry

97. The presenting diameter in a flexed occipito anterior fetus is:

a. Suboccipitobregmatic

98. The risk of PPH is highest in:

a. G3P2 with a 3.9 kg fetus

b. A primigravida with a twin pregnancy

99. One of the following is INCORRECT about labor:

a. X.ray pelvimetry can accurately predict the need for CS

100. One of the following is TRUE about IUGR:


a. 60% can be detected by serial fundal pubic hight measurement

101. One of the following is NOT correct about bacterial vaginosis:

a. It's treated with pinicillin

102. One of the following is a sex cord cell tumor:

a. Granulosa cell tumor

103. A 37 weeks gestation pregnant presents to the ER with mild vaginal bleeding. On US she's
dx to have PP and the placenta is reaching the cervical os. Next step is:

a. Deliver by CS

123
104. One of the following is correct about ovarian epithelial tumors:

a. Treated mainly by surgery

105. One of the following is NOT correct about gynecological surgeries:

a. Vaginal hysterectomy is indicated for large fibroids.

106. A primigravida presents with fully dilated cervix for 60 minutes, effective uterine
cintractions, head station is (.1., and severe caput succidenum.Next step is:

a. Deliver by CS

b. Wait another 30 minutes

107. One of the following statements about epidural anesthesia is INCORRECT:

a. Foley's catheter is optional


b. IV infusion of crystalooid prior to induction to insertion of epidural anesthesia .

124
10. 5th Year Final Exam 2010 :
1. Confirmative dx of ectopic pregnancy is:

a. laparoscope .

b. hysteroscopy

c. transvaginal exam

d. urinary b.HCG .

2. Common presentation of ectopic pregnancy is:

a. vaginal bleeding

b. lower abdominal pain

3. lab investigation that indicate ovulation:

a. progesterone level mid luteal*

b. LH level

c. FSH level

d. estrogen

4. regarding PCOS, one true:

a. most common cause of premature menopause

b. is present ultrasonagraphically in 20% of population* .

c. associated with increase insulin sensitivity

125
d. associated with (.. progesterone challenge test

e. no familial tendency .

5. one true about clomiphine citrate:

a. hot flushes

b. hirsutism

c. compete with progesterone at high centers

d. should be administered for 12 moths

e. reduce pituitary gonadtrophin secretion .

6. regarding bromocriptine, all true except:

a. can prevent lactation

b. its ergot alkaloid derivative .

7. most common cause of PPH:

a. uterine atony .

b. infections

c. coagulation defect

d. trauma .

8. all about uterine atony true, except:

a. full bladder is a cause

b. uterine massage cause lasting contraction .

126
9. polyhydromnios caused by all, except:

a. parvovirus infx*

b. renal agenesis

c. deudonal atresia

d. GDM

10. about active management of 3rd stage of labor, all true except:

a. oxytoxic drugs cause placental retention*

b. oxytocin reduces its duration

c. ergometrine C/I in heart disease

d. oxytocin cause hypotension .

11. all true about genital prolapsed, except:

a.1st degree uterine prolapsed descent of cervix within vagina

b. rectocele is prolapse of 2/3 of post vaginal wall &rectum

c. enterocele is true hernia

d. vault prolapsed commonly occur after vaginal hysterectomy

12. all can cause prolapsed, except:

a. repetitive c.section*

b. prolonged labor

c. instrumental delivery

127
d. chronic constipation .

13. all true about peuriperuim breast abscess, except:

a. staph is most common

b. diagnosis by mammogram*

c. need antibiotics

d. need surgical incision .

14. all true about bacterial vaginosis, except:

a. caused by lactobacillus*

b. cause of 2nd trimester abortion

c. ph >5

15. all are STD, except:

a. HSV .

b. BV .

c. trichomonas

d. gonorrhea

16. all cause vaginal candiasis, except:

a. DM

b. IUCD*

c. pregnancy

d. vaginal douches

128
17. all true, except:

a. syphilis primary lesion painless

b. recurrent HSV painless

c. genital warts C/I for vaginal delivery* .

18. all true about induction, except:

a. indicated in 15% of deliveries

b. oxytocin used for unfavorable cervix*

c. most common indication is postdate

d. ARM used for favorable cervix .

19. all C/I for induction, except:

a. antenatal bad CTG

b. transverse lie

c. growth restriction*

d. recent pelvic repair

20. about opioid pain relief during labor, all true, except :

a. cause of neonatal respiratory depression

b. cross placental barrier

c. mostly effective in 1st stage

d. morphine commonly used (pethidine) .

129
21. about epidural analgesia, all true except:

a. coagulation dis is a C/I

b. used with precaution in IUFD

c. should administer crystalline fluid before insertion

d. better pain relief than opioids .

22. about complete mole, all true except:

a. is tripleody .

b. no fetal tissue

c. risk of malignant sequel

d. safe to use OCP during follow up period

e. contraception during whole follow up .

23.about malignant GTN, all true except:

a. lung mets treated by radiotherapy

b. HCG is enough marker

c. treatment started after histopathological confirmation*(mostly.

d. chemotherapy sufficient cure .

24. diamniotic monochorionic twins, twin A has oligohydromnios, small size, twin B will have:

a. hypovolumia

b. reduced amniotic fluids

130
c. congestive heart failure .

25. single most imp. Factor in perinatal mortality in twins is:

a. prematurity .

b. cong. Anomalies .

c. growth restriction

d. abnormal lie .

26. 30 year , pregnant with twins, at term:

a. G2P2

b. G1P1

c. G1P1

27. relation of lowest bony part of presenting part to mother ischial spine:

a. fetal station .

28. when head is engaged, u can feel its lowest part at level of:

a. ischial spine (station 0 ) .

b. ischial tuberosity.

29. denominator of breech presentation:

a. sacrum .

b. coccyx

c. occiput

131
d. vertex .

30. false about mechanism of labor:

a. gynecoid pelvis bring vertex into occipititransverse while enter pelvic inlet .

b. shoulders should lie in AP diameter to be delivered into pelvic outlet .

c. flexion of fetus increase thru labor .

31. all indication for C/S, except:

a. transverse lie, cervix 3 cm dilated

b. placenta previa major degree

c. footling, ruptured membrane, 5cm dilated

d. cord presentation .

32. lab investigation not used for recurrent abortion:

a. thrombophilia scan

b. TORCH*( cause of abortion not of recurrent abortion) .

c.parents karyotyping

33. principle of management of septic abortion, include all except:

a. immediate evacuation of uterus*(I think bcz evacuation after 12 hrs of antibiotics.

b. IV antibiotics

34. 25 yrs, has missed abortion at 10 weeks, all should be discussed during counseling for post
abortion management, except:

132
a. Investigate cause of abortion

b. discuss most likely cause of abortion

c. contraception

d. avoid pregnancy for next 3 moths

35. not indication for anti D:

a. preterm labor*

b. cordocentesis

c. manual removal of placenta

36. about cervical incompetence, all true, except:

a. congenital cervical weakness is a cause

b. caused by previous multiple pregnancy .

c. diagnostic cone biopsy can cause it

d .tt by cervical cerclage in 13.15 weeks

37. not true about abortion:

a. cervical dilated in inevitable abortion

b. 20% of threatened abortion will progress to fetal loss .

c. US not essential for dx of incomplete

d. silent infx is most common cause .

38. during peuriperuim, one false:

133
a. external os stay open permanent

b. internal os close by 2nd day (by 1st week) .

c. lochia alba seen after 2nd week

d. lochia rubra seen in 1st few days

39. pregnant, 28 weeks, fundal height 22 weeks, all possible causes, except:

a. GDM .

b. placental insufficiency

c. wrong date

d. oligohydromnios .

40. 20 weeks, low lying placenta, ur action:

a. rescan weekly

b. rescan at 28 weeks .

c. plan for CS .

41. simple most imp. Factor in abruption placenta:

a. Pre.eclampsia

b. trauma

c. polyhydromnios

d. short cord .

42. about placenta previa, only one true:

134
a. increase risk of IUGR

b. increase risk of PPH*

c. abdominal US benefits but not accurate

d. vaginal exam safe .

43. all complication of progesterone mini pill, except:

a. breast tenderness

b. acne

c. irregular menstrual cycle

d. complex ovarian cyst (by exclusion) .

44. copper IUCD cause all, except:

a. dysmenorrhea

b. heavy menustruation

c. ectopic pregnancy

d. BV .

45. about sterilization, all true except:

a. sterilization is highly effective esp. in 1st 6.12 moths

b. tubal ligation done by rings or clips

c. antisperm ab is known complication

d. vas deference is …..(procedure of operation ) .

135
46. regarding lactation, all true except:

a. maintained by placental lactogen .

b. controlled by prolactin

c. suppressed by bromocripitine

d. human milk contain ptn less than cow milk .

47. tt for mild PID:

a. IV cephalosporin +gentamycin

b. IV cephalosporin+ IV metroniadazole

c. IV metroniadazole

d. doxycycline + metroniadazole orally .

48. definitive diagnosis of PID by:

a. laprascope .

b. U/S .

c. MRI .

d. hysteroscopy .

49. all correct about fibroids, except:

a. cervical fibroids are reassuring .

b. 3 types, subserosal, intramural, submucosa

c. subserosal is peduncleated

d. often are multiple

136
50. about fibroids & pregnancy, all true except:

a. hyaline degeneration is known complication& occur in 2nd trimester .

51. all radiological investigation for fibroid, except:

a. plain xray .

b. U/S .

c. MRI .

52. definitive diagnosis of endometriosis:

a. laprascopy .

b. lapratomy .

c. hysteroscopy .

d. histopathological .

53. all Sx of fibroids except :

a.urinary frequency

b.subfertility

c.menorrhagia

d.asymptomatic

e. All true .

54. about endometriosis , one is correct :

a. mostly present with menorrhagia

137
b. increase during pregnancy

c. tt affected by extent of disease*

d. no familial tendency .

55. One is correct about DUB

a. hypothyrodism is recognized cause

b. adenomyosis is cause

c. DUB occur in non ovulatory cycles

d. endometrial ablation used for tt .

56. menorrhagia can be treated by all except

a. mefanamic acid

b. GnRH analogue

c. progesterone

d. cupper IUCD .

57. lower abdominal pain in pregnant, all except:

a. cervical ecropion .

b. preterm labor

c. acute appendicitis

d. SCD

e. acute polyhydromnios .

138
58. about RH isoimmunization, all true except:

a. 500 IU antiD cover 5 ml blood .

b. liley’s chart & whitfeild action line are imp. In management .

c. all RH (.. people have “d” in each side of genotype

d. tends to be less severe in subsequent pregnancy .

59. all true for tt of anemia, except :

a. parentral iron to increase Hb rise*

b. S/E of oral ferrous is cause of reduced compliance

c. 0.4 mg daily folic acid is minimal requirement

d. iron increase level of Hb 0.8 mg every week .

60. all indication for delivery in PROM , except:

a. chorioamnionitis

b. fetal distress

c. abruptio placenta

d. multiple pregnancy .

61. causes of preterm, except:

a. abdominal surgery known trigger factor*

b. young maternal age

c. subclinical infection.

62 . C/I of ritodrine:

139
a. hyperthyroidism .

b. uterine atony .

c. GA 32 weeks .

d. ruptured membranes .

63. DM associated with all, except:

a. VSD

b. HTN .

c. fetal anal atresia

d. sacral agenesis is the commonest

64. hyperinsulinemia cause increase in fetal tissue, except:

a. heart

b. brain “not sure ‘ .

c. muscles

d. subcutaneous fat .

65. about HTN disorder of pregnancy, all true, except:

a. proteinuria & no HTN is hypertensive disorder of pregnancy .

b. proteinuria is 300 mg on 24 hr urine collection

c. proteinuria & HTN are imp. For dx of HELLP syndrome

d. hypotensive medication can't stop progression of pathology

140
66. about ANC, all true except:

a. reduce risk of hydrops fetalis

b. reduce risk of post term

c. urine dipstick routine every visit

e. pelvic exam is routine .

67. all correct about ANC, except:

a. air travel should be

b. rubella (–. lady should be immunized when deliver

c. exercise reduce need for induction

d. should take vit A .

68. all can be seen by US on 1st trimester, except:

a. placenta previa .

b. twin pregnancy .

69. about IUGR all true, except :

a. rare presentation of chromosomal defect .

b. caused by dieting in pregnancy .

c. caused by long standing DM .

70. about thyroid disorder in pregnancy, all true, except:

a. thyroid level is diagnostic*

141
b. clinical diagnosis is not significant

c. graves gets worse in postpartum

d. can cause preterm delivery

71. regarding epilepsy all true, except:

a. 3% risk of congenital anomaly*

b. 1/3 of pts have increased seizure frequency

c. monotherapy is gold standard

d. give folic acid .

72. all increase in DIC, except:

a. fibrin degradation product

b. PT

c. APTT

d. fibrinogen .

73. Patient with 5 cm simple cyst:

a. observe for 3 months to regress .

b. remove by laprascopy

c. lapratomy & cystectomy

d. TAHBSO .

74. a feto.protein is tumor marker of:

a. endodermal sinus tumor .

142
‫‪75. all prognostic factor for endometrial CA, except:‬‬

‫‪a. age‬‬

‫‪b. size‬‬

‫‪c. grade‬‬

‫‪d. presence or absence of fibroids .‬‬

‫بذرة الشر تهيج ‪ ،‬ولكن بذرة الخير تثمر ‪ ،‬إن األولى ترتفع في الفضاء سريعاً ‪ ،‬ولكن جذورها‬
‫في التربة قريبة حتى لتحجب عن شجرة الخير النور والهواء ‪ ،‬ولكن شجرة الخير تظل في‬
‫نموها البطيء ‪ ،‬ألن عمق جذورها في التربة يعوضها عن الدفء والهواء ‪....‬‬
‫مع أننا حين نتجاوز المظهر المزور البراق لشجرة الشر ونفحص عن قوتها الحقيقية‬
‫وصلبتها تبدو لنا واهنة هشة نافشة في غير صلبة حقيقية على حين تصبر شجرة الخير‬
‫على البلء ‪ ،‬وتتماسك للعاصفة وتظل في نموها الهادئ البطيء‬
‫َّل تحفل بما ترجمها به شجرة الشر من أقذاء وأشواك ‪...‬‬

‫الفكرة التي تحيا على نبض قلوبكم‬

‫وشكراً عميقاً لكل من ساهم في إنجاز هذا المشروع الرائع ‪):‬‬

‫‪143‬‬

Vous aimerez peut-être aussi